Está en la página 1de 201

CIRUGÍA LA MEDIDA PREVENTIVA PARA ESTA

COMPLICACIÓN:
FEMENINO DE 42ª DE EDAD A QUIEN SE LE USO DE MEDIAS PLASTICAS
REALIZO COLICISTECTOMIA ABIERTA POR
COLECISTITIS CRÓNICA LITIASICA AL SALIR FEMENINO DE 48ª MULTIPARA, OBESA,
DE QUIRÓFANO TIENE COLOCADA UNA PROGRAMADA PARA HISTERECTOMÍA
SONDA NASOGASTRICA, SONDA VESICAL, VAGINAL POR PROLAPSO UTERINO, COMO
VENOCLISIS CON HARTMAN GLUCOSADA ANTECEDENTE MENCIONA SER DIABÉTICA Y
RESPIRACIÓN 18 X MIN. PULSO 75X´. TA PRESENTA INSUFICIENCIA VENOSA DE
130/85. TEMP 36.5° ESTE PACIENTE DEBERA MIEMBROS INFERIORES, CUAL DE LAS
DEAMBULAR: SIGUIENTES COMPLICACIONES
AL DIA SIGUIENTE DE LA CIRUGÍA PULMONARES ES PROBABLE QUE
PRESENTE ESTA PACIENTE EN EL
PACIENTE DE 56ª A QUIEN SE LE REALIZO POSTOPERATORIO:
ANTRECTOMIA CON RECONSTRUCCIÓN TROMBOEMBOLIA PULMONAR
BILIAR POR PRESENTAR ULCERA GÁSTRICA
PENETRADA A PÁNCREAS SE ENCUANTRA EN FEMENINO OBESA DE 35ª LA CUAL VA A SER
EL 3ER DIA POSTOPERATORIO CON SONDA SOMETIDA A COLECISTECTOMIA POR
NASOGASTRICA, SONDA VESICAL CON PRESENTAR COLECISTITIS CRÓNICA
HARTMAN GLUCOSADA Y CIPROFLOXACINO LITIASICA ADEMÁS DE TRATAMIENTO
500MG IV CADA 12 HRS, RESP 16X/ PULSO 75X ANTIRREFLUJO POR INCOMPETENCIA DEL
TA 130/85. CUANDO SE DEBERA RETIRAR LA ESFÍNTER ESOFÁGICO INFERIOR, LAS
SONDA DE LEVIN EN ESTE PACIENTE: ORDENES PREOPERATORIOS DEBERAN
AL PRESENTAR RUIDOS INTESTINALES INCLUIR:
AUTORIZACIÓN DE LA CIRUGÍA
INGRESA MASCULINO DE 22 AÑOS AL
SERVICIO DE URGENCIAS REFIRIENDO FEMENINO DE 40ª CON SIRPA Y
DOLOR EN FOSA ILIACA DERECHA. SE DESNUTRICIÓN PROTEICO CALORICA EN LA
DIAGNOSTICA APENDICITIS Y SE REALIZA ALIMENTACIÓN INTRAVENOSA PERIFERICA
APENDICECTOMIA. EL HALLAZGO EL INCREMENTO DE CALORÍAS SE LOGRARA
QUIRÚRGICO FUE APENDICITIS EN FASE MEDIANTE:
EDEMATOSA AL MOMENTO DE SU EGRESO LÍPIDOS AL 10%
HOSPITALARIO SE LE RETIRARAN LOS
PUNTOS A LOS: PACIENTE DE 50ª CON CANCER DE ESÓFAGO
7 DIAS Y CON INANICIÓN EN ESTA CIRCUNSTANCIA
SU CORAZON Y MÚSCULO TIENEN
MASCULINO DE 22ª EL CUAL REFIERE DOLOR PREDILECCIÓN POR UNO DE LOS SIGUIENTES
EN FOSA ILIACA DERECHA EL PACIENTE FUE AMINOÁCIDOS:
SOMETIDO A APENDICECTOMIA EL DE CADENA RAMIFICADA
HALLAZGO QUIRÚRGICO FUE APENDICITIS
PERFORADA (FASEIV) QUE TIPO DE CIERRE MASCULINO DE 36ª QUIEN DURANTE LA
PARA LA PIEL Y TEJIDO CELULAR INANICIÓN AGUDA DEVIDO A CANCER
SUBCUTÁNEO ES RECOMENDADO EN ESTE LARINGEO. TIENE UNA PERDIDA NETA DE
CASO: AMINOÁCIDOS, YA QUE LA SINTESIS DE
DIFERIDO PROTEINA MUSCULAR DESAPARECE O
DESCIENDE TANTO QUE EL CATABOLISMO
FEMENINO DE 38ª MULTIPARA, CON DE LA MISMA PERMANECE SIN CAMBIOS
CISTOCELE PROGRAMADA PARA ESTA SITUACIÓN ES DEVIDA A:
COLPOPERINEOPLASTIA LA PREPARACIÓN CONCENTRACIONES BAJAS DE INSULINA
DE LA VAGINA SE RELIZA EN:
QUIRÓFANO FEMENINO DE 28ª POLITRAUMATIZADO QUE
SE ENCUENTRA EN EL POSTOPERATORIO
MASCULINO DE 22ª SOMETIDO A QUE PESA 90KG TALLA 1.5M, QUE INGIERE
LAPAROTOMIA CON HALLAZGO DE 10GM DE PROTEINAS AL DIA.EN 2 LITROS DE
PERFORACIÓN DE ILEON POR ORINA ESTA ELIMINANDO 950MG/ DIA, DE
SALMONELLOSIS AL 3ER DIA NITRÓGENO UREICO. CON ESTO EL BALENCE
POSTOPERATORIO INICIA CON FIEBRE, DE NITRÓGENO AL DIA ES DE:
DOLOR, TUMEFACCIÓN, HIPEREMIA Y 21.4GM
AUMENTO DE LA TEMPERATURA LOCAL.
CUAL DE LAS SIGUIENTES ES LA CAUSA PACIENTE QUE FUE INTERVENIDO
PROBABLE CON ESTA COMPLICAION: QUIRÚRGICAMENTE REALIZÁNDOSELE
CIERRE PRIMARIO DE LA HERIDA COLECISTECTOMIA POR PRESENTAR
COLECISTITIS CRÓNICA LITIASICA, CON EL
MASCULINO DE 72ª OBESO, SOMETIDO A POSTOPERATORIO MEDIATO PRESENTO
COLECISTECTOMIA ABIERTA POR DEHISCIENCIA DE LA HERIDA QUIRÚRGICA.
COLECISTITIS AGUDA AL 4TO, DIA POST PARA REPARAR LA COMPLICAION Y CERRAR
OPERATORIO PRESENTA EDEMA. DOLOR, LA APONEUROSIS ELIGIRA EL SIGUIENTE
HIPERTEMIA Y COLORACIÓN VIOLACEA EN MATERIAL DE SUTURA:
EL MIEMBRO INFERIOR DERECHO, CUAL ES PROLENE 1
MASCULINO DE 45ª EL CUAL SUFRIO DE AUTOMOVILISTICO CON LESIONES LEVES.
TRAUMATISMO ABDOMINAL EN ACCIDENTE PERO FUE VICTIMA DE CRISIS EMOCIONAL,
AUTOMOVILISTICO Y QUIEN A SU INGRESO A SE LE TOMA GASOMETRIA REVELANDO UNA
URGENCIAS LLEGA EN ESTADO DE CHOQUE PCO2 DE 30MMHG TAL CIFRA Y EL CUADRO
HIPOVOLEMICO PARA QUE DESENCADENE CLINICO SUGIEREN:
LA RESPUESTA BIOLÓGICA AL TRAUMA ALCALOSIS RESPIRATORIA
DEBERA EXISTIR:
INTEGRACIÓN NOCICEPTIVA EN LE CEREBRO ANTE UN PACIENTE EL CUAL PRESENTA
VOMITOS FRECUENTES DE CONTENIDO
FEMNINO DE 34ª QUIEN SUFRIO GÁSTRICO SECUNDARIO A ESTENOSIS
POLITRAUMATISMO EN ACCIDENTE PILORICA LA ANORMALIDAD METABÓLICA
AUTOMOVILISTICO Y QUE ES VISTO EN EL PREDOMINANTE QUE ESPERARIA
SERVICIO DE URGENCIAS. EN EL ESTUDIO DE ENCONTRAR SERIA:
ESTE PACIENTE SE SABE QUE DENTRO DE LA ALCALOSIS HIPOCLOREMICA
RESPUETA METABÓLICA AL TRAUMA SE
PRODUCE UNA CERIE DE HORMONAS MASCULINO DE 22ª QUIEN SE CAYO DESDE
LLAMADAS CONTRARREGULADORAS DOS UN CUARTO PISO SE LE REALIZO
DE LAS CUALES SON: HEPATECTOMIA PARCIAL, RESECCION DE
EPINEFRINA Y NORADRENALINA 30CM DE INTESTINO DELGADO ASI COMO
ESPLENECTOMIA. RECIBIO 15UNIDADES DE
SANGRE EN EL PRE Y TRANS OPERATORIO. 8
LITROS DE SOLUCION HARTMAN Y 4
UNIDADES DE PLASMA FRESCO CONGELADO
ANTE UN PACIENTE QUE SE ENCUENTRA DURANTE EL CIARRE DE LA LAPAROTOMÍA
CURSANDO EL PRIMER DIA DEL SE NOTO SANGRADO DIFUSO. DE LAS
POSTOPERATORIO POR HABER SIDO SIGUIENTES CAUSAS CUAL ES LA MAS
SOMETIDO A TIREIDOCTOMIA SUBTOTAL PROBABLE DE ESTE SANGRADO:
POR PRESENTAR BOCIO MULTINODULAR PLAQUETOPENIA
UNA VEZ SUPERADA LA FASE EBB, USTED
ESPERA QUE EL PACIENTE SE ENCUENTRE EN FEMENINO DE 28ª LA CUAL PRESENTO
LA FASE: TRAUMA ABDOMINAL CERRADO CON
DE FLUJO RUTURA HEPÁTICA E ISQUEMIA INTESTINAL
POR DESGARRO DEL MESENTERIO, DURANTE
FEMENINO DE 30ª QUIEN SUFRE ACCIDENTE UNA INTERVENCIÓN QUIRÚRGICA PRESENTA
EN CARRETERA. EN EL LUGAR DEL SANGRADO DIFUSO DE LA
ACCIDENTE SE LE ENCUENTRA PARED ABDOMINAL Y EN HIGADO DEBIDO A
SEMICONSCIENTE, TAQUICARDIA, EN UNA PROBABLE CID, EL TRATAMIENTO QUE
TORAX PULMONES VENTILADOS, ABDOMEN RELIZARA EN ESTE MOMENTO SERA:
CON PARISTALSIS DISMINUIDA Y DOLOR EMPAQUETAMIENTO ABDOMINAL
GENERALIZADO. SE LE CANALIZA CON
HARTMAN Y ES LLEVADO AL CENTRO DE FEMENINO DE 39ª CON DOLOR EN
TRAUMA MAS CERCANO. DE LAS HIPOCONDRIO DERECHO SE REALIZA USG DE
SIGUIENTES CUAL ES LA QUE DISMINUYE EN PÁNCREAS. HIGADO Y VIA BILIAR Y SE
LA FASE INICIAL EN LA RESPUESTA DETECTA ENFERMEDAD CALCULOSA DE LA
BIOLÓGICA AL TRAUMA: VESÍCULA POR LO QUE FUE SOMETIDA A
CORTISOL CELECISTECTOMIA SIMPLE SE SOLICITAN
EXAMENES PREOPERATORIOS LOS CUALES
MASCULINO DE 68ª EL CUAL FUE SOMETIDO SON NORMALES, SI USTED DESEA DETECTAR
A RESECCION ABDOMINOPERINEAL POR DEFICIENCIA DEL FACTOR II CUAL ES EL
PRESENTAR CANCER DE RECTO, DURANTE ESTUDIO UTIL PARA EVALUARLO:
LA CIRUGÍA LA PERDIDA SANGUÍNEA FUE TIEMPO DE PROTROMBINA
RESPUESTA VOLUMEN A VOLUMEN, EN EL
POSTOPERATORIO FUE MANEJADO CON MASCULINO DE 42ª QUIEN INGRESA CON
SOLUCIONES DE CLORURO DE SODIO AL SANGRADO DE TUBO DIGETIVO ALTO.
0.45% PRESENTANDO DEBILIDAD, LETARGIA, ENTRE SUS ANTECEDENTES MENCIONA
CONVULSIONES Y POSTERIORMENTE COMA ALCOHOLISMO CRÓNICO. LABORATORIO:
POR LO REFERIDO EL PACIENTE ESTA HB 10GM/ DL, HTO 33%, PLAQUETAS 300,000.
CURSANDO CON: TPT 40´. A LA EXPLORACIÓN FÍSICA
HIPONATREMIA PACIENTE TRANQUILO; CONSIENTE TA.
100/60, SE COLOCA SONDA NASOGASTRICA Y
PACIENTE QUE PRESENTO TRAUMATISMO SE LE REALIZA LAVADO GÁSTRICO, DENTRO
ABDOMINAL CERRADO Y CHOQUE DEL MANEJO DE ESTE PACIENTE INDICA
HIPOVOLEMICO, EL PUNTO DE VISTA TRANSFUSIÓN DE :
FISIOPATOLOGICA A NIVEL HORMONAL, CONCENTRADO PLAQUETARIO
ESPERARIA ENCONTRAR AUMENTO DE LA
SECRECION DE LA SIGUIENTE HORMONA: PACIENTE EL CUAL FUE SOMETIDO A
CORTISOL HEMICOLECTOMIA POR CANCER DE COLON.
EN EL POSTOPERATORIO PRESENTA CUADRO
FEMENINO DE 25ª QUIEN SUFRIO ACCIDENTE COMPATIBLE CON CID LA ALTERACIÓN
LABORATORIAL QUE ESPERARIA COMPORTAMIENTOS DE LOS LIQUIDOS
ENCONTRAR SERA: CORPORALES ES:
PRESENCIA DE P. D. F NA +

MASCULINO DE 45ª A QUIEN SE LE REALIZO FEMENINO DE 40ª LA CUAL INGRESO A LA


CESAREA Y RESECCION DE 30CM DE UCI POR PRESENTAR CHOQUE SÉPTICO
INTESTINO DELGADO POR TRAUMATISMO SECUNDARIO A UNA PERFORACIÓN
ABDOMINAL CERRADO. DURANTE LA INTESTITAL. SE LE HAN APLICADO
CIRUGÍA SE LE TRANSFUNDIERON 10 VOLÚMENES ADECUADOS DE RINGER
UNIDADES DE SANGRE DEL BANCO. A LACTADO CONTROLADOS POR LA PRESION
PRESENTADO SANGRADO DIFUSO Y PIENSA EN CUÑA PULMONAR, DOPAMINA Y
EN CID, ESTA COMPLICAION SE NOREPINEFRINA A PESAR DE LAS MEDIDAS
EVIDENCIARA POR EL SIGUIENTE ANTERIORES LA PACIENTE NO HA
RESULTADO DE LABORATORIO: MOSTRADO MEJORIA, ES MAS SE HA
PRESENCIA DE PRODUCTOS DE DIVISIÓN DE AGREGADO ICTERICIA Y CONFUCION
FIBRINA MENTAL. POR LO QUE MUESTRA EL
PACIENTE ESTA PRESENTANDO:
FEMENINO DE 28ª SE LE REALIZO CESAREA SÍNDROME DE DISFUNCIÓN ORGANICA
POR PRESENTAR OBITO DE 24 SEMANAS DE MÚLTIPLE
GESTACIÓN Y A QUIEN EN EL
POSTOPERATORIO SE LE DIAGNOSTICO CID. MASCULINO DE 55ª EL CUAL FUE SOMETIDO
CON EL OBJETO DE REPONER LA MAYOR A GASTRECTOMÍA SUBTOTAL POR
CANTIDAD DE LOS FACTORES DE LA PRESENTAR CANCER IN SITU EN EL
COAGULACION CONSUMIDOS, CUAL DE LAS POSTOPERATORIO MEDIATO ES
SIGUIENTES MEDIDAS ES LA MAYOR FUENTE INDISPENSABLE EL USO DE ANELGESICOS
DE LOS FACTORES DE LA COAGULACIÓN ´´. CON HORARIO, PARA PREVENIR LA
PLASMA FRESCO CONGELADO SIGUIENTE COMPLICACIÓN:
SÍNDROME DE DISFUNCIÓN ORGANICA
MÚLTIPLE
* MASCULIJNO DE 53ª QUIEN INGRESO A UCI
POR PRESENTAR HIPOTENCION SEVERA CON
SIFRAS TENCIONALES DE TA 70/40. MASCULINO DE 55ª EL CUAL FUE SOMETIDO
REVELADO A LA APLICAION DE VOLÚMENES A GASTRECTOMÍA SUBTOTAL POR
ADECUAOS DE RINGE LACTATO E PRESENTAR CANCER IN SITU EN EL
INOTROPICOS, ADEMÁS DE FIEBRE E POSTOPERATORIO MEDIATO ES
INSUFICIENCIA RENAL, SI FINALMENTE CON INDISPENSABLE EL USO DE ANALGÉSICOS
LAS MEDIDAD ANTERIORES LA TA NO SE CON HORARIOS, PARA PREVENIR LA
RECUPERA USTED OPTARIA POR INDICAR: SIGUIENTE COMPLICACIÓN:
APLICACION DE VASOPRESINA ATELECTASIA

MASCULINO DE 23ª QUIEN PRESENTA DESDE


HACE 3 DIAS, ICTERICIA, FIEBRE, DOLOR
ABDOMINAL, HECES HIPERPIGMENTADAS Y
FEMENINO DE 28ª QUIEN INGRESA POR ORINA CLARA. A LA EXPLORACIÓN EXISTE
SANGRADO TRANSVAGINAL DOLOR A LA PALPACIÓN DELCUADRANTE
DETERMINÁNDOSE COMO CAUSA ABORTO IZQUIERDO DONDE SE PALPA UNA MASA
INCOMPLETO EN EL PRIMER TRIMESTRE POR QUE SOBRE SALE DEL BORDE COSTAL Y
LO QUE SE RELIZA LEGRADO UTERINO A SIGUE LOS MOVIMIENTOS RESPIRATORIOS,
CONSECUENCIA DE ELLO LA PACIENTE ESTA ESTE CUADRO SE REPITE UNA O DOS VECES
EN CHOQUE HIPOVOLEMICO LA DIFERENCIA AL AÑOS Y SE PRESENTA TAMBIEN EN 2
DE O2 ARTERIO VENOSO EN PULMON EN LA HERMANOS SUYOS, EL DIAGNOSTICO
FASE TEMPRANA DEL CHOQUE PROBABLE ES:
HIPOVOLEMICO SE ENCUENTRA: HIPERESPLENISMO
PEQUEÑA DIFERENCIA DEL O2
ARTERIOVENOSO
MASCULINO DE 47ª EL CUAL HA PADECIDO
MASCULINO DE 53ª QUE INGRESO A LA UCI DE DOLOR ABDOMINAL RECURRENTE
POR PRESENTAR ESTABILIZACIÓN DEL DURANTE LOS ULTIMOS 3 AÑOS.
CHOQUE HIPOVOLEMICO Y MEJOR ACTUALMENTE PRESENTA DOLOR
CONTROL, POR LO QUE SE COLOCA UN EPIGASTRICO INTENSO Y CONTINUO,
CATETER DE SWANZ GANZ PARA VALORAR: NAUSEAS DE 6 HORAS DE VOMITOS QUE NO
P. V. C ALIVIAN EL DOLOR EN LOS ULTIMOS
MOMENTOS EL DOLOR TIENDE A
MASCULINO DE 60ª A QUIEN SE LE REALIZO IRRADIARSE AL DORSO Y HEMIABDOMEN
RESECCION INTESTINAL EN EL IZQUIERDO A LA EXPLORACIÓN PRESENTA
POSTOPERATORIO INMEDIATO EL RESISTENCIA ABDOMINAL EPIGASTRICA.
ELECTROLITO QUE REFLEJA MEJOR LA CON LOS DATOS
ALTERACIÓN EN LA COMPOSICIÓN DE LOS
ANTERIORES EL PADECIMENTO SERA:
PANCREATITIS AGUDA FEMENINO QUE ABSRCA UNA QUEMADURA
QUE ABRCA UNA SUPERFICIE CORPORAL
MASCULINO DE 47ª EL CUAL HA QUEMADURA EN TODA LA CARA ANTERIOR
PRESENTADO DOLOR ABDOMINAL DE TIPO DEL TRONCO, MIEMBRO SUPERIOR
ARDOSO DESDE HACE 3 AÑOS, QUE LO DERECHO, MANO IZQUIERDA EN LA CABEZA
DESPIERTA POR LAS NOCHES, DESDE HACE QUEMADURAS EN CEJAS, PESTAÑAS Y
UN AÑO TOMA DE FORMA IRREGULAR VIBRISAS QUEMADAS Y EXPECTORACION
ANTIÁCIDOS Y BLOQUEADORES H2 CARBONEADA ADEMÁS DE INCONCIENCIA.
PRESCRITOS POR FACULTATIVO, SIN SEGÚN ELCALCULO DE LAS REGIONES
MEJORIA APARENTE, ACTUALMENTE EL LESIONADAS. POR SU SUPERFICIE
DOLOR ES CONTINUO, MAS INTENSO Y SE CORPORALEQUIVALE A:
IRRADIA AL DORSO. LA TERAPEUTICA 50-60%
ADECUADA ES:
GASTRECTOMÍA PARCIAL MASCULINO DE 30ª QUIEN PRESENTA
QUEMADURAS SECUNDARIAS A EXPOSICIÓN
MASCULINO DE 22ª INGRESA AL SERVICIO DE DE FUEGOS PIROTÉCNICOS EN CARA,
URGENCIAS EN DONDE USTED SE MANOS, GENITALES, TORAX 70% PARA
ENCUENTRA, REFIRIENDO DOLOR EN FOSA VALORAR LA PERFUSION RENAL DURANTE
ILIACA DERECHA POR LO QUE SE LE EL TRATAMIENTO DE REPOSICIÓN DE
DIAGNOSTICA APENDICITIS, DE LOS LIQUIDOS:
SIGUIENTES EXMENES PREOPERATORIOS DEBERA VIGILARSE DIURESIS HORARIA A
CUALES SERIAN DE IMPORTANCIA : RAZON DE 30-50
RX SIMPLE DE EBDOMEN
TP Y TPT MASCULINO DE 30ª CON QUEMADURAS DEL
BIOMETRÍA HEMATICA 60% DE SEGUNDO GRADO SUPERICIALES Y
QUÍMICA SANGUÍNEA PROFUNDAS. EL CUAL HA ESTABILIZADO
HEMODINAMICO, LA DIETA DEBE HACERSE
MASCULINO DE 67ª PROSTATICO DIABÉTICO, POR LA VIA:
OBESO Y CON EPC, QUIEN HACE UN MES ORAL
EMPEZO A NOTAR MASA TUMORAL EN REGIO
INGUINAL DERECHA QUE DESAPARECE PACIENTE QUE SUFRE ACCIDENTE
RAPIDAMENTE AL ADOPRTAR EL DECUBITO AUTOMOVILISTICO Y QUE AL EXAMEN SE
DORSAL PARA MANEJAR ESTAS ENCUENTRA DESPIERTO Y RESPONDE A LAS
CONDICIONES SIGUIENTES EL TRATAMIENTO PREGUNTAS A NIVEL DE TORAX SE
INDICADO EN ESTE PACIENTE ES: ADOPTAR ENCUANTRAN DATOS QUE SUGIEREN
EL USO DE BRANGUERO NEUMOTORAX A TENSIÓN EN HEMITORAX
DERECHO, LA MEDIDA A APLICAR PARA ESTE
PACIENTE DE 50 AÑOS A QUIEN SE LE PROBLEMA SERA: TORACOCENTESIS EN 2°
PRACTICO LAPAROTOMÍA MEDIA ESPACION INTERCOSTAL
EXPLORADORA PRESENTA EN EL 5° DIA DEL
POSTOPERATORIO UNA EVICERACION. ESTA UN PACIENTE POLITRAUMATIZADO QUIEN
COMPLICACIÓN SE DEFINE COMO: EN EL EXAMEN LE ENCONTRAMOS EN
SEPARACIÓN DE TODOS LOS PLANOS DE LA HEMITORAX IZQUIERDO SILENCIO
HERIDA RESPIRATORIO A LA PERCUSIÓN
TIMPANISMO Y CON CHOQUE DE LA PUNTA A
LA DERECHA REALIZÁNDOSELE
PACIENTE DE 45ª EL CUAL PRESENTA UNA TORACOCENTESIS DE URGENCIA, LA
MASA EN REGION INGINAL IZQUIERDA JUSTIFICACIÓN DE ESTA MEDIDA ES POR:
DESDE HACE 3 AÑOS LA CUAL SE HACE ES LA DE ELECCIÓN PARA MANEJAR LA
EVIDENTE AL ESFUERZO Y DISMINUYE CONFUSIÓN PULMONAR
CON EL REPOSO. NO LLEGANDO AL ESCROTO
EN LA EXPLORACIÓN AL PEDIRLE QUE PUJE MASCULINO DE 34ª LLEVADO A URGENCIAS
LA MASA HACE CONTACTO CON EL PULPEJO POR SUFRIR ACCIDENTE AUTOMOVILISTICO
DEL DEDO EXPLORADOR EN ESTE CASO EL AL EXAMEN ENCONTRAMOS
DEFECTO SE ENCUENTRA EN: HIPERSENSIBILIDAD LEVE EN ABDOMEN Y
DENTRO DEL TRIANGULO DE HASSELBASH HEMITORAX DERECHO CON PERISTALSIS
MUY DISMINUIDA Y AUSENCIA DE METIDEZ
HEPÁTICA CUAL ESTUDIO DIAGNOSTICO:
FEMENINO DE 45ª QUIEN REFIERE LA RUPTURA DE VICERA HUECA
PRESENCIA DE UNA MASA A NIVEL
ABDOMINAL. NO DOLOROSA. A LA EF SE MASCULINO DE 41ª EL CUAL SUFRIO
ENCUENTRA MASA DE 2X2. POCO MOVIL, ACCIDENTE AUTOMOVILISTICO EN EL
BLANDA, NO DOLORSA, ENTRE APÉNDICE MOMENTO DE ARRIBAR A LA AMBULANCIA
XIFOIDES Y CICATRIZ UMBILICAL. EN LA USTED DETECTA QUE EL, PACIENTE SE
LINEA MEDIA, EN ESTE CASO CUAL SERIA LA ENCUENTRA SEMICONSCIENTE CON
TERAPEUTICA MAS ADECUADA: CIANOSIS PERIFERICA, LESION
PLASTIA MAXILOFACIAL IMPORTANTE.
MOVIMIENTOS RESPIRATORIOS APENAS
PERCEPTIBLES Y LA PRESENCIA DE
FRACTURA EXPUESTA DE TIBIA Y PERONE, CIRCUNSTANCIAS DESDE EL PUNTO DE
CONSIDERANDO LAS CONDICIONES EN LAS VISTA LABORATORIAL PODEMOS
QUE SE ENCUANTRA EL PACIENTE CUAL ES CORRABORAR NUESTRO DIAGNOSTICO SE
LA PRIMER MEDIDA QUE APLICARIA: LA ALBÚMINA SE ENCUANTRA DENTRO DEL
CRICOTIROIDOSTOMIA SIGUIENTE RANGO:
1.5-20G/DL
MASCULINO DE 60 AÑOS QUE ES INGRESADO
A CIRUGÍA PARA LAVADO DE CAVIDAD PACIENTE QUE SUFRIO QUEMADURA DE 2°
PERITONEAL POR SEPSIS ABDOMINAL EN QUE ABARCAN EL 30% DE LA SUPERFICIE
QUIRÓFANO. PRESENTA PARO CORPORAL EN ESTA PACIENTE EL
CARDIORRESPIRATORIO, A LA EXPLORACIÓN INCREMENTO DE LAS NECESIDADES
FÍSICA RUIDOS CARDIACOS POCO AUDIBLES, CALORICAS BASALES SERAN EN UN % DE
FRECUANCIA RESPIRATORIA NO 100-200
PERCEPTIBLE. NO HAY PULSOS EN ANTE UN PACIENTE POLITRAUMATIZADO Y
CAROTIDAS Y FEMORALES, CON ESTOS QUE FUE SOMETIDO A LAPAROTOMÍA
DATOS CLINICOS ES DETERMINANTE PARA EXPLORADORA PODEMOS ENCONTRAR UNA
EL DIAGNOSTICO DE PARO: DIFERENCIA QUE NO EXISTE EN EL PACIENTE
PERDIDA DE MOVIMIENTOS RESPIRATORIOS SOMETIDO A AYUNO CUAL ES ESTA?
ACTIVACION DEL SITEMA
MASCULINO DE 29ª PRESENTA DOLOR NEUROENDOCRINO
ABDOMINAL EN FID. CONTINUO,
ACOMPAÑADO DE HIPERTEMIA Y NAUSEAS,
A LA EXPLORACIÓN FÍSICA , MC BURNEY (+),
ROVSING (-), POR LO QUE ES PROGRAMADO
A CIRUGÍA, UNA VEZ QUE USTED SE VISTIO EN UN PACIENTE POLITRAUMATIZADO SE
CON ROPA QUIRÚRGICA PARA INGRESAR AL PRODUCE UNA ACTIVACION DEL PROBLEMA
AREA GRIS ADEMÁS USTED DEBE NEUROENDOCRINO
COLOCARSE: CUYA REPERFUSION METABÓLICA
BOTAS NUTRICIONAL PRODUCE:
ACELERACIÓN DEL CATABOLISMO PROFEICO
FEMENINO DE 35ª QUIEN VA A SER SOMETIDA MUSCULAR
A COLECISTECTOMIA POR COLECISTITIS
CRÓNICA LITIASICA SIN NINGUN MASCULINO DE 35ª EL CUAL SE ENCUENTRA
ANTECEDENTE PATOLÓGICO DE EN LA UCI, CON DIAGNOSTICO DE
IMPORTANCIA, LAS ORDENES PANCREATITIS AGUDA, DESDE HACE 10 DIAS
PREOPERATORIOS DEBERAN INCLUIR: ACTALMENTE EL ORGANISMO TIENE ALTOS
FIRMA DE LA AUTORIZACIÓN DE LA CIRUGÍA NIVELES DE INSULINA

MASCULINO DE 22ª INGRESA AL SERVICIO DE FEMENINO DE 28ª CON SEPSIS ABDOMINAL A


URGENCIAS REFIRIENDO DOLOR EN FID. EL LA EF PACIENTE SOMNOLIENTA. SIGNOS
HALLAZGO QUIRÚRGICO FUE APENDICITIS Y VITALES: 80/60, FC120X, FR36X, SU
SE REALIZA TRATAMIENTO INMEDIATO
APENDICECTOMIA EN FASE 1 (EDEMATOSA) CONSISTE EN:
AL MOMENTO DE SU EGRESO HOSPITALARIO CONTROL DE LA INFECCIÓN Y
LO CITA PARA EL RETIRO DE PUNTOS A LOS: MANTENIMIENTO DEL
7 DIAS VLUMEN SANGUÍNEO

MASCULINO DE 60ª CON DIAGNOSTICO DE MASCULINO DE 55 EL CUAL FUE SOMETIDO A


HERNIA INCICIONAL, INFRAUMBILICAL GASTRECTOMÍA SUBTOTAL POR CANCER IN
GIGANTE, SE PROGRAMA CIERRE DEL SITU. EN EL POSTOPERATORIO MEDIATO ES
DEFECTO HERNIARIO, PARA EVALUAR SU INDISPENSABLE EL USO DE ANALGÉSICOS
RIESGO QUIRÚRGICO DEBEMOS EVALUAR: CON HORARIOS, PARA PREVENIR LA
HISTORIA CLINICA SIGUIENTE COMPLICACIÓN:
ATELECTASIA
MASCULINO DE 72ª OBESO, SOMETIDO A
COLECISTECTOMIA ABIERTA POR MASCULINO DE 23ª QUIEN REFIERE DOLOR
COLECISTITIS AGUDA, AL 4TO. DIA DE POST ABDOMINAL DE PREDOMINIO EN
OPERATORIO PRESENTA EDEMA, DOLOR, EPIGASTRICO Y FID, PARA POSTERIORMENTE
HIPERTERMIA Y COLORACIÓN VIOLACEA EN IRRADIARSE A TODO EL ABDOMEN CON
MIEMBRO INFERIOR DERECHO, CUAL ES HIPERTERMIA Y VOMITOS. A LA EF EN
LA MEDIDA PREVENTIVA PARA ESTA ABDOMEN PERISTALSIS AUSENBTE CON
COMPLICACIÓN: REBOTE (+++) SE INTERVIENE DE URGENCIAS
UTILIZACIÓN DE MEDIAD ELASTICAS ENCONTRÁNDOSE APENDICITIS GRADO IV.
DURANTE EL CIERRE EL MANEJO ADECUADO
ANTE UN PACIENTE QUE SE ENCUANTRA PARA LA PIEL ES:
HOSPITALIZADO Y CURSANDO 2° DIA CIERRE DEFERIDO
POSTOPERATORIO POR COMPLICACIONESDE
PANCREATITIS NECROTICA HEMORRAGICA Y MASCULINO DE 56ª QUIEN PADECE DE
DE QUIEN SE SOSPECHA QUE CURSE CON DOLOR ABDOMINAL DESDE HACE 5ª
UNA DESNUTRICIÓN GRAVE, EN ESTAS ARDOROSO EN EPIGASTRIO, TOMANDO
ANTIÁZ PRESENTA DOLOR INTENSO REGION INGUINAL DERECHA QUE
ARRADIADO TODO EL ABDOMEN. A LA EF DESAPARECE RAPIDAMENTE AL ADOPTAR EL
PERISTALSIS AUSENTE, RESISTENCIA DECUBITO DORSAL. PARA MANEJAR ESTA
MUSCULAR CON REBOTE PRESENTE (+++) SE CONDICION EL TRATAMIENTO INDICADO EN
DECIDE PASAR A CIRUGÍA DENTRO DE LAS ESTE PACIENTE ES:
INCISIONES CUAL ES LA MÁS ADECUADA: TRATAR DE DISMINUIR LA OBESIDAD DEL
MEDIA SUPRA INFRAUMBILICAL PACIENTE YDESPUÉS OPERARLO

MASCULINO DE 22ª INGRESA A URGENCIAS MASCULINO 60ª PROSTATICO, DIABÉTICO,


REFIRIENDO DOLOR EN FID POR LO QUE SE OBESO QUIEN DESDE HACE TRES DIAS
DIAGNOSTICA APENDICITIS DE LOS EMPEZO A NOTAR UNA MASA EN REGION
SIGUIENTES EXAMENES PREOPERATORIO ANGUINAL IZQUIERDA HEMIESTERICA
CUAL SERIA DE MAYOR IMPORTANCIA: QUE DESAPARECE RAPIDAMENTE AL
EKG ADOPTAR EL DECUBITO DORSAL. POR TODAS
LAS PATOLÓGICAS QUE PRESENTA ESTE
PACIENTE DE 42ª CON HISTORIA DE PACIENTE LOS MAS PROBABLES ES
CUADROS DOLOROSOS EN CUADRANTE QUE LA DEFORMACIÓN MENCIONADA
SUPERIOR DERECHO ACOMPAÑADOS DE CORRESPONDA:
NAUSEAS Y VOMITOS DE 2 AÑOS DE HERNIA INGUINAL DIRECTA
EVOLUCION ACTUELMENTE CON COLURIA,
ICTERICIA Y ACOLLA. QUE ESTUDIO SE USA MASCULINO DE 67ª CON DIAGNOCTICO DE
PARA CONFIRMAR EL DIAGNOSTICO: HERNIA INGUINAL DERECHA DENTRO DE
ULTRASONOGRAFIA LOS ESTUDIOS PARACLINICOS
PREOPERATORIOS. LOS MAS IMPORTANTES
MASCULINO DE 30ª QUIEN PRESENTA DESDE EN EL CASO SON:
HACE 3 AÑOS DOLOR EPIGASTRICO TELE DE TORAX Y ELEC TROCARDIOGRAMA
RECURRENTE, EL QUE SE PRESENTA
COMÚNMENTE POR LA MADRUGADA. DESDE
HACE 6 HORAS PRESENTA HEMATEMESIS, SU EN UN PACIENTE QUEMADO DE 7OKG DE
DIAGNOSTICO ORIENTA HACIA: PESO CON 30% DE SUPERFICIE QUEMADA DE
ULCERA PEPTICA SEGUNDO GRADO, NECESITARA DURANTE
SUS PRIMERAS 8 HORAS LA SIGUIENTE
EN UN PACIENTE CON ULCERA PEPTICA CANTIDAD DE SOLUCION (PARKLAND)
PERFORADA, DENTRO DE LA EF A LA 12,000ML
PERCUSIÓN ESPERARIA ENCONTRAR:
DESAPACRICION DE LA MATIDEZ HEPÁTICA MASCULINO DE 45ª QUEMADO POR
INTOXICACIÓN CON CO2 QUIEN PRESENTA
FEMENINO DE 45ª REFIERE LA PRESENCIA DE DISNEA Y QUEMADURAS EN CARA LAS
UNA MASA A NIVEL ABDOMINAL NO CUALES NO SON DOLOROSAS, EL PACIENTE
DOLOROSA. A LA EF SE ENCUANTRA MASA DEBERA HABER PRESENTADO POR LO
DE APROXIMIDAMENTE 2X2CM. POCO MENOS UNO DE LOS SIGUIENTES DATOS A LA
MOVIL, BLANDA NO DOLOROSA, ENTRE EXPLORACIÓN FÍSICA:
APÉNDICE, XIFOIDES Y CICATRIZ UMBILICAL CAPA CUBIERTA CON HOLLÍN NEGRO
EN LA LINEA MEDICA, CUAL SERIA LA
TERAPEUTICA:
PLASTIA FEMENINO 48ª QUE VA ASER SOMETIDA A
HISTERECTOMÍA POR PRESENTAR CACU LA
MASCULINO DE 60ª QUE PRESENTA UNA SIGUIENTE MEDIDA ANTITROMBOTICA ES:
BOLA EN REGION INGUINAL DERECHA QUE MEDIAS ELASTICAS
SE DESAPARECE CON EL REPOSO, NO
DOLOROSA QUE NO LLEGA AL ESCROTO, A FEMENINO 40ª CON DIAGNOSTICO DE
LA EF LA MASA TOCA EL PULPEJO DEL MIOMATOSIS UTERINA POR LO QUE SE VA A
DEDO EXPLORADOR A TRAVEZ DEL PISO PRACTICAR HITERECTOMIA TOTAL
INGUINAL EN LA REPARACIÓN QUIRÚRGICA ABDOMINAL, NO CUENTA CON
DE ESTE DEFECTO. CUAL ES LA SUTURA MAS ANTECEDENTES DE IMPORTANCIA, POR LO
ADECUADA: CUAL ESTARA CLASIFICADA CON UN RIESGO
PROLENE ANESTESICO QUIRÚRGICO CLASE:
I
MASCULINO DE 65ª EL CUAL PRESENTA
HERNIA INGUINAL DIRECTA. EL MANEJO DE MASCULINO DE 60ª EL CUALA VA A SER
ESTA CONDICION PUEDE HACERSE POR SOMETIDO A RESECCION TRANSURETRAL DE
CUALQUIERA DE LAS SIGUIENTES VIAS: PRÓSTATA. COMO ANTECEDENTE CUANTA
VIA RETROPERITONEAL CON INFARTO AL MIOCARDIO
VIA PERITONEAL HACE 4 MESES SE CATALOGA COMO RIESGO
VIA PERITROESCOPIA ANESTESICO QUIRÚRGICO CLASE:
VIA TRACTO ILEO PURICO IV

MASCULINO DE 67ª PROSTATICO, DIABÉTICO, FEMENINO DE 23ª LA CUAL SE ENCUANTRA


OBESO Y CON EPOG, QUIEN HACE UN MES EN URGENCIAS CON DIATROSIS, TA90/50,
EMPEZO A NOTAR MASA TUMORAL EN FR24X´, FC100X´, TEMP 38°, A LA CUAL SE LE
DIAGNOSTICO TROMBOSIS MESENTERICA, EN UN PACIENTE QUE SUFRE HERIDA POR
COMO ANTECEDENTE EL PACIENTE ES ARMA BLANCA EN TORAX IZQUIERDO. CON
DIABÉTICO DESDE HACE 10 AÑOS TA90/50. INGURGITACION YUGULAR POR LO
MANEJADO CON HIPOGLUCEMIANTES QU SE LE REALIZO PERICARDIOCENTESIS DE
ORALES SEGÚN LA CLASIFICACION DE ASA. URGENCIAS. ELTRATAMIENTO CONSISTE EN:
SU RIESGO QUIRÚRGICO SERA DE: TORACOTOMIA Y MIOCARDIOGRAMA
V
MASCULINO DE 30ª QUE SUFRE ACCIDENTE
MASCULINO DE 34ª EL CUAL SUFRIO AUTOMOVILISTICO POR LO QUE ES TRAIDO A
ACCIDENTE AUTOMOVILISTICO EN URGENCIAS, ENCONTRÁNDOSELE
CARRETERA. AL MOMENTO DE ARRIBAR SEMIINCONSCIENTE. HIPOTENSO,
CON LA AMBULANCIA USTED DETECTA QUE CIANOTICO. EN CUALLO INGURGITACION
EL PACIENTE SE ENCUENTRA YUGULAR CON GRACTURA DE ARCOS
SEMICONSCIENTE, SIGNOS VITALES: COSTALES EN HEMITORAX IZQUIERDO SE LE
TA110/70, FR90X´, FC88X´, CIANOSIS TOMA LA P.V.C. LA CUAL ESTA ELEVADA.
PERIFERICA , LESION MAXILOFACIAL CUAL ES EL DIAGNOSTICO:
IMPORTANTE, MOVIMIENTOS HEMOPERICARDIO
RESPIRATORIOS SUPERFICIALES Y LA
PRESENCIA DE FRACTURA EXPUESTA DE PACIENTE DE 28ª POLITRAUMATIZADO EN
TIBIA. CUAL ES LA CONDUCTA MAS ACCIDENTE POR MOTOCICLETA NO
IMPORTANTE QUE DEBE REALIZAR: LLEVANDO CASCO PROTECTOR ES ADMITIDO
CRICOTIROIDECTOMIA A URGENCIAS CON CIANOSIS Y SEVERA
DIFICULTAD RESPIRATORIA, TA80/40 Y
SANGRADO NASAL. ADEMÁS DE FRACTURA
MASCULINO DE 44ª EL CUAL SUFRIO EXPUESTA DE FÉMUR RUIDOS
ACCIDENTE AUTOMOVILISTICO AL RESPIRATORIOS AUSENTES EN HEMITORAX
MOMENTO DE ARRIBAR EN LA AMBULANCIA DERECHO. LA PRIORIDAD DEL MENEJO SERA:
USTED DETECTA QUE EL PACIENTE SE OBTENER ACCESO IV PARA TRANSFUSIÓN DE
ENCUENTRA SEMICONSCIENTE, CON EMERGENCIA DE SANGRE TIPO O
CIANOSIS PERIFERICA, LESION
MAXILOFACIAL IMPORTANTE, EN UN PACIENTE POLITRAUMATIZADO AL
MOVIMIENTOS RESPIRATORIOS APENAS CUAL EN LA EF SE REPORTA TA 100/60, FR32X,
PERCEPTIBLES Y LA PRESENCIA DE ABDOMEN DISTENDIDO. SILENCIO
FRACTURA EXPUESTA DE LA TIBIA ABDOMINAL , REBOTE (++), TIMPANISMO
Y PERONE. CUAL ES LA MEDIDA QUE GENERALIZADO. QUE ESTUDIO CORROBORA
APLICARIA: EL DIAGNOCTICO:
COLOCAION DE LA CANULA DE GEDELE TELE DE TORAX

PACIENTE QUE SUFRE ACCIDENTE


FEMENINO DE 27ª QUIEN A SUFRIDO CAIDA AUTOMOVILISTICO Y QUE AL EXAMEN SE
DE LESION DE BAZO, HIGADO, ASI COMO DE ENCUANTRA DESPIERTO Y RESPONDE A LAS
INTESTINO DELGADO, SE TRANSFUENDEN PREGUNTAS, A NIVEL DE TORAX SE
MAS DE 10 U DE SANGRE SI EL SANGRADO ENCUANTRAN DATOS QUE SUGIEREN
PERSISTIERA A PESAR DEL SUFIECINTE NEUMOTORAX A TENSIÓN EN HEMITORAX
REMPLAZO DE COAGULANTES SE PENSARIA DERECHO, LA MEDIDA A APLICAR EN ESTE
EN LA PRESENCIA ACTUAL DE: PROBLEMA SERA: TORACOCENTESIS EN 2°
COAGULACIÓN INTRAVASCULAR ESPACIO INTERCOSTAL
DISEMINADA
MASCULINO QUIEN EN SALA DE
OPERACIONES PREVIA A LA INTUBACIÓN,
FEMNINO DE 15ª QUIEN ES ASALTADA EN LA PRESENTA PARO CARDIORRESPIRATORIO SUS
VIA PUBLICA, A LA EF SE ENCUENTRA PULSOS CENTRALES SONDEBILES, SU TA
DISEÑA, INQUIETUD, DOLOR, FR40X 60/50. FRECUENCIA RESPIRATORIA
SILENCIO RESPIRATORIO Y CLARIDAD SUPERFICIAL, DADA LA BRADICARDIA DEL
PULMONAR AUMENTADA EN LA RX DE PACIENTE, CUAL ES EL MEDICAMENTO
TORAX EL DIAGNOSTICO ES: USADO PARA SU MANEJO:
NEUMOTORAX A TENSIÓN ADRENALINA ENDOTRAQUEAL

MASCULINO DE 29ª QUE CONDICIENDO SU MASCULINO DE 38ª QUIEN SE RECIBE EN


AUTOMOVILA ALTA VELOCIDAD SUFRE URGENCIAS CON ANTECEDENTES DE
CHOQUE, Y A LA EF SE ENCUANTRA CON TA LSIONES EN TORAX Y ABDOMEN NO
100/60, EC100X´, FR24X, EN EL RESTO DEL PENETRANTES. ASI COMO FRACTURAS
EXAMEN EN HEMITORAX DERECHO MULTIPLES EN EXTREMIDADES INFERIORES
MURMULLO VESIDULAR DESMINUIDO CON A SU LLEGADA TA 60/40 RUIDOS CARDIACOS
MATIDEZ A LA PERSECUCIÓN. CUAL SERIA EL POCO PERCEPTIBLES, RESPIRACIÓN
TRATAMIENTO TERAPÉUTICO PARA SU SUPERFICIAL, PULSOS EN FEMORAL Y EN
PROBLEMA PULMONAR: PUNCION CON CAROTIDA AUSENTES SE DIAGNOSTICA
TROCAR EN 6° ESPACIO INTERCOSTAL MENIOBRA DE COMPRESIÓN EXTERNA Y
DESFIBRILACION, PERSISTE CON ARRITMIAS
VENTRICULARES. CUAL ES LA DOSIS DE
LIDOCAINA INDICADA PARA EL PERCEPTIBLE, QUE DATO ES DETERMINANTE
TRATAMIENTO: PARA HACER EL DIAGNOSTICO:
BOLO IMG/ KG IV PERDIDA DEL PULSO EN UNA ARTERIA
CENTRAL (CAROTIDA O FEMORAL)
MASCULINO DE 50ª QUIEN INGRESA AL
SERVICIO DE URGENCIAS CON FEMENINO DE 42ª QUIEN SE LE REALIZO
ANTECEDENTE DE HABER SUFRIDO COLECISTECTOMIA ABIERTA POR
ACCIDENTE AUTOMOVILISTICO, A SU COLECISTITIS CRÓNICA LITIASICA AL SALIR
LLEGADA PRESENTA DESORIENTACIÓN, A SU DE QUIRÓFANO TIENE COLOCADA SONDA
LLEGADA PRESENTA DESORIENTACIÓN, TA NASOGASTRICA, SONDA VESICAL
50/40, FRECUENCIA CARDIACA NO AUDIBLE VENOCLISIS CON HARTMAN GLUCOSADA,
SIN RESPIRACIÓN ESPONTÁNEA, PULSOS EN RESPIRACIÓN DE 18X´, PULSO 75X´, TA 130/85,
CAROTIDAS AUSENTES. POR LO QUE SE ESTA PACIENTE DEBERA DE AMBULAR:
DIAGNOSTICA PARO AL DIA SIGUIENTE DE LA CIRUGÍA
CARDIORRESPIRATORIO. SE INICIAN
MANIOBRAS DE REANIMACION CON
COMPRESIONES TORAXICAS Y
RESPIRACIONES, EN EL MONITOR SE
OBSERVA FIBRILACION VENTRICULAR, CUAL PACIENTE DE 22ª ACUDE A URGENCIAS
ES EL SIGUIENTE PASO PAR CONTINUAR CON REFIRIENDO DOLOR EN FID SE DIAGNOSTICA
EL TRATAMIENTO: APENDICITIS Y SE REALIZA
INICIAR DESFIBRILACION E INTUBAR AL APENDICECTOMIA. EL HALLAZGO
PACIENTE QUIRÚRGICO FUE APENDICITIS EN FASE
(EDEMATOSA) 1, LO CITA PARA RETIROS DE
FEMENINO DE 55ª CON CHOQUE SÉPTICO Y PUNTOS A LOS 7 DIAS
DISFUNCIÓN ORGANICA MÚLTIPLE. A LA EF
PACIENTE SOMNOLIENTO, RUIDOS MASCULINO DE 22ª SOMETIDO A LA
CARDIACOS DISMINUIDOS, FRECUENCIA LAPAROTOMÍA CON HALLAZGO DE
RESPIRATORIA NO PERCEPTIBLE PULSOS PERFORACIÓN DEL ILEON DISTAL POR
CENTRALES AUSENTES, SE HACE SALMONELLOSIS, AL 3ER. DIA
DIAGNOSTICO DE PARO POSTOPERATORIO INICIA CON FIEBRE,
CARDIORRESPIRATORIO, SE INICIA MASAJE DOLOR, TUMEFACCIÓN, HIPEREMIA Y
CARDIACO EXTERNO LA META AUMENTO DE LA TEMPERATURA LOCAL, LA
FUNDAMENTAL EN TODA SESION DE CAUSA PROBABLE DE ESTA COMPLICAION
REANIMACION CON LA COMPRESIÓN ES: FALTA DE COLOCAION DE DRENES EN LA
ES: RESTAURAR LA FUNCION CARDIACA CAVIDAD
NORMAL LO MAS PRONTO POSIBLE
MASCULINO DE 30ª POSTOPERATORIO DE
FEMENINO DE 65ª QUIEN PRESENTA SEPSIS HERNIOPLASTIA INGUINAL DERECHA
ABDOMINALSECUNDARIA DIVERTICULITIS INDICADA EN EL 2° DIA POSTOPERATORIO
COLONICA COMPLICADA, SE REALIZA PRESENTA AUMENTO DE VOLUMEN DE LA
LAVADO Y DRENAJE DE LA CAVIDAD, EN EL HERIDA CON FLUCTUACIÓN, NO DOLOROSA,
POST OPERATORIO LA PACIENTE NO SE OBSERVAN DATOS DE CELULITIS NI
CARDIOPULMONAR LA CUAL ES EFECTIVA HIPERTERMIA CON ESTOS DATOS LA
USTED INDICA QUE LA PACIENTE ES COMPLICAION QUE PRESENTA EL PACIENTE
TRASLADADA A: SU CAMA CON CONTROL ES: DEHISCENCIA
ESRICTO DE SIGNOS VITALES
PACIENTE DE 50ª CON CANCER DE ESÓFAGO
FEMENINO DE 60ª QUIEN INGRESA A Y CON INANICIÓN EN ESTAS
URGENCIAS POR ACCIDENTE CIRCUNSTANCIAS SU CORAZON Y
AUTOMOVILISTICO AL LLEGAR AL HOSPITAL MUSCULOS TIENEN PREDILECCIÓN POR UNO
SUS SIGNOS VITALES SON: TA60/40, RUIDOS DE LOS SIGUIENTES AMINOÁCIDOS:
CARDIACOS POCO PERCEPTIBLES, DE CADENA RAMIFICADA
RESPIRACIÓN SUPERFICIAL, MINUTOS
DESPUÉS DE SU LLEGADA AL MASCULINO DE 36ª QUE DURANTE LA
SERVICIO DE URGENCIAS PRESENTA PARO INANICIÓN AGUDA DEBIDO A CANCER
CARDIACO RESPIRATORIO, CUAL ES LA LARINGEO TIENE UNA PERDIDA NETA DE
MEDIDA QUE INICIO PARA RESTAURAR LA AMINOÁCIDOS YA QUE LAS SÍNTESIS
FUNCION CARDIACA: AUMENTAR EL DE PROTEINA MUSCULAR DESAPARECE O
VOLUMEN SANGUÍNEO POR MEDIO DE DESCIENDE TANTO QUE EL CATABOLISMO
SOLUCIONES DE LA MISMA PERMANECE SIN CAMBIO
ESTA SITUACIÓN ES DEBIDA A:
FEMENINO DE 60ª QUIEN ES INGRESADA A AUMENTO DE GLUCAGON
CIRUGÍA PARA REALIZAR LAVADO DE
CAVUDAD POR SEPSIS PERITONEAL, EN EN UN PACIENTE POLITRAUMATIZADO A
QUIRÓFANO PRESENTA PARO QUIEN EN EL EXAMEN ENCONTRAMOS EN
CARDIORRESPIRATORIO A LA EXPLORACIÓN HEMOTÓRAX IZQUIERDO SILENCIO
RUIDOS CARDIACOS POCO AUDIBLES, RESPIRATORIO, A LA PERCUSIÓN
FRECUENCIA RESPIRATORIA NO TIMPANISMO Y CON CHOQUE DE PUNTA A LA
DERECHA. REALIZÁNDOSELE
TORACOCENTESIS DE URGENCIAS, LA ACOMPAÑADA DE HIPEREMIA Y NAUSEAS, A
JUSTIFICACIÓN DE ESTA MEDIDA ES POR: LA EXPLORACIÓN ROVSING (+) Y BLUNBERG
LA URGENCIA DE DESCOMPRIMIR AL (+). EN EL PUNTO DE MC BURNEY PARA EL
MEDIASTINO ASEO QUIRÚRGICO DE SUS MANOS, CUAL DE
LAS SIGUIENTES SUSTANCIAS
MASCULINO DE 34ª LLEVADO A URGENCIAS ANTISÉPTICAS UTILIZARA:
POR SUFRIR ACCIDENTE AUTOMOVILISTICO HI-BISCRUB (GLUCONATO DE
AL EXMAMEN ENCONTRAMOS CLORHEXIDINA)
HIPERSENSIBILIDAD LEVE EN EL ABDOMEN
Y HEMITORAX DERECHO. CON PIRISTALSIS EN EL PISO DE CIRUGÍA, SE DISCUTE EL
MUY DISMINUIDA Y AUSENCIA DE MATIDEZ CASO DE UN PACIENTE QUE PRESENTO
HEPÁTICA; CUAL ES SU IMPRESIÓN: DESHICENCIA DE LA HERIDA QUIRÚRGICA
RUPTURA DE VICERA HUECA EN EL 5° DIA DEL POSTOPERATORIO SE
HACE MENCIONAR QUE FUE DEBIDO A MALA
FEMNINO DE 22ª QUIEN VA A SER SOMETIDO TÉCNICA QUIRÚRGICA EN EL CIERRE DE LA
A LAPAROSCOPIA PÉLVICA POR PROBABLE HERIDA ESTA COMPLICACIÓN IMPLICA:
ENDOMETRIOSIS DURANTE LA ASEPSIA DEL SEPARACIÓN PARCIAL O TOTAL DE LOS
AREA QUIRÚRGICA (PIEL) CUAL DE LAS PLANOS DE LA HERIDA
SIGUIENTES SUSTANCIAS CONSIDERA QUE
ES LA MAS ADECUADA PARA RELIZAR ESTE FEMENINO DE 48ª QUIEN INGRESA A
PROCEDIMIENTO: URGENCIAS CON CUADRO DE ABDOMEN
YODOPOVINA (ISODINE) AGUDO SECUNDARIO A COLELITITIS
COMPLICADA. A LA EXPLORACIÓN
ENCUANTRA TALLA 1.60M PESO DE 92KG TA
FEMENINO DE 38ª QUE SE ENCUENTRA EN EL 140/90 DOLOR A LA PALPACIÓN EN
5TO DIA POSTOPERATORIO POR COLECTOMIA HIPOCONDRIO DERECHO. MURPHY (+)
TOTAL PARA EL TRATAMIENTO DE SU REBOTE POSITIVO, QUE INSICION SE
COLECISTITIS ULCERATIVA CRÓNICA RECOMIENDA: SUPRA INFRAUMBILICAL
INESPECÍFICA, LA CIRUGÍA SE EFECTUA SIN
COMPLICACIONES CERRANDO LAS CAPAS DE MASCULINO DE 28ª QUIEN FUE SOMETIDO A
LA HERIDA CON SURGENTE, COMO APENDICECTOMIA ENCONTRANDO UNA
ANTECEDENTES DE IMPORTANCIA APÉNDICE CECAL GANGRENADA. ESTE
MENCIONA SER DIABÉTICA TIPO 1, EN EL ANTECEDENTE NOS PERMITE CLASIFICAR LA
2DO DIA POSTOPERATORIO PRESENTO HERIDA COMO: SUCIA
ACUMULO DE LA SANGRE EN LA HERIDA Y
HOY OBSERVA EVICERACION.
LA CAUSA MAS PROBABLE DE LA MASCULINO DE 36ª SOMETIDO A
COMPLICACIÓN ES: HERNOPLASTIA CON RESECCION DE UN
INFECCIÓN DE LA HERIDA DIVERTICULO DE MECKEL A TRAVEZ
DEL ANILLO HERNIARIO EN EL
FEMENINO DE 17 AÑOS CON DIAGNOSTICO POSTOPERATORIO PRESENTO ABSCESO EN
DE PÚRPURA TROMBOCITOPENICA LA HERIDA QUIRÚRGICA EL MANEJO DE
IDIOPATICA REFRACTARIA A TRATAMIENTO ESTA COMPLICACION SE REALIZA CON
MEDICO, PROGRAMADA PARA DRENAJE Y CIERE: DIFERIDO
ESPLENECTOMIA. DESPUÉS DE LA ASEPSIA Y
ANTISEPSIA DEL AREA QUIRÚRGICA, LA MASCULINO DE 19ª CON APENDICITIS AGUDA
COLOCACIÓN DE CAMPOS Y SABANAS SE NO COMPLICADA CUAL ES LA INCISIÓN MÁS
INDICA CON: SABANA PODALICA CONVENIENTE PARA ESTE PACIENTE:
ROCKY DAVIS
FEMENINO DE 40ª QUIEN PRESENTA DOLOR
ABDOMINAL EN ABDOMEN SUPERIOR Y QUE CUAL ES LA HORMONA QUE DISMINUYE EN
SE LE EXACERVA CON LOS ALIMENTOS LA RESPUESTA METABÓLICA AL TRAUMA:
GRASOS. A LA EXPLORACIÓN MURPHY (+), INSULINA
EL ULTRASONIDO DE HIGADO Y VIAS
BILIARES REVELA VESÍCULA DE 11X9CM, E MASCULINO DE 53ª POLITRAUMATIZADO
IMÁGENES HIPERECOGENICAS EN SU POR ACCIDENTE AUTOMOVILISTICO EN ESTA
INTERIOR, PARA LA CIRUGÍA EN QUE AREA PACIENTE LA RESPUESTA METABÓLICA AL
SE VISTE CON LA ROPA QUIRÚRGICA: TRAUMA PROVOCA
NEGRA LIBERACIÓN DE ADRENALINA, LA QUE
PROVOCARA:
MASCULINO DE 71ª QUIEN FUE SOMETIDO A ESTIMULA LA LIPÓLISIS
COLECISTECTOMIA CON INCISIÓN MEDIA
SUPRAUMBILICAL EN EL 5TO DIA DEL PACIENTE POLITRAUMATIZADO EL CUAL
POSTOPERATORIO PRESENTO ENCUENTRA EN LA UCI DEVEMOS TENER EN
EVISCERACION. LA CAUSA MÁS PROBABLE CUENTA QUE DURANTE LA RESPUESTA
DE ESTA COMPLICACION ES: METABÓLICA AL TRAUMA LA ACCION
TÉCNICA QUIRÚRGICA INADECUADA DE LAS HORMONAS
CONTRARREGULADORAS TIENE
MASCULINO DE 22ª PRESENTA DOLOR COMO RESPUESTA EN EL ORGANISMO
ABDOMINAL EN FID CONTINUO, AGREDIDO:
AUMENTO DE LA GLUCONEOGENESIS COMPLICACIONES PULMONARES
POSTOPERATORIAS:
FEMENINO DE 28ª LA CUAL PRESENTO EJERCICIOS RESPIRATORIOS MINIMO 48
TRAUMA ABDOMINAL CERRADO CON HORAS ANTES DE LA CIRUGÍA
RUPTURA HEPÁTICA E ISQUEMIA
INTESTINAL POR DESGARRO DEL PACIENTE POLITRAUMATIZADO MANTENIDO
MESETERIO DURANTE UNA INTERVENCIÓN CON NUTRICION PALENTERAL TOTAL CON
QUIRÚRGICA, PRESENTA SANGRADO DIFUSO UNA FORMULA EQUILIBRADA, PERO QUE NO
DE LA PARED ABDOMINAL Y EN HIGADO ES SUFICIENTE PARA
DEBIDO A UN PROBABLE CID, EL CUBRIR SUS REQUERIMIENTOS
TRATAMIENTO QUE SE RELIZARA ENERGÉTICOS PARA INCREMENTAR EL
EN ESTE MOMENTO SERA: APORTE ENERGÉTICO SE CONSIDERA
EMPAQUETAMIENTO ABDOMINAL A: AUMENTAR LA FRACCIÓN DE GLUCOSA
DE LA FORMULA
PACIENTE EL CUAL FUE SOMETIDO A
HEMICOLECTOMIA POR CANCER DE COLON MASCULINO DE 28ª PROVENIENTE DE LA UCI
EN EL POSTOPERATORIO PRESENTA CUADRO DONDE FUE SOMETIDO A UNA
COMPATIBLE CON CID LA ALTERACIÓN LAPAROTOMÍA EXPLORADORA POR HABER
LABORATORIAL QUE ESPERARIA SUFRIDO ACCIDENTE AUTOMOVILISTICO
ENCONTRAR SERA: MENCIONÁNDOSE QUE SE LE REALIZO
ANTITROMBINA III DISEMINADA RESECCION INTESTINAL DE 40CM DE
INTESTINO DELGADO POR PRESENTAR
FEMENINO DE 28ª CON SEPSIS ABDOMINAL Y ISQUEMIA INTESTINAL SECUNDARIA A
A LA EXPLORACIÓN FÍSICA PACIENTE DESGARRO MESENTERICO POR
SOMNOLIENTA SIGNOS VITALES TA 80/60, FC DESACELERACION LOS RECEPTORES QUE
110X, EN EL CHOQUE SÉPTICO. LA ESTIMULAN LA LIBERACIÓN DE
HIPERVENTILIACION SE ENCUANTRA EN LA CORTICOTROPA EN ESTE PACIENTE SE
FASE: ENCUANTRAN LOCALIZADOS EN:
TEMPRANA PROTUBERANCIA Y MENSENCEFALO

FEMENINO DE 32ª QUIEN REFIERE UNA MASA ANTE UN PACIENTE QUE FUE SOMETIDO A
BLANDA, NO DEPRESIBLE, FIJA E INDOLORA DRENAJE ABIERTO DE ABSCESO HEPÁTICO
Y POR ARRIBA DE LA CICATRIZ UMBILICAL. AMIBIANO, EL CUAL SE ENCUANTRA EN LAS
EN ESTE CASO EL DEFECTO SE ENCUANTRA PRIMERAS HORAS POSTOPERATORIO FASE
EN LA: EBB, USTED ESPERARIA ENCONTRAR UN
LINEA ALBA AUMENTO DE LA SIGUIENTE SUSTANCIA
PRODUCIDA POR EL ORGANISMO. CUAL ES:
DURANTE UNA COLECISTECTOMIA ABIERTA TIROXINA
ACCIDENTALMENTE SE LE CAEN LAS
TIJERAS. LAS CUALES SON MASCULINO DE 35ª EL CUAL SE ENCUENTRA
IMPRESCINDIBLES PARA RELIZAR LA EN LA UCI CON DIAGNOSTICO DE
CIRUGÍA, NECESITAN QUE LAS TIJERAS SE PANCREATITIS AGUDA DESDE
ESTERILICEN QUE SUSTANCIA PERMITE HACE 10 DIAS ACTUALMENTE EL
REALIZAR ESTE PROCEDIMIENTO ORGANISMO TIENE: NECESIDAD DE
RAPIDAMENTE: CONSUMIR CETONAS
GLUTERHALDEHIDO ACTIVADO
*ANTE UN PACIENTE QUE EN EL
MASCULINO DE 18ª CON HERIDA PRODUCIDA POSTOPERATORIO CURSO CON
POR OBJETO CORTANTE EN REGION DESEQUILIBRIO HÍDRICO PODEMOS INTUIR
FRONTAL DE LA CARA DE 3CM APROX DE QUE EL AGUA INTRACELULAR PUEDE
LONGITUD, QUE TECNICA DE SUTURA DISMINUIR DEVIDO A UNA DE LAS
USARA PARA CERRAR LA PIEL Y TENER SIGUIENTES ALTERACIONES CUAL ES ESTA:
UN RESULTADO MAS ESTETICO: REDUCCIÓN DEL POTASIO INTRACELULAR
SUTURA SUBCUTICULAR

INGRESA PACIENTE A URGENCIAS CON


HERIDA EN LA CARA ANTERIOR DE FEMENINO DE 25 AÑOS QUIEN SUFRIO
ANTEBRAZO DE APROXIMADAMENTE 3CM, ACCIDENTE AUTOMOVILISTICO CON
DE LONGITUD, REALIZA EL ASEO DE LA LESIONES LEVES, PERO FUE VICTIMA DE
HERIDA DE LA PIEL CON ISODINE, CUAL UNA CRISIS EMOCIONAL, SE LE TOMA
ES EL TIEMPO MINIMO NECESARIO PARA GASOMETRIA REVELANDO UNA PCO2 DE
OBTENER EL EFECTO ESPERADO: 30MMHG, TAL CIFRA Y EL CUADRO CLINICO
10 MINUTOS SUGIEREN: ACIDOSIS RESPIRATORIA

PACIENTE DE 58ª PROGRAMADO PARA MASCULINO DE 35ª QUIEN ES RESCATADO DE


GASTRECTOMÍA SUBTOTAL POR CANCER ACCIDENTE AUTOMOVILISTICO Y QUE
GÁSTRICO EN ETAPA TEMPRANA COMO PRESENTA FRACTURAS MULTIPLES Y
ANTECEDENTE MECIONA PADECER EXPUESTAS EN MIEMBRO SUPERIOR
ENFERMEDAD PULMONAR. DE LAS DERECHO OCACIONANDO LESION ARTERIAL.
SIGUIENTES RESPUESTAS CUAL EVITA LAS AL INICIARSE COMO PRIMER EVENTO LA
VASOCONSTRICCIÓN SE ACTIVA EL
SIGUIENTE ELEMENTO:
MICROAGREGADO PLAQUETARIO

1) ¿Cual de los siguientes antibióticos pueden alcanzar concentraciones inhibitorias en los


abscesos abdominales?
a) Gentamicina
b) Metronidazol
c) Ceftriaxona
d) Tetraciclina
e) Penicilina G sodica cristalina
2) * Masculino de 34 años con estreñimiento de más de 5 años, dos días con dolor intenso al
evacuar, sangrado rectal en goteo post evacuación, dolor al sentarse. Hemoglobina 12 grs.

consiste en:
a) Fistulectomia
b) Drenaje a cielo abierto
c) Hemorroidectomia
d) Fisurectomia más esfinterotomia lateral
e) Cauterización de la herida
3) Femenina de 36 años, politraumatizada, sometida a laparotomía exploradora por
hemoperitoneo. Al segundo dia postoperatorio presenta datos de insuficiencia renal aguda.
¿Cuál de las siguientes define esta complicación?
a) Hipercalcemia
b) Eritrocituria
c) Sobrecarga de volumen
d) Azotemia
e) Piuria
4) Femenina de 48 años de edad a quien se realizó resección pancreatoduodenal con tecnica
de whipple por carcinoma de la cabeza del páncreas. Al segundo dia postoperatorio se
encuentra en malas condiciones debido a una reposición de líquidos insuficientes. ¿Cuál
de los siguientes datos bioquímicos se relaciona con el deficit hidrico de esta paciente?
a) Hipoalbuminemia
b) Hiperazoemia
c) Hipocalcemia
d) Isonatremia
e) Hipoglucemia
5) Paciente post iliostomia, con gasto de 2.5L/24hr. Paciente refiere sed intensa, astenia y
adinamia, desubicado en tiempo y espacio, volumen urinario por hora de 15 ml.
Laboratorio: Glucosa 120, Urea 80, creatinina 1.8, Na 127, cloro 85, potasio 2. Signos
vitales: TA 90/50, FC 125, FR 22, temp. 37 grados. Que tipo de transtornos
hidroelectrolitricos presenta este paciernte?
a) Transtorno de PH y de la osmolaridad
b) Transtorno obstructivo y de formacion del tercer espacio
c) Transtornos de volumen, concentracion y composicion
d) Transtornos de distribucion e intercambio
e) Transtorno de control renal de electrolitos
6) * Dato radiologico que hace poner en duda el diagnóstico de obstrucción mecánica del
intestino?
a) Neumoperitoneo
b) Dilatación del intestino delgado
c) Dilatación del intestino grueso
d) Niveles hidro-aéreos
e) Gas en ampolla rectal
7) * Masculino de 40 años, bombero, presenta quemaduras, sin compromiso cardio-
respiratorio, lesiones en cara ant. De tórax, extremidades, superior e inf, indoloras, duras,
no palidecen a la presión, afecta menos del 50% de la superficie. ¿Qué solución se le
indica para la reanimación agresiva de líquidos?
a) Solucion Mixta
b) Fisiologica
c) Ringer Lactato
d) Glucosada
8) * Paciente con hernia en pared abdominal post incisional de 8cm de longitud, la mejor
opción de tratamiento quirúrgico es:
a) Cierre primario de la aponeurosis con sutura no absorbible
b) Desplazamiento de injerto Moo cutáneo
c) Colocar malla sustituyendo el peritoneo
d) Cierre primario de la aponeurosis con sutura absorbible
e) Colocación de malla pre peritoneal
9) Masculino 66 años 30 cigarros diarios durante 25 años. DM hace 3 años tratado con
hipoglucemiantes orales. PA: 8 meses de evolución con pérdida de peso cuantificada en
11kg, anorexia, debilidad generalizada, dolor en epigastrio irradiado a hipocondrio
derecho, ictericia en últimos 2 meses al igual que colitis, acolia, y prurito generalizado. Se
palpa hepatomegalia, vesícula biliar aumentada de tamaño con diámetros 9x6 cm.
Bilirrubina total 28, bilirrubina directa 25.2, fosfatasa alcalina 750. Hallazgos radiológicos
esperados con el paciente:
a) Dilatación de la vía biliar
b) Colecistitis aguda litiasica
c) Hepatomegalia sin dilatación de las vías biliares
d) Quiste hepático gigante
e) Colecistitis crónica litiásica
10) Femenina de 65 años hipertensa de más de 20 años cifras de 160/100, de le programó una
colecistectomia, ¿Qué riesgo quirúrgico presenta?
a) 1
b) 2
c) 3
d) 4
e) 5
11) Femenina 37 años se calló de un caballo, paciente parapléjica, con defecto de sensibilidad
en metamera T10. Signos vitales: TA: 80/50, FC: 130, FR: 22, Temp. 37°C. Presenta
piloereccion distal desde la metamera T8. ¿Que tipo de shock presenta este paciente?
a) Anafilactico
b) Séptico
c) Neurogenico
d) Hipovolemico
e) Cardiogenico
12) Niño recién nacido en quien la madre notó, en ambas regiones inguinales, una masa que
aumenta de tamaño cuando el bebé llora o puja. El médico del hospital comenta que se
trata de 2 hernias. ¿Cuál es el defecto congénito que predispone a la formación de estas
hernias?
a) Defecto de los pilares de anillo inguinal superficial
b) Defecto de fascia transversalis
c) Defecto de los pilares del anillo profundo
d) Permeabilidad del proceso peritoneo-vaginal
e) Defecto del mecanismo obturador del anillo inguinal profundo
13) Paciente masculino el cual tiene varias horas de presentar cuadro de oclusión intestinal
baja actualmente con vomitos fecaloides, ¿cuál es el mecanismo relacionado con las
caracteristicas de los vomitos?
a) La colonización del intestino delgado
b) La materia fecal tiene movimiento retrogrado
c) La oclusión es funcional
d) Existe antiperistalsis
e) Se relaciona con la perstalsis
14) Paciente de 25 años acude por quemaduras, ampollas y Eritema en cara, extremidad
superior izq y pecho. carbonización franca circunferencial de extremidad superior derecha
con Llenado capilar reducido. ¿Cuál es el manejo inicial más apropiado?
a) Excisión de todas las heridas de 3er grado
b) Escarotomía de la mano derecha
c) excisión de quemaduras de mano y cara
d) Antibióticos tópicos
e) injertos de piel de espesor parcial sobre las áreas que presentan quemaduras de 3er
grado
15) Paciente adulto con compromiso de vía aérea, imposible ventilar y se requiere de
emergencia ventilar al paciente. ¿Cual es posible por punción percutánea?
a) Cricotomia simple
b) Cricotiroidostomia
c) Traqueostomia
d) Intubacion
e) Traqueotomia
16) Paciente 22 años se cayo de cuarto piso, se realizó hepatectomía parcial y resección de 30
cm del intestino delgado, 15 unidades de paquete globular en el pre y transoperatorio, 5
litros de solución Hartmann y 4 unidades de plasma fresco congelado. Inicia con sangrado
difuso de la cavidad abdominal durante el cierre de la laparotomía. ¿Cuál es la causa más
probable?
a) Insuficiencia hepática
b) Hiperesplenismo
c) Hipoprotombinemia
d) Deficiencia del factor IX
e) Plaquetopenia
17) Femenina de 38 años, operada de colecistectomía debido a microlitiasis vesicular,
actualmente con dolor en epigastrio con ictericia, lab con fosfatasa alcalina 350. Que
patología se sospecha?
a) Hepatitis
b) Pancreatitis aguda
c) Úlcera péptica
d) Absceso Residual
e) Hematoma
18) Paciente con con trauma múltiple, multi transfundido con 10 unidades de paquete
globular. Se realizó Hepatectomía parcial y esplenectomía. Inicia con sangrado durante el
cierre, de consideró asociado a transfusión masiva, ¿cual estudio apoya esta probabilidad?
a) Determinación de factor IX
b) Cuerpos de Howell-Joly
c) Pruebas de función hepatica
d) Cuento de plaquetas
e) Biopsia de médula ósea
19) Masculino 43 años se fractura ambos fémures en accidente automovilístico. Se encuentra
con Alteración del estado de alerta, piel fría, húmeda y de aspecto marmóreo. SV: TA
80/40 Fc 120, FR 28, temp. 35.6°C. ¿Cual explica el evento fisiológico presente en este
paciente?
a) Acúmulo de ácido volátil y desviación alcali del bicarbonato
b) Disminución de la perfusion en cerebro y corazón
c) Apertura de esfínteres precapilares y de vasos de capacitancia
d) Incremento del efecto de la hormona reguladora
e) Hipoperfusion fusilar distemica

20) Paciente con pérdida de estado de alerta, sufrió lesión en región inguinal izq. Por proyectil
de arma de fuego con sangrado en orificio. Conducta que se debe adoptar:
a) Control de hemorragia
b) Continuar con ABC e ignorar hemorragia
c) Compresión directa y continuar con ABC
d) Aplicar torniquete controlado proximal y continuar con ABC
e) Pinzado hemostático y continuar ABC

21) Paciente de 49 años de edad, con choque séptico secundario a perforación intestinal, se le
aporta solución ringer-lactato (Hartmann) y de Han iniciado vasopresores. Se agregó
dificultad respiratoria y confusión mental. ¿Qué complicación presenta?
a) Sepsis severa
b) Síndrome de respuesta inflamatoria sistémica
c) Hepatitis reactiva
d) Transtorno vascular cerebral agudo
e) Choque séptico refractario

22) Mejor opción de tipo de sondaje debido a sus características y tiempo de duración para
preparación gastrica de un paciente el cual será sometido a endoscopio diagnóstico-
terapéutica por datos de retención gastrica condicionando un megacolon
a) Nelaton
b) Rush
c) Levin
d) Foley-Alcock
e) Robisson

23) Femenina 42 años, colecistectomía abierta es neumopata, nutrición normal. ¿Qué sutura se
utiliza?
a) Catgut simple 1
b) Seda 3/0
c) Vycril 3/0
d) Catgut crómico 1
e) Polipropileno 1

24) Paciente masculino de 60 años, quien se realizó laparotomía exploradora previa


colocación de catéter central. A los dos días del postoperatorio, durante la deambulación
presenta disnea e insuficiencia respiratoria, en la placa de tórax se observa una zona de
condensación pulmonar basal derecha datos que se encuentran en relación con que
complicación pulmonar.
a) Derrame pleural
b) infarto pulmonar.
c) neumotorax
d) neumonia
e) Tromboembolia pulmonar.

25) Usted acude al servicio de urgencias a valorar un paciente gue presenta cuadro de oclusión
intestinal como antecedente tiene dos cirugías previas cuál será la causa más probable?
a) Trombosis arterial
b) Adherencias
c) Hernia Ventral
d) Neoplasia de colon

26) femenino de 45 años de edad sometida a histerectomía de urgencia por sangrado


persistente. Se encontró necrosis de miomas uterinos. Ha recibido la transfusión de 5
unidades de paquete globular y 5 unidades de plasma fresco. En el tansopeatoro aparecen
datos de sangrado en caps por lo que se piensa en CID. Cual de los siguientes apoya este
diagnóstico
a) Presencia de productos de lidia de finrinogeno
b) Disminución del factor de Hageman
c) Presencia de reticulocitos
d) Antitrombina III disminuida
e) Crenocitos abundantes en orina

27) 3 year old boy spilled bleach onto lower extremities. Diagnosed with chemical burn, all
clothes are removed, in addition to resuscitation, which of the following is the most
appropriate initial management
a) Neutralize the burn wound with weak acids
b) Treatment of the burn wound with calcium gluconate gel
c) Wound debridement in the operating room
d) Washing of the burn wound with large volumes of water
e) Treatment of the burn wound with anti microbial agents

28) Una mujer de 78 años acude a Urgencias por dolor en fosa iliaca izquierda de 24 horas de
evolución asociado a fiebre y algún vómito ocasional. A la exploración destaca dolor a la
palpación de forma selectiva en la fosa iliaca izquierda con sensación de ocupación,
defensa y descompresión positiva. Ante la sospecha de diverticulitis aguda ¿cuál de las
siguientes afirmaciones es correcta
a)
la práctica de una colostomía derivativa sin resección del segmento sigmoideo
afectado
b) En caso de precisar intervención quirúrgica tras solucionarse el episodio agudo, el
abordaje laparoscópico está contraindicado.
c) En caso de absceso pélvico contenido está indicada la colocación de un drenaje
percutáneo guiado con TAC o ECO.
d) La exploración complementaria más segura y de mejor rendimiento es el enema
con contraste baritado
e) En caso de diverticulitis aguda no complicada está indicada la sigmoidectomía
efectiva tras la curación del primer episodio agudo.

29) El médico de emergencias le pide evaluar a una paciente de 46 años de edad, con dolor
abdominal en el cuadrante superior derecho, debe insistirse en la descripción del dolor,
para definir la sospecha diagnóstica ¿Cuál de los siguientes enunciados concuerda con el
cuadro clínico de esta paciente?
a) Si el dolor se inicia después de la ingesta de alimentos, se trata de úlcera
perforada.
b) si el dolor se irradia a la región inguinal se trata de cálculo ureteral.
c) si el dolor es postprandial e irradiado a hombro derecho, es colecistitis
d) Si el dolor se irradia al hombro izquierdo, se trata de colecistitis.
e) Si El dolor disminuye con el vómito, es una pancreatitis

30) A35-year-old woman undergoes an elective laparoscopic cholecystectomy for


symptomatic cholelithiasis. Which of the following wound classes best describes her
procedure?
a) Class II, Clean/contaminated
b) Class IV, Dirty
c) Class I Clean
d) Cass III, Contaminated
e) None of the above
31) Paciente masculino de 18 años que consulta por haber notado una masa en región inguinal
derecha, la cual apareció en forma espontánea. Al exámen físico, esa masa puede
reducirse con maniobras manuales cuidadosas,y al introducir el dedo explorador al anillo
inguinal superficial. se percibe el contacto del saco herniario con la maniobra de Valsalva.
A qué tipo de hernia Corresponde?
a) Hernia en pantalon
b) Hernia inguinal indirecta
c) Hernia Femoral
d) Hernia de Spigel
e) Hernia inguinal directa

32) Femenino de 53 años de edad, con un peso de 70 kgs. En estado de choque hipovolémico,
por diarrea coleriforme. Se ha iniciado la fluidoterapia, y para vigilar su respuesta se ha
colocado una sonda urinaria. Para considerar que el riñón tenga una perfusión sanguínea
adecuada. ¿ Cuánto debe de tener de gasto urinario ?
a) 10 a 20 mi/hr
b) 35 a 70 m/hr
c) Mas de 100 m/hr
d) 15 a 30 m/hr
e) 90 a 100 mi/hr
33) Masculino de 68 años de edad, el cual fue sometido a resección abdominoperineal por
presentar cáncer de recto, durante la cirugía las pérdidas sanguíneas fueron repuestas
volumen a volumen. En el postoperatorio fue manejado con soluciones glucosadas al 5%
presentando debilidad, letargia, convulsiones y posteriormente coma. Por lo referido, el
paciente está cursando con cual problema electrolítico?
a) Hiponatremia.
b) Hiperglucemia.
c) Hiperpotasemia.
d) Hipercalcemia.
e) Hipopotasemia
34) Paciente femenino de 48 años de edad que será sometida a histerectomía por presentar
metrorragia recurrente por hiperplasia endometrial. Será intervenida una vez que cuente
con valoración por cardiólogo ya que además es portadora de Hipertensión Arterial mal
controlada. ¿Qué estudios son indispensables para su evaluación cardiológica
preoperatoria?
a) CPK fracción MB, TGO Y Troponina
b) Prueba de esfuerzo en banda sin fin y EKG
c) Monitoreo tipo Holter
d) Ecocardiograma bajo estrés con dobutamina
e) Radiografia de torax y EKG

35) Ingresan dos pacientes varones de 28 y 25 años de edad, quienes sufrieron volcadura
Cursan segundo día de estancia en Unidad de Cuidados Intensivos, actualmente se
encuentran en ayuno y sin necesidad de vasopresores, con diuresis adecuada. Llama la
atención que sin ser diabéticos, mantienen cifras altas de glucemia ¿Cuál de las siguientes
sustancias promueve la gluconeogénesis evidente en estos pacientes?
a) Hormona lut
b) Luteinizante.
c) Arelina y leptina.
d) Cortisol.
e) Hormona folículo estimulante
f) Insulina.

36) Paciente masculino de 52 años de edad, alcohólico crónico, presenta gran distensión
abdominal con anasarca y dificultad respiratoria, su gasometría revela un PH. 7.44,
PaCO2: 28 mmHg. PaO2: 80 mmHg y HCO3: 28. ¿Cuál es el trastorno ácido-base
primario en este paciente?
a) Alcalosis mixta.
b) Alcalosis respiratoria compensada
c) Acidosis respiratoria.
d) Acidösis metabólica.
e) Alcalosis metabolica.
37) Durante reanimación de paciente con hernia penetrante de abdomen, por proyectil por
arma de fuego, que presenta choque hipovolemico. ¿Cual de las siguientes soluciones es
adecuada para iniciar reanimación?
a) Dextran 40
b) Cloruro de sodio al 0.9%
c) Hemacel
d) Plasma fresco
e) Hartman
38) Femenino de 58 años, post colecistomia abierta con exploración de vías biliares, por
colecistitis crónica litiasica más coledocolutiasis. La intervención quirúrgica duró 4 horas,
cursando con hipotension transoperatoria asociada a sangrado y medicamentos. Se le
administran 500 ml de solución Hartmann y dos horas después la paciente sólo ha orinado
50 ml. ¿Cual es la Fisiopatología?
a) Respuesta patológica por falta de líquidos
b) Respuesta fisiológica por vasopresina
c) Obstrucción de la sonda urinaria
d) Respuesta fisiológica por aldosterona
e) Respuesta fisiológica a la anemia

1.- cual es la prueba complementaria MENOS útil en el diagnóstico de la obstrucción intestinal de


las citadas?
a) Enema opaco
b) Rx de torax
c) Rx simple de abdomen
d) Eco abdominal
e) Rx de abdomen en bipedestación

2.- ¿cuál es el esteroide mas abundante y con mayor importancia fisiológica en el organismo?
a) Progesterona
b) Prednisona
c) Corticosterona
d) Cortisol
e) Aldosterona

3.- paciente de sexo femenino de 39 años con antecedente de funduplicatura laparoscópica hace 4
dias, reintervención por perforación de fundus gástrico hace36 hrs; presenta en las últimas 8 horas
hipoperfusión tisular marcada, choque hipovolémico grave, FC 132lpm, disminución del retorno
venoso, anuria, hipoxia severa, hipercarbia, incremento del pico inspiratorio de ventilador y la
presión intravesical de 38 mmHg. El diagnostico mas probable es
a) sx de isquemia aguda mesentérica
b) sangrado post operatorio
c) choque séptico
d) sx de compartimiento abdominal
e) sx de respuesta inflamatoria sistémica

4.- se trata de un paciente el cual se encuentra con manifestaciones clínicas de hiponatremia por
intoxicación hídrica usted decide instalar solución salina al 3% ¿Cuál es la metodología adecuada
para evitar la melanosis pontina?
a) Primero se pasará una carga Hartman de 20 ml por kg
b) Corrección de sodio hasta 130 meq en una hora
c) No deberá suspenderse la solución glucosada
d) No deberá pasar de 1 a 2 meq por hora
e) Se agrega solución de ringer

5.- femenina de 27 años operada de colecistectomía y al día siguiente presenta por drenaje salida
de material biliar en poca cantidad si se considera que la causa es una anomalía anatómica ¿Cuál
sería la causa?
a) Ligadura inadecuada del cístico
b) Hepático anómala
c) Presencia de conductos de luschka
d) Lesión hepático-derecha
e) Lesión de colédoco

6.- paciente masculino de 65 años, fue sometido a hemicolectomía izquierda por adenocarcinoma
de colon, con anastomosis primaria, para continuar con vigilancia deberá de tener seguimiento
¿Cuál de los siguientes es el marcador indicado?
a) GC
b) ACE
c) CA 125
d) AFP
e) AP

7-. Cuando un paciente es portador de esofagitis por reflujo es importante dar tratamiento para
corregir dicha alteración, ya que puede en un año tener el 1% de posibilidades de presentar la
siguiente alteración
a) Epitelio eritematoso
b) Epitelio metaplásico
c) Erosiones
d) Epitelio displásico
e) Estenosis
CIRUGÍA. PREGUNTAS PRIMER PARCIAL
33. After complete removal of a sessile polyp of 2.0 cm by 1.5
cm found 1 finger length above the anal mucocutaneous margin,
the pathologist reports it to have been a villous adenoma that
contained carcinoma in situ. Which of the following is the most
appropriate next step in management?
a. Rescission of the biopsy site with wider margins
b. No further therapy
c. Anterior resection of the rectum
d. External radiation therapy to the rectum
e. Abdominoperineal rectosigmoid resection

32. A 42-year-old man has bouts of intermittent crampy


abdominal pain and rectal bleeding. Colonoscopy is performed
and demonstrates multiple hamartomatous polyps. The patient is
successfully treated by removing as many polyps as possible
with the aid of intraoperative endoscopy and polypectomy.
Which of the following is the most likely diagnosis?
a. Peutz-Jeghers syndrome
b. Villous adenomas
c. Familial polyposis
d. Ulcerative colitis
e. Crohn colitis

31. An 80-year-old man with history of symptomatic


cholelithiasis presents with signs and symptoms of a small-
bowel obstruction. Which of the following findings would
provide the most help in ascertaining the diagnosis?
a. A leukocyte count of 40,000/ml
b. Coffee-ground aspirate from the stomach
c. A pH of 7.5, PCO2 of 50 kPa, and paradoxically acid urine
d. A palpable mass in the pelvis
e. Pneumobilia

12. Paciente hombre con sintomatología esofágica y


extraesofágica, antecedentes de ERGE, se presenta con dolor
abdominal agudo, intenso localizado en región del epigastrio
con irradiación generalizada, tiene rebote positivo,
bioquímicamente tiene leucocitosis con desviación hacia la
izquierda (neutrofilia), por lo que se integra abdomen
intraperitoneal quirúrgico. Se decide realizar laparoscopía
diagnóstica, pero usted recuerda que el procedimiento de
laparoscopía se considera estándar de oro para:
a. Apendicectomía
b. Colecistectomía
c. Gastrectomía
d. Esofagectomía
e. Funduplicatura

26. Al estado crónico del absceso perineal se le conoce cómo?


a. Síndrome hemorroidal
b. Fisura
c. Fístula
d. Absceso perineal
e. Incontinencia anal

11. Paciente masculino de 56 años de edad con manifestaciones


clínicas de sangrado rectal con evolución de seis semanas, sin
pérdida de peso. ¿Cuál es el estudio más apropiado para el
diagnóstico?
a. Radiografía simple de abdomen de pie y en decúbito y tele de
tórax
b. Colon por enema con doble contraste
c. Resonancia magnética nuclear con gadolinio
d. Colonoscopia
e. TAC abdominal virtual

22. Dentro del manejo de los pacientes quirúrgicos, el equilibrio


hidroelectrolítico es esencial para la pronta recuperación. El
proceso inflamatorio que impone la enfermedad y el manejo
quirúrgico influyen directamente en la movilización de los
líquidos corporales. Cuando estas pérdidas por distribución son
importantes, el paciente puede desarrollar hipovolemia. ¿Cuál
sería el trastorno de líquidos más común de este problema
hipovolémico?
a. Pleuresia por trasudado
b. Deficiencia de líquido extracelular
c. Intersticio patológico
d. Ascitis
e. Infiltrado acuoso

2. Se trata de paciente el cual acude a consulta por sensación de


cansancio, dolor, edema vespertino y limitación de los arcos de
movimiento con presencia de tortuosidades de ambas
extremidades inferiores, así como hiperpigmentación. Entre los
signos que presenta el paciente uno de estos corresponde a signo
de afección grave:
a. Dolor
b. Limitación de los arcos de movimiento
c. Edema
d. Hiperpigmentación
e. Venas tortuosas

29. ¿Cuál de los siguientes antibióticos puede alcanzar


concentraciones inhibitorias en los abscesos abdominales?
a. Gentamicina
b. Metronidazol
c. Tetraciclina
d. Ceftriaxona
e. Penicilina G sódica cristalina

24. En el área de hospitalización de cirugía hay 2 pacientes con


antecedente de transfusión y reanimación con líquidos por
diversos grados de choque hipovolémico. Hacia el quinto día los
electrolitos séricos son normales. Cada paciente muestra un
aumento en la diuresis llegando hasta los 100 ml/h; todos niegan
sed, los signos vitales son normales estables. Por lo anterior los
médicos restringen el aporte parenteral de cristaloides. ¿Por qué
se debe restringir el aporte de líquidos en estos pacientes?
a. Porque la hiperglucemia de la respuesta al trauma aumenta la
poliuria
b. Porque la pérdida de sodio del espacio intracelular puede
disminuirse
c. Porque no es necesario aportar el requerimiento basal de
glucosa
d. Porque la falla prerrenal puede mejorarse con esta medida
e. Porque durante la redistribución de líquidos aumenta el
flujo intravascular de retorno

27. Es uno de los mediadores más tempranos y potentes de la


respuesta ---
a. Interferón Gamma
b. Serotonina
c. IL-1
d. TNF alfa 2
e. Histamina

9. Paciente hombre de 18 años con dolor abdominal agudo de 8


horas de evolución el cual refiere signos y síntomas compatibles
con apendicitis aguda no complicada y no modificada, con una
escala de Alvarado de 8 puntos. Acorde a lo mencionado en el
libro de texto Schwartz décima edición. Con respecto al
diagnóstico de apendicitis aguda, una de las siguientes NO es
correcta:
a. Disminución de la concentración de proteína C reactiva y
fiebre
b. Dolor en flanco derecho
c. Respuesta inflamatoria sistémica con leucocitosis y
neutropenia
d. Síntomas gastrointestinales que comienzan después del inicio
del dolor
e. Síntomas gastrointestinales que comienzan antes del inicio del
dolor

15. Paciente mujer de 36 años, politraumatizada, sometida a


laparotomía exploradora por hemoperitoneo. En el segundo día
de postoperatorio manifiesta datos de insuficiencia renal aguda.
¿Cuál de las siguientes opciones define esta complicación?
a. Sobrecarga de volumen
b. Eritrocituria
c. Hipercalcemia
d. Azotemia
e. Piuria

13. Paciente mujer de 23 años la cual presenta dos situaciones


clínicas a considerar, cursa con embarazo y tiene colelitiasis la
cual ha condicionado estados frecuentes de cólico biliar, acude
con usted para su programación quirúrgica. Acorde a lo
establecido en las recomendaciones internacionales y el libro de
texto recomiendan que la colecistectomía se realice en el:
a. Primeros dos meses de embarazo
b. Últimos dos meses de embarazo
c. Tercer trimestre de embarazo
d. Segundo trimestre de embarazo
e. Primer trimestre de embarazo
19. Paciente mujer de 48 años de edad a quien se realizó
resección pancreatoduodenal con tecina de Whipple por
carcinoma de la cabeza del páncreas. Actualmente cursa el
segundo día de postoperatorio y debido a una reposición
insuficiente de líquidos se encuentra en malas condiciones
generales. ¿Cuál de los siguientes datos bioquímicos se
relaciona con el déficit hídrico de esta paciente?
a. Hipoglucemia
b. Hipoalbuminemia
c. Hipocalcemia
d. Isonatremia
e. Hiperazoemia

25. Masculino de 34 años con estreñimiento crónico de más de 5


años, dos días con dolor intenso al evacuar, así como sangrado
rectal en goteo post evaluación, dolor al sentarse. Hemoglobina
de 12 grs. Hematocrito de 38%, leucocitos de 6000, TP 12”,
TPT 34”. El tratamiento quirúrgico del caso consiste en:
a. Fistulectomia
b. Drenaje a cielo abierto
c. Hemorroidectomía
d. Cauterización de la herida
e. Fistulectomia más esfinterotomia lateral

21. Paciente hombre de 30 años de edad, cursa el sexto día de


postoperatorio por resección intestinal secundaria a perforación
traumática, periodo durante el cual ha permanecido en ayuno. El
día de ayer comenzó a fugar liquido intestinal en abundante
cantidad y se desarrolló síndrome de respuesta inflamatoria
sistémica. Signos vitales: TA 110/70 mmHg, FC 95 lpm, FR 16
rpm y temperatura 39°C. ¿Qué signos y síntomas específicos
debe considerar para clasificar el grado de déficit de agua en
este paciente?
a. Análisis clínico, análisis de laboratorio, calculo y prescripción
b. Presencia de sed, pérdidas aumentadas y signos de perfusión
c. Presencia de sed, signos ortostáticos y signos de
hipoperfusión
d. Presencia de sed e hipertermia asociada a SIRS
e. Presencia de sed, pérdidas aumentadas y tipo de material de
fuga intestinal

20. Usted evalúa a un paciente postoperado, a quien se le realizo


una ileostomía, el gasto fue de 2.5 L/24 hrs. El paciente se
refiere con sed intensa, astenia y adinamia, se encuentra
desubicado en tiempo y espacio y su volumen urinario por hora
de 15 ml. El laboratorio actual reporta; Glucosa 120 mg/dl, Urea
80, Creatinina 1.8, Na 127, cloro 85, Potasio 2. Los signos
vitales muestran TA 90/50 mmHg, FC 125 lpm, FR 22 rpm,
temperatura 37°C. ¿Qué tipo de trastornos hidroelectrolíticos
presenta este paciente?
a. Trastornos del control renal de electrolitos
b. Trastornos de volumen, concentración y composición
c. Trastorno de pH y de la osmolaridad
d. Trastornos de distribución e intercambio
e. Trastorno obstructivo y de formación del tercer espacio

10. Femenino de 23 años de edad la cual presenta dolor


abdominal de 48 hrs de evolución, periumbilical que se irradia
hacia fosa iliaca derecha, a la exploración encuentra signos
apendiculares positivos. Leucocitosis de 16,000 cel/mm3, a la
revisión por el servicio de cirugía la paciente refiere mejoría
significativa de los síntomas. ¿Cuál sería la explicación más
probable al estado actual del paciente?
a. Perforación apendicular
b. Colitis
c. Torsión ovárica
d. Embarazo ectópico
e. Enfermedad pélvica inflamatoria
30. Femenina de 38 años, la cual sufrió accidente
automovilístico (choque frontal) hace 4 horas, es rescatada por
paramédicos y llevada al hospital, refiere solamente dolor
abdominal difuso. Se encuentra consciente, orientada,
intranquila con integridad neurológica y motora, tórax con
buena movilidad y ventilación adecuada, el abdomen con
distension y dolor difuso con resistencia muscular voluntaria,
ruidos peristálticos disminuidos. Frecuencia cardiaca de 105
lpm, TA 90/50 mmHg, hematocrito de 32%, hemoglobina de 10
mg/dL. Es el mejor parámetro para evaluar de manera indirecta
perfusión tisular.
a. Oximetría de pulso
b. Gasto urinario
c. Tensión arterial
d. Presión venosa central
e. Llenado capilar

3. Paciente hombre de 72 años el cual acude por claudicación


intermitente de la extremidad inferior izquierda de 20 metros de
distancia con inicio desde hace 3 meses de evolución, con inicio
insidioso y crecimiento paulatino, como antecedente fumador de
más de 40 años de evolución con índice tabáquico de moderada
intensidad así como dislipidemia por lo que recibe tratamiento
con bezafibrato y atorvastatina. En la exploración física presenta
phlegmasia alba dolens + pulsos poplíteo y pedio disminuidos
de extremidad inferior izquierda. Con este diagnóstico usted
manda realizar el estudio considerado estándar de oro el cual es:
a. Ecografía Doppler
b. Ecografía en modo B
c. Angiografía diagnóstica con medio de contraste
d. Angiografía por resonancia con medio de contraste
e. Angiografía por tomografía con medio de contraste
8. Femenino de 23 años quien acude a urgencias con dolor
abdominal de 48 horas de evolución en cuadrante inferior
derecho, de inicio súbito en esta área. Se acompaña de nauseas y
vomito alimenticio, fiebre no especificada, disuria, descarga
vaginal. Presenta menstruación. Exploración con facies
dolorosa, diaforesis, taquicardia, deshidratada, abdomen con
dolor a la palpación profunda y a la descompresión en fosa iliaca
derecha; dolor contralateral al palpar fosa iliaca izquierda, dolor
al pellizcar piel de cresta iliaca derecha. Leucocitos de 22,500
con bandas 25% y segmentados aumentados. Se establece el
diagnostico de apendicitis aguda. La mortalidad aproximada por
apendicitis aguda perforada en esta paciente se estima en:
a. 0.3%
b. 50%
c. 0%
d. 1.7%
e. 10%

1, Masculino de 73 años con ingesta de múltiples medicamentos


antiinflamatorios que presento dolor abdominal de 36 horas de
evolución, nausea y fiebre de 38.7°C; a la exploración física se
identificó abdomen “en madera”. Laboratorio: Leucocitosis de
22,500, neutrófilos del 87% y 6% de bandas. ¿Qué estudio de
gabinete solicitaría de primera elección?
a. Radiografías simples de abdomen y tórax
b. Endoscopia de tubo digestivo alto
c. Ultrasonido abdominal
d. Tomografía computada de abdomen alto
e. Gammagrafía con tecnecio 99

¿Cuál es el dato radiológico que hace poner en duda el


diagnostico de obstrucción mecánica del intestino?
a. Neumoperitoneo
b. Dilatación de intestino delgado
c. Dilatación de intestino grueso
d. Niveles hidroaéreos
e. Gas en ampolla rectal

Paciente masculino de 40 años de edad, bombero, el cual sufre


quemaduras por fuego directo al estar trabajando, a la
exploración sin compromiso cardiorespiratorio, presenta
lesiones en cara anterior de tórax, extremidades, superior e
inferior, indoloras, duras, que no palidecen a la presión, que
afectan menos del 50% de superficie. ¿Para la reanimación
agresiva de líquidos que solución es la más indicada?
a. Solución Mixta
b. Solución Fisiológica
c. Solución
" Ringer Lactato
d. Solución glucosada
1. Femenino de 68, DM de 18 años, post operada por perforación de
colon secundaria a colon tóxico amebiano. Al 4 día post requirió 5. Paciente con trauma múltiple, multitransfundido con 10 U de paquete
asistencia ventilatoria, al 15 hemodiálisis, también se ha tenido que globular. Se le realizó hepatectomía parcial y esplenectomía. Durante
transfundir en 2 ocasiones por sangrado de TD al 5 día. Hoy se reporta el cierre de la pared abdominal inicia con sangrado difuso, se
coluria. Cultivos negativos. ¿Qué evento tisular es causa de su estado consideró asociado a transfusión masiva. ¿Cuál de los siguientes
actual? estudios apoya esta posibilidad?
- Pérdida de la capacidad de defensa tisular - Determinación del factor IX
- Hemorragias petequiales en distintos órganos - Búsqueda de cuerpos de Howell-Joly
- Pérdida de la capacidad de cicatrización de epitelios - Cuenta de plaquetas ✔ ✔ ✔
- Edema y secuestro de líquidos que dificultan la oxigenación - Pruebas de función hepática
- Infiltración por macrófagos activados en distintos órganos ✔ ✔ ✔ - Biopsia de médula ósea
6. Masculino de 60 años de edad con EPOC descompensado por
2. Femenino de 35 en UCI con Dx de drenaje de absceso pélvico.
neumonía. Ingresa a urgencias con signos de choque
Química sanguínea revela urea de 240, creatinina 5. La gasometría
descompensado, destacan clínicamente la dificultad respiratoria, la
reporta hipoxemia e hipercapnia, usa asitencia mecánica,
cianosis, el trastorno del estado de alerta y la disminución de la
presentando mejoría. TA 130/80, 110 lpm, 25 rpm, T 38. Complicación
intensidad de los pulsos radiales y pedios. Sus signos vitales son TA
que presenta la paciente
98/55 FC 130x FR 33x T 38.5°C. en la fisiopatología de este trastorno
- CID complejo:¿qué componentes se encuentran alterados causando el
- Falla orgánica múltiple ✔ ✔ ✔ trastorno ácido-base de este paciente? Co2 de 50 y HCO3 17
- Choque descompensado - Ácido láctico y bicarbonato
- Peritonitis pélvica - Ácido carbónico y ácido láctico✔ ✔ ✔
- Insuficiencia respiratoria
7. Femenina de 62 años a quien se le realizó resección intestinal con
3. Durante la colecistectomía de un paciente de 43 años, para mantener
entero-entero anastomosis termino-terminal por isquemia intestinal. Se
el control sobre el posible sangrado, ¿En qué sitio localiza la arteria
ha manejado con triple antibiótico. ¿Qué grupos generales de
cística?
bacterias debe cubrir dicho esquema?
- En la bolsa de Hartmann - Gram (-), bacilares y cocoides
- En el cuerpo de la vesícula - Gram (-) y (+), esporas
- En el triangulo de hepatocístico ✔ ✔ ✔ - Gram (+ y - ) y anaerobias ✔ ✔ ✔
- Por detrás del colédoco - Gram (+) bacilares y cocoides
- Por delante del conducto hepático derecho - Bacteroides, fusobacterium y peptostreptococcus
4. Femenino de 48 años de edad con antecedentes de DM controlada
8. Masculino de 53 portador de adenocarcinoma de colon sigmoides
con hipoglucemiantes orales. Al iniciar la intervención por vía
con histología de B1 dukes modificada. Tx adecuado
laparoscópica se descubre que la paciente presenta cirrosis hepática
no diagnosticada Y pide que revisen sus exámenes de laboratorio. El
- Ileostomía + QT
TP es de 16* con INR (International normalized ratio) de 1.0 y hay 90,000 - Hemicolectomía izquierda con anastomosis primaria ✔ ✔ ✔
plaquetas ¿cuál decisión debe tomarse? - RT
- Convertir la cirugía a abordaje abierto
- Colostomía definitiva
- Continuar la técnica laparoscópica y dejar drenes ✔ ✔ ✔
- Hemicolectomía + resección de ganglios
- Transfundir plaquetas
- Transfundir plasma fresco
- Suspender la intervención
1 de 28
No es patologico
presencia de conductos de luschka

Trastornos del control renal de electrolitos


Epitelio metaplasico

D
E
correcta: B
texto
A

A
D
C

A
A
A

texto
D
D
9. 15. Paciente mujer de 36 años, politraumatizada, sometida a 13.21. Paciente hombre de 30 años de edad, cursa el sexto día de
laparotomía exploradora por hemoperitoneo. En el segundo día de postoperatorio por resección intestinal secundaria a perforación
postoperatorio manifiesta datos de insuficiencia renal aguda. ¿Cuál de traumática, periodo durante el cual ha permanecido en ayuno. El día
las siguientes opciones define esta complicación? ♡ ♡ ♡ ♡ ♡ ♡ ♡ de ayer comenzó a fugar liquido intestinal en abundante cantidad y se
desarrolló síndrome de respuesta inflamatoria sistémica. Signos vitales:
- a. Sobrecarga de volumen
TA 110/70 mmHg, FC 95 lpm, FR 16 rpm y temperatura 39°C. ¿Qué
- b. Eritrocituria
signos y síntomas específicos debe considerar para clasificar el grado
- c. Hipercalcemia
de déficit de agua en este paciente? ♡ ♡ ♡ ♡ ♡ ♡ ♡ ♡
- d. Azotemia
- e. Piuria - a. Análisis clínico, análisis de laboratorio, calculo y prescripción
- b. Presencia de sed, pérdidas aumentadas y signos de perfusión
10.19. Paciente mujer de 48 años de edad a quien se realizó resección - c. Presencia de sed, signos ortostáticos y signos de hipoperfusión
pancreatoduodenal con tecina de Whipple por carcinoma de la
cabeza del páncreas. Actualmente cursa el segundo día de
- d. Presencia de sed e hipertermia asociada a SIRS
postoperatorio y debido a una reposición insuficiente de líquidos se - e. Presencia de sed, pérdidas aumentadas y tipo de material de
encuentra en malas condiciones generales. ¿Cuál de los siguientes fuga intestinal
datos bioquímicos se relaciona con el déficit hídrico de esta paciente?
♡♡♡♡♡ 14.31. An 80-year-old man with history of symptomatic cholelithiasis
- a. Hipoglucemia presents with signs and symptoms of a small-bowel obstruction. Which
- b. Hipoalbuminemia of the following findings would provide the most help in ascertaining the
- c. Hipocalcemia diagnosis? ♡ ♡ ♡ ♡ ♡ ♡ ♡
- d. Isonatremia
- e. Hiperazoemia - a. A leukocyte count of 40,000/ml
- b. Coffee-ground aspirate from the stomach
11.25. Masculino de 34 años con estreñimiento crónico de más de 5 años, - c. A pH of 7.5, PCO2 of 50 kPa, and paradoxically acid urine
dos días con dolor intenso al evacuar, así como sangrado rectal en
goteo post evaluación, dolor al sentarse. Hemoglobina de 12 grs.
- d. A palpable mass in the pelvis
Hematocrito de 38%, leucocitos de 6000, TP 12”, TPT 34”. El tratamiento - e. Pneumobilia
quirúrgico del caso consiste en: ♡ ♡ ♡ ♡ ♡ ♡ ♡
15.La hiperglicemia en el enfermo crítico se debe a:
- a. Fistulectomia
- Ingesta reciente de alimento
- b. Drenaje a cielo abierto
- Modificación de los receptores de insulina ♡
- c. Hemorroidectomía ♡
- Deterioro de la función de la somatostatina
- d. Cauterización de la herida
- Diabetes mellitus
- e. Fistulectomia más esfinterotomia lateral
- Gluco-neogénesis
12.¿Cuál de los siguientes antibióticos puede alcanzar concentraciones
16.En la enfermedad hemorroidal, las hemorroides se clasifican según su
inhibitorias en los abscesos abdominales?♡ ♡ ♡ ♡ ♡ ♡ ♡
localización y grado de prolapso. Las hemorroides internas se
- Ceftriaxona caracterizan por:
- Penicilina G sódica cristalina - Localización distal en relación con la línea dentada
- Gentamicina - Ser visibles en el exterior del ano
- Tetraciclina - Estar cubiertas por epitelio escamoso
- Metronidazol - Provenir del plexo hemorroidario inferior
- Localizadas proximal mente en relación con la línea dentada

2 de 28
17.Paciente mujer de 65 años de edad, diabética y con diarrea leve, al febrícula disuria sin tratamiento la exploración física reactivo bien
interrogatorio sólo refiere sed, no omesis. Signos vitales: TA 150/70, Fc hidratada cardiopulmonar sin compromiso, abdomen blando
90 lpm, Fr 16 rpm, temperatura 37ºC ¿Cuál vía representa la mejor peristalsis disminuida dolor a la palpación superficial y profunda en
forma de reponer las pérdidas en esta paciente? cuadrante inferior derecho signos apendiculares positivos llenado
- Vía oral capilar inmediato BH 13,300 leucos bandas al 5% eco normal y en la
- Catéter central escala de Alvarado ¿ que puntuación presenta
- Sonda nasogástrica -2
- Sonda nasoyeyunal -7
- Catéter periférico - 8
-6
18.Paciente masculino de 13 años, sin antecedentes de importancia para -5
el problema actual quien presenta dolor abdominal con evolución 19.Paciente masculino de 18 años que consulta por haber notado una
aproximada de 14 horas, iniciando en mesogastrio, difuso, constante, masa en región inguinal derecha, la cual apareció en forma
que se acompañó de vómito en dos ocasiones, evoluciona el dolor espontánea. Al examen físico, esa masa puede reducirse con
con migración hacia el cuadrante inferior derecho abdominal. En la maniobras manuales cuidadosas, y al introducir el dedo explorador al
exploración física se encuentran signos vitales de temperatura de 38.3 anillo inguinal superficial, se percibe el contacto de saco herniario con
la maniobra de Valsalva. ¿A qué tipo de hernia correspondería?
°C, tensión arterial 115/75 mmHg, Pulso: 90 por minuto, frecuencia - Hernia de Spigel
respiratoria: 20 por minuto. Biometría hemática con leucocitos de 18 - Hernia femoral
mil. Se realiza laparotomía exploradora y se encuentra que el - Hernia en pantalón
apéndice es macroscópicamente normal, la conducta recomendada - Hernia inguinal indirecta
es: - Hernia inguinal directa
- Apendicectomía incidental sin drenaje 20.Paciente masculino de 40 años de edad, con dolor abdominal de 6
- Exploración de íleon terminal horas de evolución. Inicialmente el dolor se localizaba en epigastrio y
- Apendicectomía y drenaje actualmente está en fosa iliaca derecha. Los SV son: TA: 110/70 mmHg
- Tomar biopsias de áreas sospechosas FC: 100x’ FR: 24x’ T:39ºC EF: a la palpación existe resistencia voluntaria
- Cierre de la cavidad sin drenajes en la fosa iliaca derecha y rebote (+). Laboratorio: Leucocitos 12,000.
Neutrófilos 90%, en banda: 7. El examen general de orina: proteínas
(trazas) y eritrocitos (+). ¿con que patología debe hacer diagnóstico
1. Paciente masculino de 13 años, sin antecedentes de importancia para diferencial a pesar de que le agnóstico parece claro, dado los
el problema actual quien presenta dolor abdominal con evolución resultados de laboratorio?
aproximada de 14 horas, iniciando en mesogastrio, difuso, constante, - Pancreatitis aguda
que se acompañó de vómito en dos ocasiones, evoluciona el dolor - Colecistitis aguda
con migración hacia el cuadrante inferior derecho abdominal. En la - Colitis espástica
exploración física se encuentran signos vitales de temperatura de - Adenitis mesentérica
38.3°C, tensión arterial 115/75 mmHg, pulso 90 por minuto, frecuencia - Litiasis ureteral derecha
respiratoria 20 por minuto, biometría hemática con leucocitos de 18
mil. El manejo en el hospital de Zoquipan aplicando los criterios de 21.¿Cuál es el diámetro normal del conducto colédoco en un ultra-
diagnósticos de Alvarado deber ser: sonograma?
- Solicitar exámenes complementarios ♡ ¡¡¡¡¡¡¡ - 6 a 7 mm
- 1 a 2 mm
2. Masculino de 9 años de edad traído por dolor abdominal de 18 horas - 13 a 15 mm
en mesogastrio migra cuadrante inferior derecho con anorexia - 3 a 4 mm
- 9 a 10 mm
3 de 28
25.22. Dentro del manejo de los pacientes quirúrgicos, el equilibrio
22.Paciente mujer de 17 años con dolor abdominal agudo con escala de hidroelectrolítico es esencial para la pronta recuperación. El proceso
Álvarado de 4 puntos. El diagnóstico diferencial de apendicitis aguda inflamatorio que impone la enfermedad y el manejo quirúrgico influyen
en la mujer puede ser causado por trastornos ovulatorios por lo que el directamente en la movilización de los líquidos corporales. Cuando
derrame de sangre y líquido folicular puede producir dolor abdominal estas pérdidas por distribución son importantes, el paciente puede
leve. El dolor y la sensibilidad son bastantes difusos, y la leucocitosis y desarrollar hipovolemia. ¿Cuál sería el trastorno de líquidos más común
la fiebre son mínimas o no ocurren dando un diagnóstico denominado de este problema hipovolémico?
mittelschmerz, el cual se caracteriza por presentarse en: - a. Pleuresia por trasudado
- La mitad del ciclo menstrual - b. Deficiencia de líquido extracelular
- El final del ciclo menstrual - c. Intersticio patológico
- El inicio del ciclo menstrual - d. Ascitis de exudado
- La menopausia - e. Infiltrado acuoso
- La fase premenstrual 26.2. Se trata de paciente el cual acude a consulta por sensación de
cansancio, dolor, edema vespertino y limitación de los arcos de
23.11. Paciente masculino de 56 años de edad con manifestaciones movimiento con presencia de tortuosidades de ambas extremidades
clínicas de sangrado rectal con evolución de seis semanas, sin pérdida inferiores, así como hiperpigmentación. Entre los signos que presenta el
de peso. ¿Cuál es el estudio más apropiado para el diagnóstico? paciente uno de estos corresponde a signo de afección grave:
- a. Radiografía simple de abdomen de pie y en decúbito y tele de - a. Dolor
tórax - b. Limitación de los arcos de movimiento ♡
- b. Colon por enema con doble contraste - c. Edema
- c. Resonancia magnética nuclear con gadolinio - d. Hiperpigmentación
- d. Colonoscopia - e. Venas tortuosas
- e. TAC abdominal virtual
27.27. Es uno de los mediadores más tempranos y potentes de la
respuesta
24.Femenina de 38 años, la cual, sufrió accidente automovilístico
- a. Interferón Gamma
(choque frontal) hace 4 horas, es rescatada por paramédicos y - b. Serotonina
llevada al hospital, refiere solamente dolor abdominal difuso. Se - c. IL-1
encuentra consciente, orientada, intranquila con integridad - d. TNF alfa 2
neurológica y motora, tórax con buena movilidad y ventilación - e. Histamina
adecuada, el abdomen con distensión y dolor difuso con resistencia 28.24. En el área de hospitalización de cirugía hay 2 pacientes con
antecedente de transfusión y reanimación con líquidos por diversos
muscular voluntaria, ruidos peristálticos disminuidos. Frecuencia
grados de choque hipovolémico. Hacia el quinto día los electrolitos
cardiaca de 105 latidos por minuto, tensión arterial de 90/50 mmHg, séricos son normales. Cada paciente muestra un aumento en la
hematocrito de 32%, hemoglobina de 10 mg/dl. Es el mejor parámetro diuresis llegando hasta los 100 ml/h; todos niegan sed, los signos vitales
para evaluar de manera indirecta perfusión tisular: son normales estables. Por lo anterior los médicos restringen el aporte
- Gasto urinario ♡ parenteral de cristaloides. ¿Por qué se debe restringir el aporte de
líquidos en estos pacientes?
- Tensión arterial - a. Porque la hiperglucemia de la respuesta al trauma aumenta la
- Oximetría de pulso poliuria
- Llenado capilar ♡ - b. Porque la pérdida de sodio del espacio intracelular puede
disminuirse
- Presión venosa central - c. Porque no es necesario aportar el requerimiento basal de glucosa
- d. Porque la falla prerrenal puede mejorarse con esta medida
- e. Porque durante la redistribución de líquidos aumenta el flujo
intravascular de retorno ♡
4 de 28
32.3. Paciente hombre de 72 años el cual acude por claudicación
29.9. Paciente hombre de 18 años con dolor abdominal agudo de 8 horas intermitente de la extremidad inferior izquierda de 20 metros de
de evolución el cual refiere signos y síntomas compatibles con distancia con inicio desde hace 3 meses de evolución, con inicio
apendicitis aguda no complicada y no modificada, con una escala de insidioso y crecimiento paulatino, como antecedente fumador de más
Alvarado de 8 puntos. Acorde a lo mencionado en el libro de texto de 40 años de evolución con índice tabáquico de moderada
Schwartz décima edición. Con respecto al diagnóstico de apendicitis
intensidad así como dislipidemia por lo que recibe tratamiento con
aguda, una de las siguientes NO es correcta: bezafibrato y atorvastatina. En la exploración física presenta
- a. Disminución de la concentración de proteína C reactiva y fiebre phlegmasia alba dolens + pulsos poplíteo y pedio disminuidos de
- b. Dolor en flanco derecho extremidad inferior izquierda. Con este diagnóstico usted manda
- c. Respuesta inflamatoria sistémica con leucocitosis y neutropenia realizar el estudio considerado estándar de oro el cual es:
- d. Síntomas gastrointestinales que comienzan después del inicio del - a. Ecografía Doppler
dolor
- e. Síntomas gastrointestinales que comienzan antes del inicio del - b. Ecografía en modo B
dolor - c. Angiografía diagnóstica con medio de contraste ♡
- d. Angiografía por resonancia con medio de contraste
30.20. Usted evalúa a un paciente postoperado, a quien se le realizo una
- e. Angiografía por tomografía con medio de contraste
ileostomía, el gasto fue de 2.5 L/24 hrs. El paciente se refiere con sed
intensa, astenia y adinamia, se encuentra desubicado en tiempo y
33.8. Femenino de 23 años quien acude a urgencias con dolor
espacio y su volumen urinario por hora de 15 ml. El laboratorio actual
abdominal de 48 horas de evolución en cuadrante inferior derecho, de
reporta; Glucosa 120 mg/dl, Urea 80, Creatinina 1.8, Na 127, cloro 85, inicio súbito en esta área. Se acompaña de nauseas y vomito
Potasio 2. Los signos vitales muestran TA 90/50 mmHg, FC 125 lpm, FR 22 alimenticio, fiebre no especificada, disuria, descarga vaginal. Presenta
rpm, temperatura 37°C. ¿Qué tipo de trastornos hidroelectrolíticos menstruación. Exploración con facies dolorosa, diaforesis, taquicardia,
presenta este paciente? deshidratada, abdomen con dolor a la palpación profunda y a la
descompresión en fosa iliaca derecha; dolor contralateral al palpar
- a. Trastornos del control renal de electrolitos
fosa iliaca izquierda, dolor al pellizcar piel de cresta iliaca derecha.
- b. Trastornos de volumen, concentración y composición Leucocitos de 22,500 con bandas 25% y segmentados aumentados. Se
- c. Trastorno de pH y de la osmolaridad establece el diagnostico de apendicitis aguda. La mortalidad
- d. Trastornos de distribución e intercambio aproximada por apendicitis aguda perforada en esta paciente se
- e. Trastorno obstructivo y de formación del tercer espacio estima en:
- a. 0.3%
- b. 50%
31.10. Femenino de 23 años de edad la cual presenta dolor abdominal - c. 0%
de 48 hrs de evolución, periumbilical que se irradia hacia fosa iliaca - d. 1.7%
derecha, a la exploración encuentra signos apendiculares positivos. - e. 10%
Leucocitosis de 16,000 cel/mm3, a la revisión por el servicio de cirugía
34.Paciente con perforación de sigmoides de 8 hrs de evolución. Si se
la paciente refiere mejoría significativa de los síntomas. ¿Cuál sería la
aplican antibióticos previos a cirugía. Concepto correcto en este caso
explicación más probable al estado actual del paciente?
- Es profilaxis
- a. Perforación apendicular
- Requiere de antibióticos
- b. Colitis
- No se requiere antibióticos
- c. Torsión ovárica
- Es indispensable para profilaxis cultivo
- d. Embarazo ectópico
- La herida Qx es clase 1
- e. Enfermedad pélvica inflamatoria

5 de 28
35.Masculino de 65 años sometido a hemicolectomía izquierda por 40.A la exploración física de un paciente usted encuentra triada de
Adenocarcinoma de colon, con anastomosis primaria. Para continuar Charcot ¿por lo que su razonamiento clínico es?
con vigilancia debe de tener seguimiento. ¿ Cuál de los siguientes es - colangitis
el marcador indicado?
- AP - hidrocolecisto
- ACE - colelitiasis
- CA 125 - colecistitis
- AFP - coledocolitiasis
- GC
36.Paciente de 55 con rectorragia, con evolución de 3 meses. ¿Cual 41.En relación a la causa de presentación de fiebre en Cx en el post
estudio debe ser importante para su diagnóstico? ¿cuáles son las más frecuentes?
- Eco
- TAC - Infección de herida Qx y venopunción
- Rectosigmoidoscopia - Infección de herida Qx, IVU, neumonías
- Exploración anal - Neumonía y bronquitis
- Colonoscopía - Catéter de sonda urinaria y neumonía
37.Citocina proinflamatoria, activador primario de la reacción febril en un - Tromboflebitis, atelectasia
px sometido a laparotomía por perforación de íleon
- IL 1 42.Femenino de 53 pesa 70 kg. Está en shock hipovolémico por diarrea
- IL 6 coleriforme. Se ha iniciado fluidoterapia y para vigilar su respuesta se
- TNF
- IL 10 ha colocado una sonda urinaria. Para considerar que el riñón tenga
- IL 2 una perfusión sanguínea adecuada ¿Cuánto debe tener de gasto
38.Hombre de 52 años a quien se le realizó colecistectomía abierta por urinario?
colecistitis aguda litiásica no complicada al salir del quirófano TA - Más de 100 ml/hr
130/80 FC 75 FR 16 temperatura de 36.5 °C en reposo prolongado se - 10 a 20 ml/hr
considera factor de riesgo por cualquier causa en todos los pacientes - 35 a 70 ml/hr
¿ cuándo es conveniente indicar la deambulación a la paciente?
- 15 a 30 ml/hr
- 8 días después
- 6 días después - 90 a 100 ml/hr
- cuando la paciente lo pida
- al día siguiente de la intervención 43.Masculino de 58 con EPOC, fue intervenido de urgencia por abdomen
- 2hrs después agudo el diagnóstico postoperatorio fue piocolecisto perforado. Fue
extubado y continuó su vigilancia posoperatoria en el área de
39.Mujer de 23 años, cursa embarazo y tiene colelitiasis, la cual ha recuperación la SaO2 de 85% y con respiración superficial. 150/70
condicionado estados frecuentes de cólico biliar. Acude para su mmHg, 110 lpm, 18 rpm, T 36, PH 7.33, HCO3 18, Na 135, Cl 110, PCO2
progresión quirúrgica. ¿Cuándo se recomienda hacer la 48 mmgh. ¿cuál es el trastorno ácido base presente?
colecistectomía? - Ac Mixta NO compensada ♡
- Primeros 2m del embarazo - Alcalosis R NO compensada
- 2 trimestre - Ac Met compensada
- 1 trimestre - Ac Met con AG elevado
- 3 trimestre - Al Met
- Últimos 2m del embarazo

6 de 28
44.Masculino de 20 a quien se le practicó laparotomía por apendicitis 48.Paciente hombre con sintomatología esofágica y extraesofágica,
perforada. Al 5 días postoperatorio presenta fiebre persistente, venas antecedentes de ERGE, se presenta con dolor abdominal agudo,
del cuello colapsados, 90/50 TA, 130 lpm, 32 rpm, t 39, llenado capilar 4 intenso localizado en región de epigastrio con irradiación generalizada
y rebote positivo, bioquímicamente tiene leucocitosis con desviación a
segundos y pulsos débiles, diuresis en 8 hrs de 100 ml. ¿qué la izquierda (neutrofilia) por lo que se integra abdomen agudo
diagnóstico integra con estos datos? intraperitoneal quirúrgico. Se decide realizar laparoscopia diagnóstica.
- Atelectasia Recuerda que el procedimiento de laparoscopia se considera
- Infarto superior estándar de oro para:
- Shock séptico - Funduplicatura
- Esofagectomía
- Embolia pulmonar - Gastrectomía
- Tamponamiento cardiaco - Apendicectomía
- Colecistectomía
45.Portador de apendicitis. El área de asepsia del abdomen que deberá
ser realizada considerando los siguientes límites 49.Femenina de 18 años sometida a Cx por apendicitis. Al 8 día postQx,
- Cresta iliaca a reborde costal presenta fiebre. Posible causa
- IVU
- Cicatriz umbilical a sínfisis del pubis - Absceso residual
- Ángulo de Lewis a sínfisis del pubis - Atelectasia
- Línea submamaria a tercio superior de muslos - TEP
- Apéndice xifoides a sínfisis del pubis - Venoclisis
50.Se le inicia ayuno previo Cx. El ayuno es una agresión al organismo,
46.Se realiza colonoscopia a px con antecedentes de sangrado rectal. sin embargo, debe prescribirse esta indicación antes de todo evento
¿Cuál de los siguientes tiene la posibilidad de desarrollar carcinoma invasivo. ¿Por qué es necesaria esta prescripción?
de colón? - Induce adaptación temprana al ayuno postoperatorio
- Hemorroides - Elimina riesgo de estimulación vagal
- Disminuye riesgo de broncoaspiración
- Pólipo adenomatoso - Mantiene la vía aérea permeable
- Divertículos - Que el paciente vomite durante la Cx
- Fístula perianal 51.A 45 woman with history of heavy ingestion of AINE’S presents with
- Hemangioma acute abdominal pain. She undergoes exploratory laparotomy 30 hrs
after onset of symptoms and is found to have a perforated duodenal
47.A 55 man complains of chronic intermittent epigastric pain. A ulcer. Tx
gastroscopy demonstrates a 2 cm prepyloric ulcer. Biopsy of the ulcer - Simple closure with omental patch
yields no malignant. After a 6 week trial of medical therapy, the ulcer is
- Highly selective vagotomy with omental patch
- Truncal vagotomy and antrectomy
unchanged. Which of the following is the best step in his management - Truncal vagotomy and pyloroplasty
- Vagotomy and pyloroplasty - Hemigastrectomy
- Repeat trial of medical therapy 52.Antibiótico profiláctico en operaciones de de intestino grueso
- Local excision of the ulcer - Clindamicina
- Partial gastrectomy with vagotomy and Billroth 1 reconstruction - Metronidazol
- Highly selective vagotomy - Cefotaxima
- Quinolona
- Amikacina
7 de 28
5. Paciente femenino de 45 años de edad sometida a histerectomía de
SE REPETÍA MUCHO urgencia por sangrado persistente. El hallazgo transoperatorio fue
1. Al estado crónico del absceso perianal se le conoce cómo? necrosis de miomas uterinos. Ha recibido la transfusión de 5 unidades
- Fístula de paquete globular y 5 unidades de plasma fresco. En el
- Absceso perianal transoperatorio aparecen datos de sangrado en capa por lo que se
- Fisura piensa en CID. ¿Cuál de los siguientes apoya este diagnóstico?
- Síndrome hemorroidal - Antitrombina III disminuida
- Incontinencia anal - Crenocitos abundantes en la orina
- Presencia de productos de lisis de fibrinógeno
2. Paciente masculino de 52 años de edad, alcohólico crónico, presenta - Presencia de reticulocitos
gran distensión abdominal, con anasarca y dificultad respiratoria, su - Disminución del factor de Hageman
gasometría revela un PH 7.44, PaCO2: 28 mmHg, PaCO2: 80 mmHg y
HCO3: 28. ¿Cuál es el trastorno ácido-base primario en este paciente? 6. Paciente masculino de 39 años de edad, con antecedente de herida
- Acidosis metabólica penetrante de abdomen por arma blanca. Durante el transoperatorio
- Alcalosis respiratoria compensada se encuentra hemoperitoneo de 2 litros, su piel muestra los signos
- Alcalosis mixta clásicos de hipoperfusión. Mientras la piel y vísceras intraabdominales
- Acidosis respiratoria son sacrificadas, contrariamente corazón, cerebro y músculo se
- Alcalosis metabólica mantienen perfundidos. De las siguientes opciones. ¿Cuál da
explicación a este hecho fisiopatológico?
3. El médico de emergencias le pide evaluar a una paciente de 46 años - Cierre selectivo de los esfínteres precapilares aumento de
contractilidad cardiaca
de edad, con dolor abdominal en el cuadrante superior derecho, debe
- La vasodilatación en los órganos críticos y músculo
insistirse en la descripción del dolor, para definir la sospecha - Efecto inotrópico y cronotrópico negativo de catecolaminas
diagnóstica. ¿Cuál de los siguientes enunciados concuerda con el - Efectos vasoconstrictor de la PaCO2 aumentada
cuadro clínico de esta paciente? - La constricción de lechos capilares de los órganos críticos
- Si el dolor es postprandial e irradiado a hombro derecho, es
7. Paciente masculino de 43 años de edad quien en accidente
colecistitis.
automovilístico sufre factura de ambos fémures. A la evaluación inicial
- Si el dolor se irradia a la región inguinal, se trata de calculo ureteral lo encuentra con alteración del estado de alerta, con piel fría humeda
- Si el dolor se irradia a hombro izquierdo, se trata de colecistitis y de aspecto marmóreo. Sus signos vitales son: TA 80/40 FC 120 x FR
- Si el dolor disminuye con el vómito, es una pancreatitis 28x y T 35.6ºC. ¿Cuál de los siguientes enunciados explica el evento
- Si el dolor se inicia después de la ingesta de alimentos, se trata de fisiopatológico presente en este paciente?
- Incremento del efecto de la hormona reguladora
úlcera perforada
- Acumulo de ácido volátil y desviación álcali del bicarbonato
- Hipoperfusión tisular sistémica
4. En presencia de íleo por un cálculo biliar, la mayoría de las veces la - Apertura de esfínteres pre capilares y de vasos de capacitancia
obstrucción se hallará en: - Disminución de la perfusión en cerebro y corazón
- Sigmoides
8. Masculino de 43 sufre fractura de fémur y pelvis. Evaluación inicial
- Ileon proximal alteración de alerta, piel fría húmeda y marmórea. TA 80/40, 120 lpm,
- Ileon terminal 28 rpm, T 35.6. Principal determinante en la sobrevida del px
- Duodeno - Tiempo de hipoperfusión
- Yeyuno - Manejo temprano de falla orgánica múltiple
- Demora en aporte nutricional
- Control temprano de sus Fx
- Combinación de ayuno e hipotermi
8 de 28
12.Ingresen los pacientes varones de 28 y 25 años de edad, quienes
9. Se encuentra en el servicio de cirugía, al pase de visita se le pide sufrían volcadura. Cursa el segundo día de estancia en unidad de
coloque sonda para preparación gástrica a un paciente, el cual será cuidados intensivos, actualmente se encuentran en ayuno y sin
sometido mañana a endoscopía diagnóstico-terapéutica por datos necesidad de vas opresores, con diuresis adecuada. Llama la atención
compatibles con retención gástrica condicionando un mega esfínter. que sin ser diabéticos, mantienen cifras altas de glucemia. ¿Cuál de la
Derivado a la indicación usted recuerda que la mejor opción del tipo siguientes sustancias promueve la gluconeogénesis evidente de estos
de sonda debido a las características y tiempo de duración es: pacientes?
- Robisson - Cortisol
- Rush - Insulina
- Foley-Alcock - Hormona luteinizante
- Nelaton - Grelina y leptina
- Levin - Hormona folículo estimulante
10.Paciente femenino de 49 años de edad, quien ingresa a la unidad de 13.Masculino de 68 años de edad, el cual fue sometido a la resección
cuidados intensivos por choque séptico intestinal; Se han reportado abdominoperineal por presentar cáncer de recto, durante la cirugía,
volúmenes adecuados de solución de Ringer-lactato (Hartmann) y las pérdidas sanguíneas fueron respuestas volumen a volumen. En el
postoperatorio fue manejado con sus soluciones glucosadas al 5%,
vasopresores, ha sido guiado por presión venosa central y la diuresis. A presentando debilidad, letargia, convulsiones y posteriormente coma.
pesar del manejo, ahora se han agregado dificultad respiratoria, Por lo referido, el paciente está cursando con cual problema
ictericia y confusión mental. ¿Cuál es la complicación que presenta electrolítico?
esta paciente? - Hipopotasemia
- Choque séptico refractario - Hipercalcemia
- Hiperpotasemia
- Síndrome de respuesta inflamatoria sistémica - Hiponatremia
- Sepsis severa - Hiperglucemia
- Hepatitis reactiva
- Trastorno vascular cerebral agudo 14.Con respecto a la anatomía de la región inguinal, el epónimo que
recibe el ligamento inguinal se le conoce con el nombre de:
11.Paciente femenino de 58 años de edad, post operada de - Poupart
- Gimbernat
colecistectomía abierta con exploración de vías biliares, por - Cooper
colecistitis crónica litiásica más coledocolitiasis. La intervención - Pectíneo
quirúrgica tuvo una duración de 4 horas, cursando con hipotensión - Lagunar
transoperatoria asociada a sangrado y medicamentos. Se le
administraron solamente 500 ml de solución Hartman y dos horas 15.Trastorno que se asocia con frecuencia, con una concentración de
después, la paciente sólo orinado 50 ml. ¿Cuál es la fisiopatología de potasio anormalmente baja en el plasma, como consecuencia de la
este hecho? LAS 3 ESTÁN BIEN amortiguación celular:
- Respuesta patológica por falta de líquidos - Acidosis metabólica
- Obstrucción de la sonda urinaria - Acidosis respiratoria
- Respuesta fisiológica a la anemia - Alcalosis metabólica
- Respuesta fisiológica por aldosterona - Alcalosis respiratoria
- Respuesta fisiológica por vasopresina - Todas las anteriores

9 de 28
16.Paciente masculino de 22 años de edad, quien sufrió caída de un 20.Primeros síntomas claros de Ca colorrectal
cuarto piso. Se realizó hepatectomía parcial y recección de 30 cm de - Cambio en defecaciones y hemorragia rectal
intestino delgado, recibió 15 unidades de paquete globular en el pre y - Hemorragia rectal y dolor abdominal
transoperatorio, 5 litros de solución Hartmann y 4 unidades de plasma - Masa en CID y dolor
fresco congelado. Durante el cierre de la laparotomía, inicia con - Meteorismo y dispepsia
sangrado difuso de la cavidad abdominal (en capa). ¿Cuál es la - Pérdida de peso
siguientes opciones es la causa más probable de este sangrado?
- Hiperesplenismo 21.Respecto a la distribución de líquidos, en un mujer de 70 kg las
- Plaquetopenia siguientes aseveraciones son verdaderas EXCEPTO
- Deficiencia del factor IX - 3-3.5 L corresponden a plasma
- Insuficiencia hepática
- Hipoprotombinemia - Agua extracelular es de 14 L
- Agua extracelular es 40 L
17.Femenino de 15 años de edad quien acude por abdomen doloroso de - Agua corporal total comprende 55-60% de masa corporal total
12 horas devolución en 15 sospecha apendicitis aguda. El cirujano - Agua intracelular es de 22 L
revise el expediente de la paciente y le informa que le pasará a
quirófano una vez que usted recabe el resultado de laboratorio cuál 22.Las siguientes aseveraciones son ciertas con respecto a la fase EBB de
valora la vía intrínseca de la coagulación. ¿Qué estudio es éste?
- Fibrinógeno la respuesta metabólica al trauma, EXCEPTO
- Tiempo de sangrado - Ocurre inmediatamente posterior a la lesión
- Tiempo parcial de tromboplastina - Duración de 12-24 hrs
- Tiempo de trombina - Reducción de presión sanguínea, GC, temperatura
- Tiempo de protombina - Existe un estado hipermetabólico
18.¿Cuál es el dato radiológico que hace poner en duda el diagnostico - Se asocia a hemorragia
de obstrucción mecánica del intestino?
- Neumoperitoneo ♡ 23.¿Cuál es el esteroide más abundante y con mayor importancia
- Dilatación de intestino delgado fisiológica en el organismo?
- Dilatación de intestino grueso - Cortisol
- Niveles hidroaéreos - Prednisona
- Gas en ampolla rectal ♡ - Aldosterona
- Progesterona
19.Masculino de 73 años con ingesta de múltiples medicamentos - Corticoesterona
antiinflamatorios que presento dolor abdominal de 36 horas de
evolución, nausea y fiebre de 38.7°C; a la exploración física se
identificó abdomen “en madera”. Laboratorio: Leucocitosis de 22,500, 24.La hiperglucemia en el enfermo crítico se debe a:
neutrófilos del 87% y 6% de bandas. ¿Qué estudio de gabinete - Deterioro de función de la somatostatina
solicitaría de primera elección? - Ingesta reciente de alimento
- Radiografías simples de abdomen y tórax ♡ - Gluconeogénesis ♡
- Endoscopia de tubo digestivo alto ♡
- Modificación de receptores de insulina ♡
- Ultrasonido abdominal
- Tomografía computada de abdomen alto - DM
- Gammagrafía con tecnecio 99

10 de 28
25.Paciente del sexo femenino de 47 años con antecedente de
funduplicatura laparoscópica hace 3 días, reintervención por 29.32. A 42-year-old man has bouts of intermittent crampy abdominal
perforación de fundus gástrico hace 24 horas; presenta en las últimas 6 pain and rectal bleeding. Colonoscopy is performed and demonstrates
horas hipoperfusión tisular marcada, choque hipovolémico grave, multiple hamartomatous polyps. The patient is successfully treated by
frecuencia cardíaca de 120 latidos por minuto, disminución del retorno removing as many polyps as possible with the aid of intraoperative
venoso, anuria, hipoxia severa, hipercarbia, incremento del piso endoscopy and polypectomy. Which of the following is the most likely
inspiratorio del ventilador y la presión intravesical de 38 mmHg. El diagnosis?
diagnóstico más probable es: - a. Peutz-Jeghers syndrome ♡
- choque séptico - b. Villous adenomas
- síndrome de compartimiento abdominal - c. Familial polyposis ♡
- sangrado post operatorio
- síndrome de isquemia aguda mesentérica - d. Ulcerative colitis
- síndrome de respuesta inflamatoria sistémica - e. Crohn colitis

26.La señora Lucia de 65 años de edad hipertensa de más de 20 años


con cifras de 160/100 en el momento de la visita con el anestesiólogo,
está programa para una colecistectomía, ¿Qué riesgo quirúrgico
presentará?
- Clase 1
- Clase 2
- Clase 3
- Clase 4
- Clase 5
27.Paciente masculino de 75 con antecedentes de diabetes mellitus,
sedentarismo y tabaquismo, acude por presentar hematoquecia de 1
semana de evolución, además de periodos de diarrea y ocasional
estreñimiento, acompañados de dolor difuso, ¿con los datos cuál sería
su diagnóstico? (cambios en defecación y hematoquecia=CA colon)
- Amibiasis invasora
- Ca colon
- Apendicitis complicada
- Diverticulitis complicada
- Trombosis arterial mesentérica
28.33. After complete removal of a sessile polyp of 2.0 cm by 1.5 cm
found 1 finger length above the anal mucocutaneous margin, the
pathologist reports it to have been a villous adenoma that contained
carcinoma in situ. Which of the following is the most appropriate next
step in management?
- a. Rescission of the biopsy site with wider margins ♡
- b. No further therapy
- c. Anterior resection of the rectum
- d. External radiation therapy to the rectum ♡
- e. Abdominoperineal rectosigmoid resection
11 de 28
5. Paciente femenino de 48 años de edad que será sometida a
NO SE REPETÍA histerectomía por presentar metrorragia recurrente por hiperplasia
1. Paciente masculino el cual tiene varias horas de presentar cuadro de endometrial. Será intervenida una vez que cuente con valoración por
oclusión intestinal baja actualmente con vómitos fecaloides cuál es el cardiólogo ya que además es portadora de hipertensión arterial mal
mecanismo relacionada con las características de los vómitos? controlada. ¿Qué estudios son indispensables para su evaluación
cardiológica preoperatoria?
- Existe antiperistalsis - Ecocardiograma bajo estrés con dobutamina
- Se relaciona con la peristalsis - Monitoreo tipo Holter
- La oclusión es funcional - CPK fracción MB, TGO y Troponina
- La colonización de intestino delgado - Radiografía de tórax y EKG
- La materia fecal tiene movimiento retrógrado - Prueba de esfuerzo en banda sin fin y EKG
6. Cuál es el tipo de drenaje que debe utilizarse en un paciente de 9 años
2. Femenino de 42 años de edad sometida a colecistectomía abierta es
masculino postquirúrgico de apendicitis aguda fase cuatro?
neumópata y su estado de nutrición es regular. ¿Qué material de
- Saratoga
sutura sugiere para el cierre de su aponeurosis?
- Pudenz
- Vycril 3/0 - Penrose
- Catgut crómico 1 - Sonda en T
- Seda 3/0 - Drenovac
- Polipropileno 1
- Catgut simple 1 7. En un paciente con pérdida del estado de alerta, que sufrió una lesión
3. Niño recién nacido en quien la madre notó, en ambas regiones en la región y nariz queda por proyectil por arma de fuego, con
inguinales, una masa que aumenta de tamaño cuando el bebé llora. El sangrado profuso a través del orificio de entrada. ¿Cuál es la conducta
médico del hospital, comenta que se trata de dos hernias. ¿Cuál es el que se debe adoptar?
defecto congénito que predispone a la formación de estas hernias? - Continuar con el ABC e ignorar la hemorragia
- Defecto de los pilares del anillo inguinal profundo - Compresión directa y continuar el ABC
- Permeabilidad del proceso peritoneo vaginal - Pinzado hemostático y continuar el ABC
- Defecto del mecanismo obturador del anillo inguinal profundo - Control de la hemorragia ya que tiene prioridad
- Defecto de los pilares de anillo en el superficial - Aplicar torniquete controlado proximal y continuar con el ABC
- Defecto de la fascia transversalis
8. Paciente masculino de 60 años a quien se realizó laparotomía
4. En un paciente adulto que tiene compromiso de la vía aérea, es exploradora previa colocación de catéter central. A los dos y ese post
imposible iniciar la ventilación asistida y se requiere de manera operatorio, durante el día de ambulación presenta disnea e
urgente ventilar el paciente. ¿Cuál de los siguientes métodos es insuficiencia respiratoria, en la placa de tórax se observa una zona de
posible por punción percutánea? condensación pulmonar basal derecha datos que se encuentran en
- Traqueostomía relación con qué complicación pulmonar:
- Cricotomía simple - Tromboembolia pulmonar
- Intubación - Derrame pleural
- Traqueostomía - Neumotórax
- Cricotiroidostomía - Neumonía
- Infarto pulmonar
12 de 28
13.Se le conoce a la condición que presenta un paciente, al referir
9. Paciente femenino de 37 años que ingresa a urgencias por haber vómito con sangre:
sufrido caída desde la montura de 1 caballo, inmediatamente usted - Hematoquezia
nota que la paciente se encuentra parapléjica, descubre defecto de - Melena
sensibilidad a nivel de la metámera T10. Los signos vitales son TA: 80/50 - Hematemesis
mmHg FC: 130x’ FR: 22x’ T:37ºC. Presenta pilo erección distal desde la - Epistaxis
metámera T8. ¿Qué tipo de shock presente esta paciente? - Guayaco
- Neurogénico
- Séptico 14.Femenina de 69 años con antecedentes de cesárea hace 35 años y
- Anafiláctico cirugía de región umbilical hace cinco años. Cuadro clínico de 4 horas
- Cardiogénico devolución caracterizado por dolor abdominal infraumbilical que
- Hipovolémico aumenta con el ejercicio y disminuye con el reposo, estado nauseoso
y masa de 9 cm, infraumbilical dolorosa, no reductible a las
10.Paciente hombre de 48 años de edad de 65 kg de peso, postoperado maniobras, peristalsis normal. Hemoglobina de 13.1 y leucocitos 8000.
de cáncer gástrico, cursa 3er día de postoperatorio con emesis Qué es el factor de riesgo más frecuente para recurrencia de la hernia:
frecuentes, su diuresis ha sido 2,000 ml en 24 h, por laboratorio la Hg 13 - Cirugía de urgencia
gr/dL, glucosa 200 mg/dL, urea de 20 mg/dL, Na 144 meq, Cl 98 meq, K - Anemia
2.5 meq ¿Cuál es el déficit de potasio porcentual que presenta el - Fibrosis pulmonar
paciente? - Infección del herida
- 5% - Tabaquismo
- 10%
- 15% 15.¿Cuál es el elemento más importante del canal inguinal?
- 20% - Pared posterior
- 25% - Pared anterior
- Pared inferior
11.Considerando la osmolaridad plasmática dentro del rango de 280-300
- Anillo inguinal superficial o externo
mosm y los índices de natremia entre 135-145 meq, ¿Cuál de las - Anillo inguinal profundo o interno
siguientes soluciones cristaloides se le considera como solución
hiperosmolar hipotónica? 16.Uno de los siguientes que se mencionan, pertenece a los factores
- Hartmann intrínsecos del paciente para las complicaciones en la cirugía:
- Antecedentes personales patológicos
- NaCl al 0.9% - Tricotomía 45 minutos antes de la cirugía
- Glucosada al 5% - Asepsia
- Glucosada al 10% - Antisepsia
- Mixta (NaCl al 0.9% + Glucosada al 5%) - Ventilación del quirófano adecuad

12.Paciente masculino de 28 años de edad, el cual presenta hernia 17.En la reanimación inicial con líquidos intravenosos en un paciente con
inguinal sin compromiso, no acepta cirugía ¿Cuál sería la choque hemorrágico se utiliza preferentemente:
complicación más grave? - Sangre total
- Encarcelación - Plasma
- Orquitis-isquemia
- Seminoma - Cristaloides isotónicos
- Estrangulación de contenido - Paquetes globulares
- Expansores de plasma

13 de 28
18.Topografía donde debe realizarse la colocación de un mini sello 23.Paciente masculino de 40 años de edad, bombero, el cual sufre
pleural: quemaduras por fuego directo al estar trabajando, a la exploración sin
- 2º espacio intercostal, línea media clavicular compromiso cardiorespiratorio, presenta lesiones en cara anterior de
- 5º espacio intercostal, línea media axilar tórax, extremidades, superior e inferior, indoloras, duras, que no
- 4º espacio intercostal, línea axilar posterior palidecen a la presión, que afectan menos del 50% de superficie.
- 6º espacio intercostal, línea axilar anterior ¿Para la reanimación agresiva de líquidos qué solución es la más
- 5º espacio intercostal, línea medio clavicular indicada?
- Solución Mixta
19.Usted acude al servicio de urgencias a valorar un paciente que - Solución Fisiológica
presenta cuadro de oclusión intestinal como antecedente tiene dos - Solución Ringer Lactato
cirugías previas cuál sería la causa más probable? - Solución glucosada
- Trombosis arterial 24.Masculino de 35 con herida penetrante en región poplítea. EF: TA
- Adherencias 90/50, 125 lpm, T 36, 20 rpm. No hay sangrado activo. ¿Componente
- Volvulos reconocido que se activa a la exposición de colágeno y previene el
- Neoplasia de colon sangrado? “hemostasia primaria”
- Hernia ventral - Fosfolípido
20.Masculino de 69 años de edad con EPOC Y actualmente como que - Plasminógeno
pronto espasmo leve y tos frecuente. Ingresa al servicio de - Fibrina
emergencias presente del hospital comunitario de segundo nivel - Plaquetas
donde se intervino quirúrgicamente por colecistitis crónica o izada. El - Fibrinopéptido
motivo de traslado es evisceración. Además de controlar el EPOC, se 25.Masculino de 31 con contusión abdominal. TA 90/50, 145 lpm, 24 rpm,
programará para el cierre de la pared. ¿Cómo debe manejarse el T 35.5. Piel muestra cambios clásicos de choque. Enunciado que
plano aponeurótico de esta herida? describe las características de la piel del px
- Aplicando puntos con ácido poliglicólico #00 - Piel fría, húmeda, pálida sin cianosis
- Colocando una malla de Marlex pre-peritoneal - Rubincunada, rojo cereza, caliente
- Afrontando con grapas de titanio - Fría, seca, eutérmica sin cianosis
- Aplicando puntos con polipropileno #1 - Fría, palidez, húmeda, pegajosa
- Suturando con crómico reforzado #1 - caliente, sudorosa, pegajosa con acrocianosis
21.Se trata de masculino de 22 años de edad con antecedente de 26.Masculino de 35 pesa 70. Padece pancreatitis necrótica, tiene 3 días
apendicectomías de 5 días por apendicitis fase 4. La herida quedó de evolución. Sin fiebre, tiene sed, TA 100/70, 110 lpm, 24 rpm, diuresis
abierta en los planos de grasa subcutánea y piel y ahora se encuentra horaria 20 ml/hr. ¿Dónde se encuentra el líquido faltante?
limpia y granulada. ¿Cómo debe manejar esta herida? - Espacio transcelular
- Se le brinda el tejido de granulación y se colocan puntos separados - Tercer espacio secuestro
- Bajo anestesia local, aseo de la herida - 2 espacio
- Aplicando hidrocoloides dentro del herida - 1er espacio
- Continuar con curaciones antisépticos - Espacio intracelular
- Es momento de saturarla con puntos separados
27.Femenino de 58, con DM, presenta datos de insuficiencia venosa
22.Indicación RELATIVA para un abordaje transtorácico para la periférica sometida a colecistectomía por piocolecisto. Medida para
funduplicatura de Nissen evitar TEP
- Hernia paraesofágica - O2 y heparina
- Esplenectomía previa - Nebulizaciones
- Hernia Hiatal 1 - Ejercicios resp pre Cx
- Reparación previa de hernia hiatal - Enoxaparina y medidas de compresión
- Hernia hiatal grande - Antibióticos y enoxaparina
14 de 28
28.Femenina de 38 años que sufre accidente automovilístico choque 33.Trastorno ácido base causado por hipoventilación que conduce a
frontal hace cuatro horas abdomen distendido dolor difuso ruidos retención de CO2
peristáltico los disminuidos frecuencia cardiaca de 105 lpm TA 90/ 50 - Alcalosis R
mmHg hematocrito 32% hemoglobina de 10 mg/dl el mejor parámetro - Alcalosis R + Ac R
para evaluar de manera indirecta la perfusión tisular DUDA - Ac M
- presión venosa central - Alcalosis M
- llenado capilar - Ac R
- oximetría de pulso
- tensión arterial 34.Masculino de 57 con HAS, DM2. Hace 9 años tuvo colecistectomía y
- gasto urinario funduplicatura. Tiene 3 meses de cambios intestinales, diarrea,
evacuaciones con moco y sangre fresca escasa, tenesmo rectal,
29.Masculino de 78 con antecedentes de tos crónica de 5 años Tx con sensación de distensión abdominal, meteorismo, aerofagia, peristalsis
loratadina, 4-5 tequilas semanales x 20 años. Hace 4 meses presenta aumentada, pérdida de 4 kg en 6 meses. Abdomen blando y
disfagia a sólidos, halitosis, regurgitación y aumento de volumen depresible, peristalsis normal, Tacto rectal con masa tumoral fungosa a
postprandial inmediato del cuello. EF pérdida ponderal de 10 kg, 7 cm del margen anal de aproximadamente 6 cm de dm, que ocluye
lengua saburral. Hb 11.2. Dx parcialmente la luz intestinal. 1er estudio a realizar
- ERGE - USG rectal
- Fístula traqueoesofágica - TAC de abdomen
- Divertículo de Zenker - RM
- Acalasia esofágica - Colonoscopia con biopsia
- Espasmo difuso del esofágo - Marcador tumoral

30.Presencia de precipitaciones (litos) en vesícula 35.Paciente masculino cumple 24 horas de posoperatorio de


- Colecistitis funduplicatura tipo Nisse, el cual presenta fiebre cuantificada en
- Hidropesia vesicular 38.7°C, diaforesis, astenia y adinamia. Usted requiere iniciar protocolo
- Colangitis de estudio de las causas de fiebre por lo que solicitará los siguientes
- Colelitiasis estudios considerando la mejor opción:
- Coledocolitiasis - EGO + Tele de Tórax + Descartar Flebitis

31.Paciente con posible litiasis vesicular, 1er estudio a realizar 36.Px donde NO es prioridad establecer protocolo de nutrición parenteral
- TAC de forma inmediata
- Colangio percutánea - Hipertenso
- Eco - Diabético
- RM - Politraumatizado
- CPRE - Oligúrico
- Pneumopata
32.Principal catión en el cuerpo y que desempeña un papel esencial en
muchos procesos metabólicos y fisiológicos 37.Durante la reanimación de un paciente con herida penetrante de
- Ca abdomen por proyectil, presenta shock hipovolémico. Sol adecuada
- K - Haemaccel
- Mg - Plasma fresco
- Fósforo - Hartman
- Na - NaCl 0.9%
SODIO ES EXTRACELULAR Y POTASIO INTRACELULAR - Dextran 40
15 de 28
38.Femenino de 44 con Dx de úlcera péptica programado para realizar
endoscopia. ¿Cuál debe preceder al estudio?
- Px en decúbito ventral
- Sonda vesical
- Verifica que se encuentre en ayuno de 8 h
- Aplicación de enema evacuante
- Colocación de sonda nasogástrica

16 de 28
NO TIENE OPCIONES 10.ingresa masculino de 22 años ingresa al servicio de urgencias
1. La úlcera de etiología arterial se localiza preferentemente en la planta refiriendo dolor en fosa iliaca derecha. se diagnostica apendicitis y se
del pie: realiza apendicectomia. el hallazgo quirúrgico fue apendicitis en fase
- Verdadero edematosa (fase 1) al momento de su egreso hospitalario se le
retiraran los puntos a los:
2. La úlcera de etiología venosa se localiza preferentemente en el dorso - 7 dias
del pie.
- Falso 11.masculino de 22a el cual refiere dolor en fosa iliaca derecha el
paciente fue sometido a apendicectomia el hallazgo quirúrgico fue
3. La estructura anatómica que se genera a nivel de los vasos apendicitis perforada (fase iv) que tipo de cierre para la piel y tejido
epigástricos inferiores como su límite superolateral + la vaina del celular subcutáneo es recomendado en este caso:
músculo recto como borde medial + los ligamentos inguinales
- diferido
pectíneo como límite inferior se le conoce con el nombre de triángulo
de: 12.masculino de 23a quien refiere dolor abdominal de predominio en
- Hesselbach epigastrico y fid, para posteriormente irradiarse a todo el abdomen
con hipertermia y vomitos. a la ef en abdomen peristalsis ausenbte con
4. Las complicaciones mecánicas de las várices son:
rebote (+++) se interviene de urgencias encontrándose apendicitis
- Varicorragia externa e interna grado iv. durante el cierre el manejo adecuado para la piel es:
5. El complejo flebostático se caracteriza por edema celulitis, dermatitis,
- cierre deferido
eczema e hiperpigmentación, ulceración de la piel en el tercio inferior
13.femenino de 38a multipara, con cistocele programada para
de la pierna, así como várices.
colpoperineoplastia la preparación de la vagina se reliza en:
- Verdadero
- quirófano
6. ¿Cuál es la prueba complementaria MENOS útil en el diagnóstico de la
obstrucción intestinal, de las citadas? 14.masculino de 22a sometido a la laparotomía con hallazgo de
- Rx de tórax perforación del ileon distal por salmonellosis, al 3er. dia postoperatorio
inicia con fiebre, dolor, tumefacción, hiperemia y aumento de la
7. La trombosis venosa profunda cursa con un cuadro clínico de fiebre temperatura local, la causa probable de esta complicaion es:
elevada, cefaleas, náuseas y enrojecimiento de la extremidad - falta de colocación de drenes en la cavidad ♡
- Falso - cierre primario de la herida ♡
8. femenino de 40a con sirpa y desnutrición proteico calorica en la 15.masculino de 72a obeso, sometido a colecistectomía abierta por
alimentación intravenosaperiferica el incremento de calorías se colecistitis aguda al 4to, día post operatorio presenta edema. dolor,
lograra mediante: hipertermia y coloración violácea en el miembro inferior derecho, cual
- lípidos al 10% es la medida preventiva para esta complicación:
- uso de medias elásticas
9. paciente de 56a a quien se le realizo antrectomia con reconstrucción
biliar por presentar ulcera gástrica penetrada a páncreas se encuantra 16.femenino de 48a multipara, obesa, programada para histerectomía
en el 3er dia postoperatorio con sonda nasogastrica, sonda vesical vaginal por prolapso uterino, como antecedente menciona ser
con hartman glucosada y ciprofloxacino 500mg iv cada 12 hrs, resp diabética y presenta insuficiencia venosa de miembros inferiores, cual
16x/ pulso 75x ta 130/85. cuando se debera retirar la sonda de levin en de las siguientes complicaciones pulmonares es probable que
este paciente: presente esta paciente en el postoperatorio:
- al presentar ruidos intestinales - tromboembolia pulmonar
17 de 28
17.paciente de 50a con cáncer de esófago y con inanición en esta 24.paciente politraumatizado el cual encuentra en la uci debemos tener
circunstancia su corazón y músculo tienen predilección por uno de los en cuenta que durante la respuesta metabólica al trauma la acción de
siguientes aminoácidos: las hormonas contrarreguladoras tiene como respuesta en el
- de cadena ramificada organismo agredido:
- aumento de la gluconeogenesis
18.masculino de 36a quien durante la inanición aguda debido a cáncer 25.ante un paciente que se encuentra cursando el primer día del
laríngeo. tiene una perdida neta de aminoácidos, ya que la síntesis de postoperatorio por haber sido sometido a tireidoctomia subtotal por
proteína muscular desaparece o desciende tanto que el catabolismo presentar bocio multinodular una vez superada la fase ebb, usted
de la misma permanece sin cambios esta situación es debida a: espera que el paciente se encuentre en la fase:
- concentraciones bajas de insulina ♡ - de flujo
- aumento de glucagon ♡ 26.ante un paciente que fue sometido a drenaje abierto de absceso
hepático amibiano, el cual se encuantra en las primeras horas
19.femenino de 28a politraumatizado que se encuentra en el
postoperatorio fase ebb, usted esperaria encontrar un aumento de la
postoperatorio que pesa 90kg talla 1.5m, que ingiere 10gm de
siguiente sustancia producida por el organismo. cual es:
proteinas al dia.en 2 litros de orina esta eliminando 950mg/ dia, de
- tiroxina
nitrógeno ureico. con esto el balance de nitrógeno al día es de:
- 21.4 gm 27.femenino de 30a quien sufre accidente en carretera. en el lugar del
accidente se le encuentra semiconsciente, taquicardia, en torax
20.paciente que fue intervenido quirúrgicamente realizándosele pulmones ventilados, abdomen con paristalsis disminuida y dolor
colecistectomia por presentar colecistitis crónica litiasica, con el generalizado. se le canaliza con hartman y es llevado al centro de
postoperatorio mediato presento dehisciencia de la herida quirúrgica. trauma mas cercano. de las siguientes cual es la que disminuye en la
para reparar la complicaion y cerrar la aponeurosis eligira el siguiente fase inicial en la respuesta biológica al trauma:
material de sutura: - cortisol
- prolene 1
28.paciente que presento traumatismo abdominal cerrado y choque
21.masculino de 60a que presenta una bola en region inguinal derecha hipovolemico, el punto de vista fisiopatologica a nivel hormonal,
que se desaparece con el reposo, no dolorosa que no llega al escroto, esperaria encontrar aumento de la secrecion de la siguiente hormona:
a la ef la masa toca el pulpejo del dedo explorador a travez del piso - cortisol
inguinal en la reparación quirúrgica de este defecto. cual es la sutura
29.femenino de 25a quien sufrió accidente automovilistico con lesiones
mas adecuada:
leves. pero fue víctima de crisis emocional, se le toma gasometría
- prolene
revelando una pco2 de 30 mmhg tal cifra y el cuadro clínico sugieren:
22.masculino de 45a el cual sufrió de traumatismo abdominal en - alcalosis respiratoria
accidente automovilistico y quien a su ingreso a urgencias llega en
estado de choque hipovolémico para que desencadene la respuesta 30.ante un paciente el cual presenta vomitos frecuentes de contenido
biológica al trauma deberá existir: gástrico secundario a estenosis pilorica la anormalidad metabólica
- integración nociceptiva en le cerebro predominante que esperaria encontrar seria:
- alcalosis hipocloremica
23.femenino de 34a quien sufrió politraumatismo en accidente 31.femenino de 28a la cual presento trauma abdominal cerrado con
automovilístico y que es visto en el servicio de urgencias. en el estudio ruptura hepática e isquemia intestinal por desgarro del mesenterio,
de este paciente se sabe que dentro de la respuesta metabólica al durante una intervención quirúrgica presenta sangrado difuso de la
trauma se produce una serie de hormonas llamadas pared abdominal y en hígado debido a una probable cid, el
contrarreguladoras dos de las cuales son: tratamiento que realizará en este momento será:
- epinefrina y noradrenalina - empaquetamiento abdominal
18 de 28
32.femenino de 39a con dolor en hipocondrio derecho se realiza usg de 38.femenino de 28a se le realizo cesárea por presentar obito de 24
páncreas. higado y via biliar y se detecta enfermedad calculosa de la semanas de gestación y a quien en el postoperatorio se le diagnostico
vesícula por lo que fue sometida a celecistectomia simple se solicitan cid. con el objeto de reponer la mayor cantidad de los factores de la
examenes preoperatorios los cuales son normales, si usted desea coagulacion consumidos, cual de las siguientes medidas es la mayor
detectar deficiencia del factor ii cual es el estudio util para evaluarlo: fuente de los factores de la coagulacióń́.
- tiempo de protrombina - plasma fresco congelado
39.masculino de 53a quien ingreso a uci por presentar hipotensión severa
33.masculino de 42a quien ingresa con sangrado de tubo digetivo alto.
con cifras tensionales de ta 70/40. revelado a la aplicación de
entre sus antecedentes menciona alcoholismo crónico. laboratorio: hb
volúmenes adecuaos de ringer lactato e inotropicos, además de fiebre
10gm/ dl, hto 33%, plaquetas 300,000. tpt 40́. a la exploración física
e insuficiencia renal, si finalmente con las medidas anteriores la ta no
paciente tranquilo; consiente ta. 100/60, se coloca sonda nasogastrica
se recupera usted optaría por indicar:
y se le realiza lavado gástrico, dentro del manejo de este paciente
- aplicacion de vasopresina
indica transfusión de:
- concentrado plaquetario 40.femenino de 28a quien ingresa por sangrado transvaginal
determinándose como causa aborto incompleto en el primer trimestre
34.paciente el cual fue sometido a hemicolectomia por cáncer de colon. por lo que se realiza legrado uterino a consecuencia de ello la
en el postoperatorio presenta cuadro compatible con cid la alteración paciente esta en choque hipovolémico la diferencia de o2 arteria
laboratorial que esperaría encontrar será: venoso en pulmón en la fase temprana del choque hipovolémico se
- presencia de p. d. f ♡ encuentra:
- antitrombina III diseminada ♡ - pequeña diferencia del o2 arteriovenoso
35.masculino de 23a quien presenta desde hace 3 días, ictericia, fiebre, 41.masculino de 53a que ingreso a la uci por presentar estabilización del
dolor abdominal, heces hiperpigmentadas y orina clara. a la choque hipovolemico y mejor control, por lo que se coloca un cateter
exploración existe dolor a la palpación del cuadrante izquierdo donde de swanz ganz para valorar:
se palpa una masa que sobre sale del borde costal y sigue los - p. v. c
movimientos respiratorios, este cuadro se repite una o dos veces al 42.masculino de 60a a quien se le realizo resección intestinal en el
años y se presenta también en 2 hermanos suyos, el diagnostico postoperatorio inmediato el electrolito que refleja mejor la alteración
probable es:
en la composición de los comportamientos de los liquidos corporales
- hiperesplenismo es:
- na+
36.masculino de 60 años que es ingresado a cirugía para lavado de
43.femenino de 40a la cual ingreso a la uci por presentar choque séptico
cavidad peritoneal por sepsis abdominal en quirófano. presenta paro
secundario a una perforación intestital. se le han aplicado volúmenes
cardiorrespiratorio, a la exploración física ruidos cardiacos poco
adecuados de ringer lactado controlados por la presion en cuña
audibles, frecuancia respiratoria no perceptible. no hay pulsos en
pulmonar, dopamina y norepinefrina a pesar de las medidas anteriores
carotidas y femorales, con estos datos clinicos es determinante para el
la paciente no ha mostrado mejoria, es mas se ha agregado ictericia y
diagnostico de paro:
confusión mental. por lo que muestra el paciente esta presentando:
- perdida de movimientos respiratorios ♡
- síndrome de disfunción orgánica múltiple
- pérdida del pulso en una arteria central (carótida o femoral) ♡
44.masculino de 55a el cual fue sometido a gastrectomía subtotal por
37.masculino de 45a a quien se le realizo cesárea y reseccion de 30cm presentar cancer in situ en el postoperatorio mediato es indispensable
de intestino delgado por traumatismo abdominal cerrado. durante la el uso de anelgesicos con horario, para prevenir la siguiente
cirugía se le transfundieron 10 unidades de sangre del banco. a complicación:
presentado sangrado difuso y piensa en cid, esta complicaion se
- síndrome de disfunción orgánica múltiple ♡
evidenciara por el siguiente resultado de laboratorio:
- presencia de productos de división de fibrina - atelectasia ♡
19 de 28
45.masculino de 47a el cual ha padecido de dolor abdominal recurrente 52.femenino que abarca una quemadura que abarca una superficie
durante los últimos 3 años. actualmente presenta dolor epigástrico corporal quemadura en toda la cara anterior del tronco, miembro
intenso y continuo, nauseas de 6 horas de vomito que no alivian el superior derecho, mano izquierda en la cabeza quemaduras en cejas,
dolor en los últimos momentos el dolor tiende a irradiarse al dorso y pestañas y vibrisas quemadas y expectoracion carboneada además
hemiabdomen izquierdo a la exploración presenta resistencia de inconciencia. según elcalculo de las regiones lesionadas. por su
abdominal epigastrica. con los datos anteriores el padecimiento será: superficie corporal equivale a:
- pancreatitis aguda - 50-60%
53.masculino de 30a quien presenta quemaduras secundarias a
46.masculino de 47a el cual ha presentado dolor abdominal de tipo exposición de fuegos pirotécnicos en cara, manos, genitales, torax 70%
ardoso desde hace 3 años, que lo despierta por las noches, desde para valorar la perfusion renal durante el tratamiento de reposición de
hace un año toma de forma irregular antiácidos y bloqueadores h2 liquidos:
prescritos por facultativo, sin mejoria aparente, actualmente el dolor es - deberá vigilarse diuresis horaria a razón de 30-50
continuo, mas intenso y se irradia al dorso. la terapeutica adecuada
es: 54.masculino de 30a con quemaduras del 60% de segundo grado
- gastrectomía parcial supericiales y profundas. el cual ha estabilizado hemodinamico, la
47.masculino de 22a ingresa al servicio de urgencias en donde usted se dieta debe hacerse por la via:
encuentra, refiriendo dolor en fosa iliaca derecha por lo que se le - oral
diagnostica apendicitis, de los siguientes exámenes preoperatorios
cuales serian de importancia: 55.paciente que sufre accidente automovilístico y que al examen se
- rx simple de abdomen, tp y tpt, bh, qs encuentra despierto y responde a las preguntas a nivel de tórax se
encuentran datos que sugieren neumotorax a tensión en hemitorax
48.masculino de 67a prostático diabético, obeso y con epc, quien hace derecho, la medida a aplicar para este problema será:
un mes empezó a notar masa tumoral en región inguinal derecha que - toracocentesis en 2º espacio intercostal
desaparecer rápidamente al adoptar el decúbito dorsal para manejar
estas condiciones siguientes el tratamiento indicado en este paciente 56.un paciente politraumatizado quien en el examen le encontramos en
es: hemitorax izquierdo silencio respiratorio a la percusión timpanismo y
- adoptar el uso de branguero con choque de la punta a la derecha realizándosele toracocentesis de
49.paciente de 50 años a quien se le practico laparotomía media urgencia, la justificación de esta medida es por:
exploradora presenta en el 5° día del postoperatorio una eviceracion. - es la elección para manejar la confusión pulmonar ♡
esta complicación se define como: - urgencia de descomprimir el mediastino ♡
- separación de todos los planos de la herida
57.masculino de 34a llevado a urgencias por sufrir accidente
50.paciente de 45a el cual presenta una masa en region inginal izquierda
automovilistico al examen encontramos hipersensibilidad leve en
desde hace 3 años la cual se hace evidente al esfuerzo y disminuye
abdomen y hemitorax derecho con peristalsis muy disminuida y
con el reposo. no llegando al escroto en la exploración al pedirle que ausencia de matidez hepática cual es su impresión:
puje la masa hace contacto con el pulpejo del dedo explorador en
- ruptura de vicera hueca
este caso el defecto se encuentra en:
- dentro del triangulo de hasselbash 58.masculino de 41a el cual sufrió accidente automovilístico en el
momento de arribar a la ambulancia usted detecta que el, paciente se
51.femenino de 45a quien refiere la presencia de una masa a nivel encuentra semiconsciente con cianosis periférica, lesion maxilofacial
abdominal. no dolorosa. a la ef se encuentra masa de 2x2 cm . poco importante. movimientos respiratorios apenas perceptibles y la
móvil, blanda, no dolorosa, entre apéndice xifoides y cicatriz umbilical. presencia de fractura expuesta de tibia y perone, considerando las
en la linea media, en este caso cual seria la terapéutica mas condiciones en las que se encuantra el paciente cual es la primer
adecuada: medida que aplicaria:
- plastia - cricotiroidostomia
20 de 28
65.paciente que sufrió quemadura de 2° que abarcan el 30% de la
59.paciente de 28 a politraumatizado en accidente por motocicleta no superficie corporal en esta paciente el incremento de las necesidades
llevando casco protector es admitido a urgencias con cianosis y calóricas basales serán en un % de
severa dificultad respiratoria, ta80/40 y sangrado nasal. además de - 100-200
fractura expuesta de fémur ruidos respiratorios ausentes en hemitorax
derecho. la prioridad del menejo será: 66.en un paciente quemado de 70 kg de peso con 30% de superficie
- obtener acceso iv para transfusión de emergencia de sangre tipo O quemada de segundo grado, necesitara durante sus primeras 8 horas
la siguiente cantidad de solución (parkland):
60.masculino de 44a el cual sufrio accidente automovilistico al momento - 12,000ml
de arribar en la ambulancia usted detecta que el paciente se 67.ante un paciente politraumatizado y que fue sometido a laparotomía
encuentra semiconsciente, con cianosis periferica, lesion maxilofacial exploradora podemos encontrar una diferencia que no existe en el
importante, movimientos respiratorios apenas perceptibles y la paciente sometido a ayuno cual es esta?
presencia de fractura expuesta de la tibia y perone. cual es la medida
- activación del sistema neuroendocrino
que aplicaria:
- colocaion de la canula de gedele 68.en un paciente politraumatizado se produce una activacion del
problema neuroendocrino cuya reperfusion metabólica nutricional
61.masculino de 34a el cual sufrio accidente automovilistico en carretera.
produce:
al momento de arribar con la ambulancia usted detecta que el
- aceleración del catabolismo proteico muscular
paciente se encuentra semiconsciente, signos vitales: ta110/70, fr90x́,
fc88x́, cianosis periferica , lesion maxilofacial importante, movimientos 69.femenino de 28a con sepsis abdominal a la ef paciente somnolienta.
respiratorios superficiales y la presenciadefracturaexpuestade tibia. signos vitales: 80/60, fc120x, fr36x, su tratamiento inmediato consiste
cual es la conducta mas importante que debe realizar: en:
- cricotiroidectomia - control de la infección y mantenimiento del volumen sanguíneo
70.femenino de 28a con sepsis abdominal y a la exploración física
62.masculino de 29a presenta dolor abdominal en fid. continuo, paciente somnolienta signos vitales ta 80/60, fc 110x, en el choque
acompañado de hipertermia y nauseas, a la exploración física, mc séptico. la hiperventilación se encuentra en la fase:
burney (+), rovsing (-), por lo que es programado a cirugía, una vez - temprana
que usted se vistio con ropa quirúrgica para ingresar al área gris
además usted debe colocarse: 71.masculino de 56a quien padece de dolor abdominal desde hace 5a
- botas ardoroso en epigastrio, tomando antiáz presenta dolor intenso
arradiado todo el abdomen. a la ef peristalsis ausente, resistencia
63.masculino de 60a con diagnostico de hernia incicional, infraumbilical muscular con rebote presente (+++) se decide pasar a cirugía dentro
gigante, se programa cierre del defecto herniario, para evaluar su de las incisiones cual es la más adecuada:
riesgo quirúrgico debemos evaluar: - media supra infraumbilical
- historia clinica
72.masculino de 22a ingresa a urgencias refiriendo dolor en fid por lo que
64.ante un paciente que se encuentra hospitalizado y cursando 2° día se diagnostica apendicitis de los siguientes examenes preoperatorio
postoperatorio por complicaciones de pancreatitis necrotica cual seria de mayor importancia:
hemorragica y de quien se sospecha que curse con una desnutrición - ekg
grave, en estas circunstancias desde el punto de vista laboratorial
podemos corraborar nuestro diagnostico se la albúmina se encuantra 73.paciente de 42a con historia de cuadros dolorosos en cuadrante
dentro del siguiente rango: superior derecho acompañados de nauseas y vomitos de 2 años de
- 1.5-20 g/dl evolucion actuelmente con coluria, ictericia y acolla. que estudio se
usa para confirmar el diagnostico:
- ultrasonografia
21 de 28
74.masculino de 30a quien presenta desde hace 3 años dolor epigastrico 83.masculino de 60a el cuala va a ser sometido a reseccion transuretral
recurrente, el que se presenta comúnmente por la madrugada. desde de próstata. como antecedente cuanta con infarto al miocardio hace 4
hace 6 horas presenta hematemesis, su diagnostico orienta hacia: meses se cataloga como riesgo anestesico quirúrgico clase:
- ulcera peptica - IV
75.en un paciente con ulcera peptica perforada, dentro de la ef a la
84.femenino de 23a la cual se encuantra en urgencias con diatrosis,
percusión esperaria encontrar: ta90/50, fr24x́, fc100x́, temp 38°, a la cual se le diagnostico trombosis
- desapacricion de la matidez hepática mesenterica, como antecedente el paciente es diabético desde hace
10 años manejado con hipoglucemiantes orales según la clasificacion
76.masculino de 65a el cual presenta hernia inguinal directa. el manejo
de asa. su riesgo quirúrgico sera de:
de esta condicion puede hacerse por cualquiera de las siguientes vias:
- via retroperitoneal, via peritoneal, via peritroescopia, via trato ileo -V
purico
85.femenino de 27a quien a sufrido caida de lesion de bazo, higado, asi
77.masculino de 67a prostatico, diabético, obeso y con epog, quien hace como de intestino delgado, se transfuenden mas de 10 u de sangre si
un mes empezo a notar masa tumoral en region inguinal derecha que el sangrado persistiera a pesar del sufiecinte remplazo de coagulantes
desaparece rapidamente al adoptar el decubito dorsal. para manejar se pensaria en la presencia actual de:
esta condicion el tratamiento indicado en este paciente es: - coagulación intravascular diseminada
- tratar de disminuir la obesidad del paciente y luego operarlo
86.femnino de 15a quien es asaltada en la via publica, a la ef se
78.masculino 60a prostatico, diabético, obeso quien desde hace tres dias encuentra diseña, inquietud, dolor, fr40x silencio respiratorio y claridad
empezo a notar una masa en region anguinal izquierda hemiesterica pulmonar aumentada en la rx de torax el diagnostico es:
que desaparece rápidamente al adoptar el decubito dorsal. por todas - neumotorax a tensión
las patológicas que presenta este paciente los mas probables es que
87.masculino de 29a que condiciendo su automovila alta velocidad sufre
la deformación mencionada corresponda: choque, y a la ef se encuantra con ta 100/60, ec100x́, fr24x, en el resto
- hernia inguinal directa del examen en hemitorax derecho murmullo vesidular desminuido con
79.masculino de 67a con diagnóstico de hernia inguinal derecha dentro matidez a la persecución. cual seria el tratamiento terapéutico para su
de los estudios paraclinicos preoperatorios. los mas importantes en el problemapulmonar:
caso son: - punción con trócar en 6º eic
- tele de torax y elec trocardiograma 88.en un paciente que sufre herida por arma blanca en torax izquierdo.
80.masculino de 45a quemado por intoxicación con co2 quien presenta con ta90/50. ingurgitacion yugular por lo qu se le realizo
disnea y quemaduras en cara las cuales no son dolorosas, el paciente pericardiocentesis de urgencias. eltratamiento consiste en:
debera haber presentado por lo menos uno de los siguientes datos a la - toracotomia y miocardiograma
exploración física:
- capa cubierta con hollín negro 89.masculino de 30a que sufre accidente automovilistico por lo que es
traido a urgencias, encontrándosele semiinconsciente. hipotenso,
81.femenino 48a que va aser sometida a histerectomía por presentar cianotico. en cuallo ingurgitacion yugular con gractura de arcos
cacu la siguiente medida antitrombotica es: costales en hemitorax izquierdo se le toma la p.v.c. la cual esta
- medias elasticas elevada. cual es el diagnostico:
- hemopericardio
82.femenino 40a con diagnostico de miomatosis uterina por lo que se va
a practicar hiterectomia total abdominal, no cuenta con antecedentes 90.en un paciente politraumatizado al cual en la ef se reporta ta 100/60,
de importancia, por lo cual estara clasificada con un riesgo anestesico fr32x, abdomen distendido. silencio abdominal , rebote (++),
quirúrgico clase: timpanismo generalizado. que estudio corrobora el diagnoctico:
- IV - tele de torax
22 de 28
91.masculino quien en sala de operaciones previa a la intubación, 97.masculino de 30a postoperatorio de hernioplastia inguinal derecha
presenta paro cardiorrespiratorio sus pulsos centrales sondebiles, su ta indicada en el 2° dia postoperatorio presenta aumento de volumen de
60/50. frecuencia respiratoria superficial, dada la bradicardia del la herida con fluctuación, no dolorosa, no se observan datos de celulitis
paciente, cual es el medicamento usado para su manejo: ni hipertermia con estos datos la complicaion que presenta el paciente
- adrenalina endotraqueal es:
- dehiscencia
92.masculino de 38a quien se recibe en urgencias con antecedentes de
98.femnino de 22a quien va a ser sometido a laparoscopia pélvica por
lsiones en torax y abdomen no penetrantes. asi como fracturas
multiples en extremidades inferiores a su llegada ta 60/40 ruidos probable endometriosis durante la asepsia del area quirúrgica (piel)
cardiacos poco perceptibles, respiración superficial, pulsos en femoral cual de las siguientes sustancias considera que es la mas adecuada
para relizar este procedimiento:
y en carotidaausentes sediagnostica meniobra de compresión externa
y desfibrilacion, persiste con arritmias ventriculares. cual es la dosis de
- yodopovina (isodine)
lidocaina indicada para el tratamiento: 99.femenino de 38a que se encuentra en el 5to dia postoperatorio por
- bolo img/ kg iv colectomia total para el tratamiento de su colecistitis ulcerativa
crónica inespecífica, la cirugía se efectua sin complicaciones
93.masculino de 50a quien ingresa al servicio de urgencias con cerrando las capas de la herida con surgente, como antecedentes de
antecedente de haber sufrido accidente automovilistico, a su llegada importancia menciona ser diabética tipo 1, en el 2do dia
presenta desorientación, a su llegada presenta desorientación, ta postoperatorio presento acumulo de la sangre en la herida y hoy
50/40, frecuencia cardiaca no audible sin respiración espontánea, observa eviceracion. la causa mas probable de la complicación es:
pulsos en carotidas ausentes. por lo que se diagnostica paro - infección de la herida
cardiorrespiratorio. se inician maniobras de reanimacion con
compresiones toraxicas y respiraciones, en el monitor se observa
100.femenino de 17 años con diagnostico de púrpura trombocitopenica
fibrilacion ventricular, cual es el siguiente paso par continuar con el
idiopatica refractaria a tratamiento medico, programada para
tratamiento:
esplenectomia. después de la asepsia y antisepsia del area quirúrgica,
- iniciar desfibrilación e intubar al paciente
la colocación de campos y sabanas se indica con:
94.f e m e n i n o d e 5 5 a c o n c h o q u e s é p t i c o y d i s f u n c i ó n - sabana podalica
organicamúltiple.alaef paciente somnoliento, ruidos cardiacos
disminuidos, frecuencia respiratoria no perceptible pulsos centrales 101.femenino de 40a quien presenta dolor abdominal en abdomen
ausentes, se hace diagnostico de paro cardiorrespiratorio, se inicia superior y que se le exacerva con los alimentos grasos. a la
masaje cardiaco externo la meta fundamental en toda sesion de exploración murphy (+), el ultrasonido de higado y vias biliares revela
reanimacion con la compresión es: vesícula de 11x9cm, e imágenes hiperecogenicas en su interior, para
- restaurar la función cardiaca normal lo más pronto posible la cirugía en que area se viste con la ropa quirúrgica:
95.femenino de 65a quien presenta sepsis abdominalsecundaria - negra
diverticulitis colonica complicada, se realiza lavado y drenaje de la
cavidad, en el post operatorio la paciente cardiopulmonar la cual es 102.masculino de 71a quien fue sometido a colecistectomia con incisión
efectiva usted indica que la paciente es trasladada a: media supraumbilical en el 5to dia del postoperatorio presento
- su cama con control estricto de signos vitales evisceracion. la causa más probable de esta complicacion es:
96.femenino de 60a quien ingresa a urgencias por accidente - técnica quirúrgica inadecuada
automovilistico al llegar al hospital sus signos vitales son: ta60/40,
ruidos cardiacos poco perceptibles, respiración superficial, minutos 103.masculino de 22a presenta dolor abdominal en fid continuo,
después de su llegada al servicio de urgencias presenta paro cardiaco acompañada de hiperemia y nauseas, a la exploración rovsing (+) y
respiratorio, cual es la medida que inicio para restaurar la funcion blunberg (+). en el punto de mc burney para el aseo quirúrgico de sus
cardiaca: manos, cual de las siguientes sustancias antisépticas utilizara:
- aumentar el volumen sanguíneo por medio de soluciones - hi-biscrub (gluconato de clorhexidina)
23 de 28
104.en el piso de cirugía, se discute el caso de un paciente que presento 114.ingresa paciente a urgencias con herida en la cara anterior de
deshicencia de la herida quirúrgica en el 5° dia del postoperatorio se antebrazo de aproximadamente 3cm, de longitud, realiza el aseo de
hace mencionar que fue debido a mala técnica quirúrgica en el cierre la herida de la piel con isodine, cual es el tiempo minimo necesario
de la herida esta complicación implica: para obtener el efecto esperado:
- separación parcial o total de los planos de la herida - 10 minutos
105.femenino de 48a quien ingresa a urgencias con cuadro de abdomen 115.paciente de 58a programado para gastrectomía subtotal por cancer
agudo secundario a colelititis complicada. a la exploración encuantra gástrico en etapa temprana como antecedente meciona padecer
talla 1.60m peso de 92kg ta 140/90 dolor a la palpación en hipocondrio enfermedad pulmonar. de las siguientes respuestas cual evita las
derecho. murphy (+) rebote positivo, que insicion se recomienda: complicaciones pulmonares postoperatorias:
- supra infraumbilical - ejercicios respiratorios minimo 48 horas antes de la cirugía
116.paciente politraumatizado mantenido con nutricion palenteral total
106.masculino de 28a quien fue sometido a apendicectomia con una formula equilibrada, pero que no es suficiente para cubrir sus
encontrando una apéndice cecal gangrenada. este antecedente nos requerimientos energéticos para incrementar el aporte energético se
permite clasificar la herida como: considera a:
- sucia - aumentar la fracción de glucosa de la fórmula
117.masculino de 28a proveniente de la uci donde fue sometido a una
107.masculino de 36a sometido a hernoplastia con reseccion de un laparotomía exploradora por haber sufridoaccidenteautomovilistico
diverticulo de meckel a travez del anillo herniario en el postoperatorio mencionándose que se le realizo reseccion intestinal de 40cm de
presento absceso en la herida quirúrgica el manejo de esta intestino delgado por presentar isquemia intestinal secundaria a
complicacion se realiza con drenaje y ciere: desgarro mesenterico por desaceleracion los receptores que estimulan
- diferido la liberación de corticotropa en este paciente se encuantran
108.masculino de 19a con apendicitis aguda no complicada cual es la localizados en:
incisión más conveniente para este paciente: - protuberancia y mensencefalo
- rocky davis 118.masculino de 35a el cual se encuentra en la uci con diagnostico de
pancreatitis aguda desde hace 10 dias actualmente el organismo
109.cual es la hormona que disminuye en la respuesta metabólica al
tiene:
trauma:
- necesidad de consumir cetonas
- insulina 119.ante un paciente que en el postoperatorio curso con desequilibrio
110.masculino de 53a politraumatizado por accidente automovilistico en
hídrico podemos intuir que el agua intracelular puede disminuir devido
esta paciente la respuesta metabólica al trauma provoca liberación de
a una de las siguientes alteraciones cual es esta:
adrenalina, la que provocara:
- reducción del potasio intracelular
- estimula la lipólisis
111.femenino de 32a quien refiere una masa blanda, no depresible, fija e 120.femenino de 25 años quien sufrio accidente automovilistico con
indolora y por arriba de la cicatriz umbilical. en este caso el defecto se lesiones leves, pero fue victima de una crisis emocional, se le toma
encuantra en la: gasometria revelando una pco2 de 30mmhg, tal cifra y el cuadro
- linea alba clinico sugieren:
112.durante una colecistectomia abierta accidentalmente se le caen las - acidosis respiratoria
tijeras. las cuales son imprescindibles para relizar la cirugía, necesitan 121.masculino de 35a quien es rescatado de accidente automovilistico y
que las tijeras se esterilicen que sustancia permite realizar este que presenta fracturas multiples y expuestas en miembro superior
procedimiento rapidamente: derecho ocacionando lesion arterial. al iniciarse como primer evento
- gluterhaldehido activado la vasoconstricción se activa el siguiente elemento:
113.masculino de 18a con herida producida por objeto cortante en - microagregado plaquetario
region frontal de la cara de 3cm aprox de longitud, que tecnica de
sutura usara para cerrar la piel y tener un resultado mas estetico:
- sutura subcuticular
24 de 28
AMBU (air-mask-bag unit), e identificar una saturación arterial de
ORDI oxígeno de 90%: ¿Qué manejo debe continuar este paciente?
1. Paciente masculino de 36 años el cual es portador de insuficiencia - Colocar puntas nasales
venosa de extremidades inferiores ¿cuál de las siguientes es una - Intubación endotraqueal con apoyo mecánico
medida antitrombótica? - Colocar mascarilla con reservorio
- Aspirina - Colocar cánula de Guedel
- Vitamina K - Traqueostomía de urgencias
- Anticoagulante
- Embolectomia 6. Una mujer de 78 años acude a urgencias por dolor en fosa iliaca
izquierda de 24 horas de evolución asociado a fiebre y algún vomito
2. Tipo de cirugía que se practica a los pacientes pediátricos que ocasional. Al exploración destaca dolor a palpación de forma
presentan hernia inguinal indirecta: selectiva en fosa iliaca izquierda con sensación de ocupación,
- Orquidopexia defensa y descompresión positiva. Ante la sospecha de diverticulitis
- Técnica extraperitoneal aguda, ¿cuál es la siguientes afirmaciones es correcta?
- Colocación de malla - En caso de diverticulitis aguda no complicada, está indicada la
- Plastia del piso del canal inguinal sigmoidectomía electiva tras la curación del primer episodio agudo
- Ligadura alta del saco herniario - En caso de precisar intervención quirúrgica tras solucionarse el
episodio agudo, el abordaje laparoscópico estrá contraindicado
3. Paciente femenina de 38 años de edad, fue operada de - Si se produjera una peritonitis generalizada, la técnica quirúrgica
colecistectomía por ser portadora de micro litiasis vesicular, más adecuada es la práctica de una colostomía derivariva sin
actualmente con dolor importante en el epigastrio con ictericia, el resección del segmento sigmoideo afectado
laboratorio reporta fosfatasa alcalina de 350 U ¿cuál patología - En caso de absceso pélvico contenido, está indicada la colocación
debemos de sospechar? de un drenaje percutáneo guiado con TAC o ecografía ♡
- Úlcera péptica - La exploración complementaria más segura y de mejor rendimiento
- Hepatitis es el enema con contraste bario
- Absceso residual
- Hematoma 7. Paciente masculino de 54 años de edad portador de estenosis pilórica
- Pancreatitis aguda por proceso cicatrizal de ulceración ¿qué procedimiento quirúrgico es
el adecuado?
4. Usted participa en la certificación de un hospital, dentro de las
- Antrectomía y gastroduodenoanastomosis
especificaciones que deberá tener un quirófano en base a la
- Piloroplastía y vagotomía trocular ♡
normativa de la Secretaría de salud, ésta deberá tener la siguiente
característica: - Dilatación de estenosis
- Una humedad del 50-60% ♡ - Piloromiotomía ♡
- Una presión negativa - Antrectomía y gastroyeyunoanastomosis
- Una temperatura de 22° al 24°
- Una area de 20 m2 4x5 M. 8. Masculino de 78 años, quien ingresa por abdomen agudo y choque
- Filtración de aire 15 veces por minuto séptico, probablemente debidos a enfermedad diverticular
complicada. El control de la vía aérea y la reanimación con líquidos
5. Paciente masculino de 62 años, post operado de hepatectomía del son prioritarios en la atención de este paciente antes de ser llevado a
lóbulo izquierdo. Ingresa extubado cuidados intensivos y 4 horas cirugía, para el control de la sepsis. Recordando que el objetivo es
después inicia con respiración agónica. Sus SV: TA: 110/70, FR: 30X’, FC recuperar y mantener la homeostasis; se encuentra canalizado sin
120 X’, T 36.5ºC. Su gasometría reporta PCO2 50 mmHg, Po2 45 mmHg y embargo necesitamos medir la PVC requerimos ¿realizar el siguiente
SaO2 de 78%. Después de aplicar O2 a través de presión positiva con procedimiento?
- Colocación del sello de agua
25 de 28
- Aplicar catéter en vena subclavia 13.Paciente postquirúrgico de colecistectomía laparoscópica, el cual es
- Vena de la fosa antecubital por punción las 24 horas posteriores a la cirugía presenta distensión abdominal, se
- Aplicar catéter en femoral por punción le solicita radiografía de abdomen encontrando aire libre
- Vena safena interna por venodisección subdiafragmático derecho, además de niveles hidroaéreos, exámenes
de laboratorio con Na-140 meq, K-4 meq, Cl-100 meq, ¿cuál sería su
9. Paciente femenino de 35 años de edad, obesa, quien será sometida a diagnóstico?
colecistectomía por presentar colecistitis crónica litiásica y además - Imagen compatible con sombra gástrica
tratamiento antirreflujo por hernia hiatal. ¿Cuál es la siguientes - Hematoma subdiafragmático
acciones preoperatorias es indispensable desde el punto de vista - Íleo postquirúrgico
médico-legal? - Bilioma
- Tele de tórax - Perforación de víscera hueca
- Expediente con laboratorios
- Electrocardiograma 14.Usted decide intubar a un paciente por presentar datos de
- Tricotomía en piso insuficiencia respiratoria, como se corrobora intubación exitosa?
- Firma de consentimiento informado -Observación de tubo en bronquios en RX
-Ruidos respiratorios en 4 puntos y SaO2 mayor de 80%
10.Paciente masculino de 60 años diabético mal control metabólico con -Paso del tubo por la hipofaringe
absceso perianales el cual es drenado se le informa que le quedará -Ruidos respiratorios en dos sitios del tórax
una secuela, mencione la más común: -Gasometría con CO2 de 45 mmHg y O2 de 70 mmHg
- Hemorroide externa
- Hemorroide interna 15.Paciente sometida a laparatomía exploradora por presentar
- Fistula embarazo tubárico roto. Al día siguiente del postoperatorio se detecta
- Polipo hematoma de la herida quirúrgica. ¿Cuál es la causa más frecuente
- Fisura de esta complicación?
- Deficiente función del tapón de fibrina ♡
11.Paciente femenina de 30 años portadora de micro litiasis vesicular,
- Anemia durante el embarazo
tiene dolor importante con amilasa de 1000 U/l fosfatasa de 500 U, se
- No se transfundieron plaquetas
observa tinte ictérico con evolución de 12 horas ¿cuál sería la acción
- Consumo de factores por sangrado
adecuada en estos momentos?
- inadecuada de hemostasia ♡
- CPRE + esfinterotomía
- Colecistectomia abierta y coledocotomía
- No realizar ninguna acción 16.Se trata de paciente de sexo femenino de 25 años, quien cursa con
- Colecistectomia laparoscópica estreñimiento severo, dolor intenso y sangrado escaso posterior al
- Colecistectomía abierta evacuación. Al examen clínico se aprecia desgarro lineal en el
anodermo. Lo más probable es que en este desgarro se localice en:
12.Paciente masculino de 50 años se le realizará hemicolectomía -Lateral izquierdo distal a la línea dentada
izquierda con anastomosis por un adenocarcinoma ¿cuál de los -Posterior proximal a la línea dentada
siguientes se considera factor intrínseco? -Lateral derecho distal a la línea dentada
- Falla renal -Anterior distal a la línea dentada
- Técnica quirúrgica -Posterior distal a la línea dentada
- Ventilación del quirófano ++ frecuente en linea media posterior, pero npi si es distal o proximal
- Instrumental
- Duración de lavado quirúrgico 17.A 35-year-old woman undergoes an elective laparoscopic
cholecystectomy for symptomatic cholelithiasis. Which of the following
wound classes best describes her procedure?
26 de 28
- Class III, Contaminated - Clase 5 (60%)
- None of the above - Clase 1 (15%)
- Class II, Clean/contaminated - Clase 3 (30-40%)
- Class I, Clean - Clase 4 (40%)
- Class IV, Dirty

18.A 25-year-old man is brought to the emergency room after sustaining 22.Masculino de 13 operado por apendicitis. Posterior presenta cicatriz
burns during a fire in his apartment. He has blistering and erythema of patológica
his face, left upper extremity and chest. He also has circumferential - Melanocítica
frank charring of his right upper extremity with decreased capillary refill. - Hematoma
He is agitated, hypotensive and tachycardic. Which of the following is - Úlcera
the most appropriate initial management of his wounds? - Seroma
- Topical antibiotics should be applied to the burn wounds - Hipertrófica
- Split-thickness skin grafts over the areas of third-degree burns.
- Excision of facial and hand burns. 23.Masculino portador de cirrosis hepática acude por hernia
- Escharotomy of the right upper extremity inguinoescrotal de varios años. Actualmente no se reduce y se
- Excision of all third-degree burns. encuentra abdomen distendido, si no se opera ¿Qué complicación
presentará ya que esté ocluido?
19.A 3-year-old boy is brought to the emergency room after spilling - Estrangulamiento y necrosis
bleach onto his lower extremities. He is diagnosed with a chemical burn - TEP
and all involved clothing are removed. In addition to resuscitation, - Shock hipovolémico
which of the following is the most appropriate initial management on - Trombosis venosa
this patient? - Incarceramiento
- Neutralize the burn wound with weak acids
- Treatment of the wound with calcium gluconate gel 24.Px que es operado por cuadro apendicular encontrándose apéndice
- Lavage of the burn wound with large volumes of water G4. ¿Cuál complicación se presenta si al 2 día se presenta salida de
- Wound debridement in the operating room material fecaloide por el drenaje?
- Treatment of the burn wound with antimicrobial agents - Absceso residual
- Dehiscencia de base apendicular
- Necrosis de ciego
ORDI
- Contaminación por anaerobios
20.Px portadora de pancreatitis edematosa con presencia de cálculo - Fístula
enclavado en colédoco a nivel ampular con importante dolor en 25.Masculino de 66 con cirrosis, presenta hemorragia de TDA por varices
epigastrio. ¿Qué acción debería realizarse antes de las 48 hrs para esofágicas. Sonda para controlar el sangrado
evitar el desarrollo de pancreatitis grave?
- Pezzer
- Aplicar antibióticos - Murphy
- Ayuno - Foley
- SNG - Nelaton
- Esfinterotomía de Oddi x CPRE - Sengstaken-Blakemore
- Lavado peritoneal
26. Femenino de 67 con neumopatía crónica obstructiva de 20 años.
21.Femenino de 45 pesa 60 kg, sometida a histerectomía bajo bloqueo
Parámetro a disminuir para los requerimientos calóricos
epidural por miomatosis uterina. Perdió 2.5 Lt de sangre. De acuerdo
con el colegio americano de Cx, ¿clase de hemorragia? DUDA - CHOS
- Clase 2 (15-30%) - aas
- Proteína
27 de 28
banco
PRIMER

1ER PARCIAL

1. Masculino de 35 con herida penetrante en región poplítea. EF: TA 90/50, 125 lpm, T
36, 20 rpm. No hay sangrado activo. ¿Componente reconocido que se activa a la
exposición de colágeno y previene el sangrado? “hemostasia primaria”

R: Fosfolípido // Plasminógeno // Fibrina // Plaquetas // Fibrinopéptido

2. Paciente con perforación de sigmoides de 8 hrs de evolución. Si se aplican antibióticos


pre Cx. Concepto correcto

R: Es profilaxis // Requiere de antibióticos // No se requiere antibióticos // Es indispensable


para profilaxis cultivo // La herida Qx es clase 1

3. Masculino de 31 con contusión abdominal. TA 90/50, 145 lpm, 24 rpm, T 35.5. Piel
muestra cambios clásicos de choque. Enunciado que describe las características de la
piel del px

R: Piel fría, húmeda, pálida sin cianosis // Rubincunada, rojo cereza, caliente // Fría, seca,
eutérmica sin cianosis // Fría, palidez, húmeda, pegajosa // caliente, sudorosa, pegajosa con
acrocianosis

4. Masculino de 35 pesa 70. Padece pancreatitis necrótica, tiene 3 días de evolución. Sin
fiebre, tiene sed, TA 100/70, 110 lpm, 24 rpm, diuresis horaria 20 ml/hr. ¿Dónde se
encuentra el líquido faltante?

R: Espacio transcelular // Tercer espacio secuestro // 2 espacio // 1er espacio // Espacio


intracelular

5. Indicación RELATIVA para un abordaje transtorácico para la funduplicatura de Nissen

R: Hernia paraesofágica // Esplenectomía previa // Hernia Hiatal 1 // Reparación previa de


hernia hiatal // Hernia hiatal grande

6. Px de 55 con rectorragia de 3 meses. Estudio para su Dx

R: Eco // TAC // Rectosigmoidoscopia // Exploración anal // Colonoscopía

7. Femenino de 58, con DM, presenta datos de insuficiencia venosa periférica sometida a
colecistectomía por piocolecisto. Medida para evitar TEP

R: O2 y heparina // Nebulizaciones // Ejercicios resp pre Cx //Enoxaparina y medidas de


compresión // Antibióticos y enoxaparina
8. Masculino de 65 años sometido a hemicolectomía izquierda por Adenocarcinoma de
colon, con anastomosis primaria. Para continuar con vigilancia debe de tener
seguimiento. Marcador indicado

R: AP // ACE // CA 125 // AFP // GC

9. Femenina de 18 años sometida a Cx por apendicitis. Al 8 día postQx, presenta fiebre.


Posible causa

R: IVU // Absceso residual // Atelectasia // TEP // Venoclisis

10. Citocina proinflamatoria, activador primario de la reacción febril en un px sometido a


laparotomía por perforación de íleon

R: IL 1 // IL 6 // TNF // IL 10 // IL 2

11. Femenino de 68, DM de 18 años, post operada por perforación de colon secundaria a
colon tóxico amebiano. Al 4 día post requirió asistencia ventilatoria, al 15 hemodiálisis,
también se ha tenido que transfundir en 2 ocasiones por sangrado de TD al 5 día. Hoy
se reporta coluria. Cultivos negativos. ¿Qué evento tisular es causa de su estado actual?

R: Pérdida de la capacidad de defensa tisular // Hemorragias petequiales en distintos órganos


// Pérdida de la capacidad de cicatrización de epitelios // Edema y secuestro de líquidos que
dificultan la oxigenación // Infiltración por MCF en órganos

12. Femenino de 35 en UCI con Dx de drenaje absceso pélvico. Urea de 240, creatinina 5.
La gasometría reporta hipoxemia e hipercapnia, usa asitencia mecánica, presentando
mejoría. TA 130/80, 110 lpm, 25 rpm, T 38. Complicación

R: CID // Falla orgánica múltiple // Choque descompensado // Peritonitis pélvica //


Insuficiencia respiratoria

13. Masculino de 43 sufre fractura de ambos fémur. Evaluación inicial alteración de alerta,
piel fría húmeda y marmórea. TA 80/40, 120 lpm, 28 rpm, T 35.6. Fisiopatología

R: Disminución de perfusión en cerebro y corazón // Apertura de esfínteres precapilares y de


vasos de capacitancia // Hipoperfusión tisular sistémica // Incremento de hormona reguladora
// Acúmulo de Ac volátil y desviación álcali del bicarbonato

14. Masculino de 43 sufre fractura de fémur y pelvis. Evaluación inicial alteración de alerta,
piel fría húmeda y marmórea. TA 80/40, 120 lpm, 28 rpm, T 35.6. Principal
determinante en la sobrevida del px
R: Tiempo de hipoperfusión // Manejo temprano de falla orgánica múltiple // Demora en aporte
nutricional // Control temprano de sus Fx // Combinación de ayuno e hipotermia

15. A 45 woman with a history of heavy AINE ingestion. She undergoes exploratory
laparotomy 30 hours after onset of symptoms and is found () duodenal ulcer. Tx de
elección

R: Truncal vagotomy and antrectomy // Simple closure with omental patch // Hemigastrectomy
// Truncal vagotomy and pyloroplasty // High selective vagotomy with omental patch

16. Masculino de 34 con estreñimiento crónico de 5 años. Tiene 2 días con dolor intenso al
evacuar, así como sangrado rectal en goteo post evacuación, dolor al sentarse. Hb de
12, Hto 38, Leucos 6000, TP 12, TPT 34. Tx Qx

R: Cicatrización de herida // Fisurectomía + esfinterotomía lateral // Drenaje a cielo abierto //


Fistulectomía // Hemorroidectomía

17. Características de hemorroide interna

R: Ser visibles en el exterior del ano // Provienen del plexo hemorroidal inferior // Localizadas
proximalmente en relación con línea dentada // Estar cubiertas por epitelio escamoso //
Localización distal en relación línea dentada

18. Femenino de 23 presenta dolor abdominal de 48 h periumbilical que irradia a FID.


Signos apendiculares + . Leucos 16,000. Refiere mejoría. ¿Cual es la exploración más
probable al estado actual?

R: EPI // Colitis // Perforación apendicular // Torsión ovárica // Embarazo ectópico

19. Mujer de 36 politrauma, sometida a laparotomía exploradora por hemoperitoneo. Al 2


día post manifiesta IRA. Define complicación

R: Sobrecarga de volumen // Eritrocituria // Azoemia // Hipercalcemia // Piuria

20. Masculino de 56 con sangrado rectal de 6 semanas, sin pérdida de peso. Estudio Dx

R: TAC // Colonoscopia // Colon por enema doble contraste // RM con gadolinio // Rx simple

21. Masculino de 9 años de edad traído por dolor abdominal de 18 horas en mesogastrio
migra cuadrante inferior derecho con anorexia febrícula disuria sin tratamiento la
exploración física reactivo bien hidratada cardiopulmonar sin compromiso, abdomen
blando peristalsis disminuida dolor a la palpación superficial y profunda en cuadrante
inferior derecho signos apendiculares positivos llenado capilar inmediato BH 13,300
leucos bandas al 5% eco normal y en la escala de Alvarado ¿ que puntuación presenta
R: 2// 7// 8// 6// 5 //

22. En el área de hospitalización de cirugía hay dos pacientes con antecedentes de


transfusión y reanimación con líquidos por diversos grados de choque hipovolémico
cada paciente muestra un aumento de diuresis llegando a 100 ml/hr todos niegan sed ,
signos viales estables por lo anterior los médicos restringen el aporte parenteral de
cristaloides ¿porque debe de restringir el aporte de líquidos en estos pacientes?

R: Durante la redistribución de líquidos aumenta el flujo intravascular de retorno // la


hiperglucemia de la respuesta el trauma aumenta la poliuria // no es necesario aportar el
requerimiento basal de glucosa // falla prerrenal puede mejorarse con esta medida// la pérdida
de sodio del espacio intracelular puede disminuirse

23. El estado crónico de la absceso perianales se le conoce como?

R:síndrome de hemorroidal // fístula // incontinencia anal //fisura // absceso perianal

24. Masculino de 73 con ingesta de múltiples antiinflamatorios que presentó dolor


abdominal de 36h, náusea, fiebre, T de 38.7, abdomen en madera, leucos 22,500,
Neutrófilos 87%, 6% bandas. Estudio de gabinete de 1ra elección

R: USG abdominal // Rx simple de abdomen y tórax // TAC // Gammagrafía con tecnecio 99


// Endoscopia TDA

25. Antibiótico que alcanza concentraciones inhibitorias en abscesos abdominales

R: Penicilina G sódica cristalina // Tetraciclina // Ceftriaxona // Metronidazol // Gentamicina

26. Femenino de 23 acude por dolor abdominal de 48 h en CID, súbito. Se acompaña de


náuseas y vómito alimenticio, fiebre, disuria, descarga vaginal. Presenta menstruación.
EF: fascies de dolor, diaforesis, taquicardia, deshidratada, dolor a palpación, dolor
contralateral al palpar FII, dolor al pellizcar cresta ilíaca derecha, Leucos 22,500, 25%
bandas. Apendicitis aguda. Mortalidad de de apendicitis aguda perforada

R: 0 / 10 / 50 / 1.7 / 0.3 %

27. Femenina de 38 años que sufre accidente automovilístico choque frontal hace cuatro
horas abdomen distendido dolor difuso ruidos peristáltico los disminuidos frecuencia
cardiaca de 105 lpm TA 90/ 50 mmHg hematocrito 32% hemoglobina de 10 mg/dl el
mejor parámetro para evaluar de manera indirecta la perfusión tisular

R: presión venosa central// llenado capilar // oximetría de pulso // tensión arterial // gasto
urinario
EXAMEN B4

1. Masculino de 78 con antecedentes de tos crónica de 5 años Tx con loratadina, 4-5


tequilas semanales x 20 años. Hace 4 meses presenta disfagia a sólidos, halitosis,
regurgitación y aumento de volumen postprandial inmediato del cuello. EF pérdida
ponderal de 10 kg, lengua saburral. Hb 11.2. Dx

R: ERGE // Fístula traqueoesofágica // Divertículo de Zenker // Acalasia esofágica // Espasmo


difuso del esofágo

2. Masculino de 56 con sangrado rectal de 6 semanas, sin pérdida de peso. Estudio Dx

R: TAC // Colonoscopia // Colon por enema doble contraste // RM con gadolinio // Rx simple

3. After a removal of a sessile polyp of 2x1 cm found 1 finger lenght above the anal
mucocutaneous margin, the pathologist reports it tot have been a vilous adenoma that
contained carcinoma in situ. Next step in management

R: External RT to the rectum (NO) // Abdominoperineal rectosigmoid resection // No further


therapy // Reexcision of the biopsy site with wider margins // Anterior resection of the rectum

4. Primeros síntomas claros de Ca colorrectal

R: Cambio en defecaciones y hemorragia rectal // Hemorragia rectal y dolor abdominal // Masa


en CID y dolor // Meteorismo y dispepsia // Pérdida de peso

5. A 42 year old man has bouts of intermittent crampy abdominal pain and rectal bleeding.
Colonoscopy is performed and demonstrates multiple hamartomatous polyps. They
removed as many polyps as possible. Dx

R: Chron // Familial polyposis // Villous adenomas // CUCI // Peutz-Jeghers Sx

6. Trastorno asociado a potasio bajo en plasma, como consecuencia de la amortiguación


celular

R: Ac R // Ac M // Alcalosis R // Alcalosis M // Todas las anteriores


7. An 80 year old man with a history of symptomatic cholelithiasis presents small-bowel
obstruction. Which of the following findings would provide the most help in
ascertaining the Dx.

R: PH 7.5, PCO2 50, paradoxically acid urine // Palpable mass in the pelvis // Coffee grounds
aspirate from the stomach // Leucos 40,000 / Pneumobilia

8. Respecto a la distribución de líquidos, en un M de 72 kg las siguientes aseveraciones


son verdaderas EXCEPTO

R: 3-3.5 L corresponden a plasma // Agua extracelular es de 14 L // Agua extracelular es 40 L


// Agua corporal total comprende 55-60% de masa corporal total // Agua intracelular es de 22
L

9. Esteroide más abundante y de mayor importancia fisiológica

R: Cortisol // Prednisona // Aldosterona // Progesterona // Corticoesterona

10. Presencia de precipitaciones (litos) en vesícula

R: Colecistitis // Hidropesia vesicular // Colangitis // Colelitiasis // Coledocolitiasis

11. La hiperglucemia en el enfermo crítico se debe a:

R: Deterioro de función de la somatostatina // Ingesta reciente de alimento // Gluconeogénesis


// Modificación de receptores de insulina // DM

12. Mujer de 65 con DM, diarrea leve. Refiere sed. TA 150/70, 90 lpm, 16 rpm, T 37. Mejor
vía para reponer pérdidas

R: Nasogástrica // Nasoyeyunal // Cateter periférico // VO // Cateter central

13. DM normal del conducto colédoco en USG

R: 6-7 mm // 1-2 mm // 13-15 mm // 3-4 mm // 9-10 mm

14. Femenino de 39 años con antecedentes de funduplicatura laparoscópica hace cuatro


días presenta en las últimas ocho horas hipoperfusión tisular marcada choque
hipovolémico grave FR de 132 disminución del retorno venoso y hipercarbia
incremento del pico inspiratorio del ventilador y la presión intravesical de 38 mmHg el
diagnóstico más probable es:

R: choque séptico // síndrome de compartimiento abdominal // sangrado post operatorio //


síndrome de isquemia aguda mesentérica // síndrome de respuesta inflamatoria sistémica(NO)
15. Mujer de 65 hipertensa de 20 años con 160/100 al momento de visita. Programada para
colecistectomía. Riesgo Qx

R: Clase 1 / 2 / 3 / 4 / 5

16. Evalúa a un px post operado por ileostomía. El gasto fue de 2.5 L/24 h. Refiere sed
intensa, astenia y adinamia, desubicado en tiempo y espacio, vol urinario 15/h. Glucosa
120, urea 80, creatinina 1.8, Na 127, Cl 85, K 2,. TA 90/50, 125 lpm, 22 rpm, T 37.
Trastorno hidroeléctrico

R:Distribución e intercambio // Obstructivo y de formación de 3 espacio // Control renal de


electrolitos // T de pH y osmolaridad // T de volumen, concentración y composición

17. Paciente con cansancio, dolor, edema vespertino y limitación de los arcos de
movimiento con tortuosidades de ambas extremidades inferiores, así como
hiperpigmentación. ¿Qué signo corresponde a una afección grave?

R: Dolor // Hiperpigmentación // Limitación del arco de movimiento // Edema // Venas


tortuosas

18. Hombre de 52 años a quien se le realizó colecistectomía abierta por colecistitis aguda
litiásica no complicada al salir del quirófano TA 130/80 FC 75 FR 16 temperatura de
36.5 °C en reposo prolongado se considera factor de riesgo por cualquier causa en todos
los pacientes ¿ cuándo es conveniente indicar la deambulación a la paciente?

R: 8 días después // 6 días después // cuando la paciente lo pida // al día siguiente de la


intervención // 2hrs después

19. El equilibrio hidroeléctrico es esencial para la pronto recuperación. El proceso


inflamatorio que impone la enfermedad y el manejo quirúrgico influyen directamente
en la movilización de líquidos corporales. Cuando estas pérdidas por distribución son
importantes, el paciente puede desarrollar hipovolemia. ¿Cuál sería el trastorno de
líquidos más común de este problema hipovolémico?

R: Déficit de líquido extracelular // Pleuresía por trasudado // Infiltrado acuoso // Intersticio


patológico // Ascitis

20. Las siguientes aseveraciones son ciertas con respecto a la fase EBB de la respuesta
metabólica al trauma, EXCEPTO
R: Ocurre inmediatamente posterior a la lesión // Duración de 12-24 hrs // Reducción de presión
sanguínea, GC, temperatura // Existe un estado hipermetabólico // Se asocia a hemorragia

21. Mediador más temprano y potente en la respuesta a una lesión aguda o infección es:
R: IFN-gamma // IL-1 // TNF-a2 // Histamina // Serotonina

22. Masculino de 75 con DM, sedentarismo, tabaquismo. Acude por hematoquecia de 1


semana, además de periodos de diarrea y ocasional estreñimiento, acompañados de
dolor difuso. Dx (cambios en defecación y hematoquecia=CA colon)

R: Amibiasis invasora // Ca colon // Apendicitis complicada // Diverticulitis complicada //


Trombosis arterial mesentérica

23. Mujer de 23 años, cursa embarazo y tiene colelitiasis, la cual ha condicionado estados
frecuentes de cólico biliar. Acude para su progresión quirúrgica. ¿Cuándo se
recomienda hacer la colecistectomía?

R: Primeros 2m del embarazo // 2 trimestre // 1 trimestre // 3 trimestre // Últimos 2m del


embarazo

24. Masculino de 23, cursa el 6 día post por resección intestinal secundaria a perforación
traumática, periodo durante el cual ha permanecido en ayuno. Ayer comenzó a fugar
líquido intestinal en abundante cantidad y se desarrolló Sx de respuesta inflamatoria
sistémica. TA 110/70, 95 lpm, 16 rpm, T 39. ¿Qué datos específicos debe considerar
para clasificar el grado de deshidratación?

R: Sed, signos ortostáticos, signos de hipoperfusión // sed e hipertermia asociada a SIRS // Sed,
pérdidas aumentadas y signos de perfusión // Sed, pérdidas aumentadas y tipo de material
fugado // Análisis clínico, laboratorio, cálculo y prescripción

25. A la exploración física de un paciente usted encuentra triada de Charcot ¿por lo que su
razonamiento clínico es?

R: colangitis // hidrocolecisto //colelitiasis //colecistitis // coledocolitiasis

26. Paciente con posible litiasis vesicular, 1er estudio a realizar

R: TAC // Colangio percutánea // Eco // RM // CPRE

27. Paciente de 72 años el cual acude por claudicación intermitente de la extremidad


inferior izquierda de menos de 20 m exploración física phIegnacia Alba dolens + pulsos
poplíteo y pedio disminuidos de extremidad inferior izquierda con este diagnóstico
usted manda realizar el estudio considerado estándar de oro el cual es?

R: angiografía RM contraste // angiografía diagnóstica con medio de contraste // ecografía en


modo B // ecografía Doppler // angiografía por tomografía en medio de contraste
28. Principal catión en el cuerpo y que desempeña un papel esencial en muchos procesos
metabólicos y fisiológicos

R: Ca // K // Mg // Fósforo // Na

29. ***Paciente hombre con sintomatología esofágica y extra esofágica antecedente


ERGE. Se presenta por dolor abdominal agudo, intenso en epigastrio con irradiación
generalizada, rebote (+), neutrofilia, por lo que se integra abdomen agudo
intraperitoneal Qx. Se decide realizar laparoscopia diagnóstica pero usted recuerda que
el procedimiento de la laparoscopia se considera estándar de oro para?

R: esofagectomía // colecistectomía //apendicectomías // gastrectomía funduplicatura

30. Trastorno ácido base causado por hipoventilación que conduce a retención de CO2

R: Alcalosis R // Alcalosis R + Ac R // Ac M // Ac M // Alcalosis M // Ac R

31. En el área de hospitalización de cirugía hay dos pacientes con antecedentes de


transfusión y reanimación con líquidos por diversos grados de choque hipovolémico
hacia el quinto día los electrolitos séricos son normales cada paciente muestra un
aumento de diuresis llegando hasta los 100 ml todos niegan ser los signos vitales son
normales estables por lo anterior los médicos restringen el aporte parenteral de
cristaloides ¿por qué se debe restringir el aporte de líquidos en estos pacientes?

R= durante la redistribución de líquidos aumenta el flujo intravascular // hiperglucemia de la


respuesta al trauma aumenta la poliuria// no es necesario aportar el requerimiento basal de la
glucosa // falla perirrenal puede mejorarse con esta medida // pérdida de Na del espacio
intracelular puede disminuirse

32. Mujer de 48 años a quien se le realizó resección pancreatoduodenal con técnica de


whipple por Ca de cabeza de páncreas. Cursa el 2 días post y debido a una reposición
insuficiente de líquidos está mal. Dato bioquímico que se relaciona con el déficit hídrico
de esta paciente

R: Hiperazoemia // Hipocalcemia // Isonatremia // Hipoalbuminemia // Hipoglucemia

33. Masculino de 57 con HAS, DM2. Hace 9 años tuvo colecistectomía y funduplicatura.
Tiene 3 meses de cambios intestinales, diarrea, evacuaciones con moco y sangre fresca
escasa, tenesmo rectal, sensación de distensión abdominal, meteorismo, aerofagia,
peristalsis aumentada, pérdida de 4 kg en 6 meses. Abdomen blando y depresible,
peristalsis normal, Tacto rectal con masa tumoral fungosa a 7 cm del margen anal de
aproximadamente 6 cm de dm, que ocluye parcialmente la luz intestinal. 1er estudio a
realizar
R: USG rectal // TAC de abdomen // RM // Colonoscopia con biopsia // Marcador tumoral

PARCIAL B5

1. Px donde NO es prioridad establecer protocolo de nutrición parenteral de forma


inmediata

R: Hipertenso // Diabético // Politraumatizado // Oligúrico // Pneumopata

2. Masculino de 22 que se cayó. Se realizó hepatectomía parcial y resección de 30 cm de


ID, recibió 15 U de paquete globular en el pre y transoperatorio, 5 lt de Hartmann y 4
unidades de plasma fresco. Durante el cierre de laparotomía, inicia con sangrado difuso
de la cavidad abdominal (en capa). Causa

R: I hepática // Plaquetopenia // Hiperesplenismo // Hipoprotrombinemia // Déficit Factor 9

3. Durante la reanimación de un paciente con herida penetrante de abdomen por proyectil,


presenta shock hipovolémico. Sol adecuada

R: Haemaccel // Plasma fresco // Hartman // NaCl 0.9% // Dextran 40

4. Px femenino de 45 años sometida a histerectomía por sangrado persistente. Se


encuentra necrosis de miomas. Recibió transfusión de 5U de sangre y 5 de plasma.
Aparecen datos de CID. ¿Qué dato apoya el Dx?

R: Disminución de F hageman // Presencia de reticulocitos // Presencia de productos de lisis


fibrinógeno // Antitrombina 3 disminuida // Crenocitos abundante en la orina

5. Se le pide que coloque sonda para preparación gástrica el cual será sometido a
endoscopia x datos de retención gástrica condicionando un mega estómago. Mejor
sonda por características y duración

R: Rush // Nelaton // Robinson // Foley

6. En relación a la causa de presentación de fiebre en Cx en el post ¿cuáles son las más


frecuentes?

R: Infección de herida Qx y venopunción // Infección de herida Qx, IVU, neumonías //


Neumonía y bronquitis // Catéter de sonda urinaria y neumonía // Tromboflebitis, atelectasia
7. Femenino de 44 con Dx de úlcera péptica programado para realizar endoscopia. ¿Cuál
debe preceder al estudio?

R: Px en decúbito ventral // Sonda vesical // Verifica que se encuentre en ayuno de 8 h //


Aplicación de enema evacuante // Colocación de sonda nasogástrica

8. Femenina de 38 años operada por colecistectomía por ser portadora de micro litiasis
vesicular. Actualmente con dolor importante en epigastrio con ictericia, fosfatasa
alcalina de 350 U. Patología a sospechar

R: Pancreatitis aguda // Absceso residual // Hepatitis // Úlcera péptica // Hematoma

9. Femenina de 49 ingresó a UCI por shock séptico secundario a perforación intestinal; se


le han aportado volúmenes adecuados de Ringer y vasopresores. Su manejo ha sido
guiado por la PVC y diuresis. Se le agrega dificultad respiratoria, ictericia, confusión
mental. Complicación

R: TVC // Hepatitis reactiva // Sepsis severa // Shock séptico refractario // SIRS

10. Masculino de 53 con adenocarcinoma de colon sigmoides con B1 dukes modificada.


Tx

R: Ileostomía + QT // Hemicolectomía izquierda con anastomosis primaria // RT // Colostomía


definitiva // Hemicolectomía + resección de ganglios

11. An 80 years old man with a history of cholelithiais presents SyS of a small bowel
obstruction. Which of the following findings would provide the most help in
ascertaining the Dx?

R: Leucos 40,000 // Coffee ground aspirate from the stomach // Pneumobilia // A palpable mass
in the pelvis // PH 7.5, PCO2 50, paradoxically acid urine

12. Masculino de 52, alcohólico crónico presenta gran distensión abdominal con anasarca
y dificultad respiratoria. PH 7.44, PaCO2 28, PaO 80. Tx ácido base

R: Ac M // Al M // Al R compensada // Ac R // AL mixta

13. Ingresan 2 varones de 28 y 25 sufrieron volcadura. Cursan 2 días de estancia en UCI.


Actualmente están en ayuno y sin necesidad de vasopresores, con diuresis adecuada.
Sin ser DM, tienen hiperglucemia. Sustancia que promueve gluconeogénesis

R: Hormona folículo estimulante // Cortisol // Grelina y leptina // LH // Insulina

14. Se le inicia ayuno previo Cx. El ayuno es una agresión al organismo, sin embargo, debe
prescribirse esta indicación antes de todo evento invasivo. ¿Por qué es necesaria esta
prescripción?
R: Induce adaptación temprana al ayuno postoperatorio // Elimina riesgo de estimulación vagal
// Disminuye riesgo de broncoaspiración // Mantiene la vía aérea permeable // Que el paciente
vomite durante la Cx

15. En su presencia de íleo por un cálculo biliar, sitio mas afectado (Pregunta
autocompletada)

R: Válvula Ileocecal o Íleo distal

16. Px de 46 con dolor abdominal en CSD, debe insistirse en la descripción del dolor para
el Dx. Enunciado que concuerde con el cuadro clínico

R: Si el dolor se irradia a hombro izquerdo es colecistitis // Si el dolor es postprandial e


irradiado a hombro derecho es colecistitis // Si el dolor disminuye con el vómito es pancreatitis
// Si el dolor se inicia después de la ingesta de alimentos se trata de úlcera perforada // Si el
dolor se irradia a la región inguinal es cálculo ureteral.

17. En presencia de íleo por un cálculo biliar, la mayoría de las veces la obstrucción se
hallará en:

R: Sigmoides // Íleon proximal // Duodeno // íleon terminal // Yeyuno

18. Femenino de 53 pesa 70 kg. Está en shock hipovolémico por diarrea coleriforme. Se ha
iniciado fluidoterapia y para vigilar su respuesta se ha colocado una sonda urinaria. Para
considerar que el riñón tenga una perfusión sanguínea adecuada ¿Cuánto debe tener de
gasto urinario?

R: Sacar gasto urinario (0.5-1) x (Peso del paciente) à 35-70 ml

Masculino de 58 con EPOC, tuvo piocolecisto perforado. Fue extubado y su SaO2 de 85%.
150/70 mmHg, 110 lpm, 18 rpm, T 36, PH 7.33, HCO3 18, Na 135, Cl 110, PCO2 48. Trastorno
ácido base

R: Ac Mixta NO compensada // Alcalosis R NO compensada // Ac Met compensada // Ac Met


con AG elevado // Al Met

19. A 45 woman with a history of heavy AINE ingestion. She undergoes exploratory
laparotomy 30 hours after onset of symptoms and is found () duodenal ulcer. Tx de
elección

R: Truncal vagotomy and antrectomy // Simple closure with omental patch // Hemigastrectomy
// Truncal vagotomy and pyloroplasty // High selective vagotomy with omental patch

20. Femenina de 62 años a quien se le realizó resección intestinal con entero-entero


anastomosis termino-terminal por isquemia intestinal. Se ha manejado con triple
antibiótico. ¿Qué grupos generales de bacterias debe cubrir dicho esquema?
R: Gram (-), bacilares y cocoides // Gram (-) y (+), esporas // Gram (+ y - ) y anaerobias //
Gram (+) bacilares y cocoides // Bacteroides, fusobacterium y peptostreptococcus

21. Femenino de 58 post operada por colecistectomía. La intervención duró 4 hrs, cursando
con hipotensión postoperatoria asociado a sangrado y medicamentos. Se dieron sólo
500 ml de Hartmann y 2 hrs después solo ha orinado 50 ml. Fisiopatología LAS 3
ESTÁN BIEN

R: Respuesta fisiológico a la anemia // Respuesta fisiológica por vasopresina // Obstrucción de


la sonda urinaria // Respuesta fisiológica por aldosterona // Respuesta fisiológica por falta de
líquidos

22. Masculino de 23, cursa el 6 día post por resección intestinal secundaria a perforación
traumática, periodo durante el cual ha permanecido en ayuno. Ayer comenzó a fugar
líquido intestinal en abundante cantidad y se desarrolló Sx de respuesta inflamatoria
sistémica. TA 110/70, 95 lpm, 16 rpm, T 39. ¿Qué datos específicos debe considerar
para clasificar el grado de deshidratación?

R: Sed, signos ortostáticos, signos de hipoperfusión // sed e hipertermia asociada a SIRS // Sed,
pérdidas aumentadas y signos de perfusión // Sed, pérdidas aumentadas y tipo de material
fugado // Análisis clínico, laboratorio, cálculo y prescripción

23. Masculino de 39 años con antecedente de herida penetrante de abdomen por arma
blanca. Transoperatorio se encuentra hemoperitoneo de 2 lt, su piel con signos de
hipoperfusión, mientras la piel y vísceras intraabdominales son sacrificadas,
contrariamente corazón, cerebro y músculo se mantienen perfundidos. Explicación
fisiológica

R: Cierre selectivo de esfínteres precapilares aumento de contractilidad cardiaca

24. ¿Quién valora la vía intrínseca de la coagulación?

R: Tiempo de trombina // T de sangrado // T de protrombina // T parcial de tromboplastina //


Fibrinógeno

25. Masculino de 68 sometido a resección abdominoperineal por cáncer de recto, pérdidas


de volumen a volumen. En el post se manejó con glucosada 5% presentando debilidad,
letargia, convulsiones, coma. Problema electrolítico

R: Hiponatremia // Hiperglucemia // Hiper Ca // Hiper K //Hipo K

26. Antibiótico profiláctico en operaciones de de I grueso

R: Clindamicina // Metronidazol // Cefotaxima // Quinolona // Amikacina


27. Mujer de 36 politrauma, sometida a laparotomía exploradora por hemoperitoneo. Al 2
día post manifiesta IRA. Define complicación

R: Sobrecarga de volumen // Eritrocituria // Azoemia // Hipercalcemia // Piuria

28. Masculino de 43 sufre fractura de ambos fémur. Evaluación inicial alteración de alerta,
piel fría húmeda y marmórea. TA 80/40, 120 lpm, 28 rpm, T 35.6. Fisiopatología

R: Disminución de perfusión en cerebro y corazón // Apertura de esfínteres precapilares y de


vasos de capacitancia // Hipoperfusión tisular sistémica // Incremento de hormona reguladora
// Acúmulo de Ac volátil y desviación álcali del bicarbonato

29. Masculino de 20 a quien se le practicó laparotomía por apendicitis perforada. Al 5 dí


post presenta fiebre, venas del cuello colapsados, 90/50 TA, 130 lpm, 32 rpm, t 39,
llenado capilar 4 s y pulsos débiles, diuresis en 8 hrs de 100 ml. Dx (SRIS)

R: Atelectasia // Infarto superior // Shock séptico // Embolia pulmonar // Tamponade

30. Estudio indispensable para evaluación cardio preoperatoria en feminio de 48 con HAS
mal controlada

R: Monitoreo holter // Biomarcdores // Eco bajo estrés con dobutamina // Rx tórax y EKG //
Prueba de esfuerzo en banda sin fin y ekg

*ORDINARIO B4

1. Masculino de 62 posoperado de hepatectomía de L izquierdo. Ingresa extubado a UCI


y 4 hrs después inicia con respiración agónica. 110/70 mmHg, 30 rpm, 120 lpm, T 36.5,
PCO2 50 mmHg, PO2 45 mmHg, SaO2 78%. Después de dar O2 x presión positiva
con AMBU e identificar SatO2 90%, ¿Qué manejo debe continuar?

R: Cánula de Guedel // Mascarilla con reservorio // Traqueotomía de urgencia // Intubación //


Puntas nasales

2. Masculino de 60 DM mal controlado con absceso perianal el cual es drenado y se le


informa que le quedará una secuela. Más común

R: Fístula // Pólipo // Hemorroide interna // Hemorroide externa // Fisura


3. Px portadora de pancreatitis edematosa con presencia de cálculo enclavado en colédoco
a nivel ampular con importante dolor en epigastrio. ¿Qué acción debería realizarse antes
de las 48 hrs para evitar el desarrollo de pancreatitis grave?

R: Aplicar antibióticos // Ayuno // SNG // Esfinterotomía de Oddi x CPRE // Lavado peritoneal

4. Masculino el cual tiene varias horas de presentar cuadro de oclusión intestinal baja
actualmente con vómitos fecaloides, ¿Cuál es el mecanismo relacionado con las
características de los vómitos?

R: Se relaciona con peristalsis // La materia fecal tiene movimiento retrógrado // Antiperistalsis


// La oclusión es funcional // La colonización del ID

5. Femenino de 45 pesa 60 kg, sometida a histerectomía bajo bloqueo epidural por


miomatosis uterina. Perdió 2.5 Lt de sangre. De acuerdo con el colegio americano de
Cx, ¿clase de hemorragia?

R: Clase 2 (15-30%) // Clase 5 (60%) // Clase 1 (15%) // Clase 3 (30-40%) // Clase 4 (40%)

6. A 45 woman with a history of heavy ingestion of AINE’S presents with acute


abdominal pain. She undergoes exploratory laparotomy 30 hrs after onset of symptoms
and is found to have a perforated duodenal ulcer. Tx

R: Simple closure with omental patch // Highly selective vagotomy with omental patch //
Truncal vagotomy and antrectomy // Truncal vagotomy and pyloroplasty // Hemigastrectomy

7. Mujer de 78 acude por dolor en FII de 24 hrs asociado a fiebre y algún vómito ocasional.
EF: dolor a palpación selectiva en FII con sensación de ocupación, defensa y
descompresión (+). Ante la sospecha de diverticulitis aguda, Afirmación correcta

R: En caso de DA no complicada, está indicada la sigmoidectomía electiva tras la curación del


1er episodio // En caso de precisar Cx tras solucionarse el episodio agudo, la laparoscopia es
CI // Si se produjera una peritonitis generalizada, el Tx es colostomía derivativa sin resección
del segmento sigmoideo afectado // En caso de absceso pélvico contenido, está indicada la
colocación de un drenaje percutáneo guiado por TAC o Eco // La exploración complementaria
más segura y de mejor rendimiento es el enema con bario

8. Masculino de 13 operado por apendicitis. Posterior presenta cicatriz patológica

R: Melanocítica // Hematoma // Úlcera // Seroma // Hipertrófica*

9. Femenino de 35, obesa, será sometida a colecistectomía por colecistitis crónica litiásica
y además Tx antirreflujo por hernia hiatal. Acción pre Cx es indispensable desde el
punto médico-legal
R: Expediente con lab // EKG // Tele de tórax // Tricotomía en piso // Firma de consentimiento
informado

10. Masculino portador de cirrosis, acude por hernia inguinoescrotal de varios años.
Actualmente no se reduce y se encuentra abdomen distendido, si no se opera, ¿Qué
complicación presentará ya que esté ocluido?

R: Estrangulamiento y necrosis // TEP // Shock hipovolémico // Trombosis venosa //


Incarceramiento

11. Tipo de Cx que se practica en Px pediátricos con hernia inguinal indirecta

R: Extraperitoneal // Orquidopexia // Plastia del piso del canal inguinal // Ligadura alta del saco
herniario // Colocación de malla

12. Masculino de 36 con I venosa de ext inferiores. Medida antitrombótica (Anticoag de


por vida)

R: Observación // Anticoagulante // Vit K // Aspirina // Embolectomía

13. Masculino de 50 al cual se le realizará hemicolectomía izquierda con anastomosis por


adenocarcinoma. Factor intrínseco

R: Duración de lavado Qx // Falla renal // Técnica Qx // Instrumental // Ventilación del


quirófano

14. Portador de apendicitis. El área de asepsia del abdomen que deberá ser realizada
considerando los siguientes límites

R: Cresta iliaca a reborde costal // Cicatriz umbilical a sínfisis del pubis // Ángulo de Lewis a
sínfisis del pubis // Línea submamaria a tercio superior de muslos // Apéndice xifoides a sínfisis
del pubis

15. Femenina de 38 operada por colecistectomía x microlitiasis vesicula. Actualmente con


dolor en epigastrio con ictericia. Fosfatasa alcalina 350. Dx

R: Pancreatitis aguda // Absceso residual // Hepatitis // Hematoma // Úlcera péptica

16. Px que es operado por cuadro apendicular encontrándose apéndice G4. ¿Cuál
complicación se presenta si al 2 día se presenta salida de material fecaloide por el
drenaje?
R: Absceso residual // Dehiscencia de base apendicular // Necrosis de ciego // Contaminación
por anaerobios // Fístula

17. Masculino de 54 con estenosis pilórica por proceso cicatrizal de ulceración. ¿Qué Cx
es la adecuada?

R: Antrectomía y gastro duodeno anastomosis // Piloromiotomia // Piloroplastia y vagotomía


troncular // Dilatación de estenosis // Antrectomía y gastroyeyunoanastomosis

18. Dentro de las especificaciones que deberá tener un quirófano en base a la normativa de
la SSA, este deberá tener la siguiente….

R: Área de 20 m2 4x5 m // Humedad del 50-60% // Filtración de aire 15 V/m // T de 22-24 //


Presión (-)

19. Masculino de 66 con cirrosis, presenta hemorragia de TDA por varices esofágicas.
Sonda para controlar el sangrado

R: Pezzer // Murphy // Foley // Nelaton // Sengstaken-Blakemore

20. Femenino de 67 con neumopatía crónica obstructiva de 20 años. Parámetro a disminuir


para los requerimientos calóricos

R: CHOS // aas // Proteína // Almidón // Lípidos

21. Femenina de 25 años con estreñimiento severo, dolor y sangrado escaso posterior a
evacuación. Se aprecia desgarro lineal en el anodermo. Lo más probable es que este
desgarro se localice en….

R: Posterior distal a línea dentada // Lateral derecho distal a línea dentada // Anterior distal a
línea dentada // Posterior proximal a línea dentada // Lateral izquierdo distal a línea dentada

22. Px sometida a laparotomía por embarazo tubárico roto. Al día siguiente post Cx se
detecta hematoma de la herida Qx. Causa más frecuente de esta complicación

R: Inadecuada de hemostasia // No se transfundieron plaquetas // Consumo de factores por


sangrado // Deficiente función de tapón de fibrina // Anemia durante el embarazo

23. A 55 man complains of chronic intermittent epigastric pain. A gastroscopy


demonstrates a 2 cm prepyloric ulcer. Biopsy clear. After a 6 week trial of medical
therapy, the ulcer is unchanged. Which of the following is the best step in his
management
R: Vagotomy and pyloroplasty // Repeat trial of medical therapy // Local excision of the ulcer
// Partial gastrectomy with vagotomy and Billroth 1 reconstruction // Highly selective
vagotomy

24. Px postQx de colecistectomía laparoscópica, el cual 24 hrs posteriores presenta


distensión abdominal. Rx de abdomen con aire libre subdiafragmático derecho, además
de niveles hidroaéreos. Na de 140, K de 4, Cl de 100. Dx

R: Íleo post Qx // Perforación de víscera hueca // Imagen compatible con sombra gástrica //
Hematoma subdiafragmático // Bilioma

25. Se realiza colonoscopia a px con antecedentes de sangrado rectal. ¿Cuál de los


siguientes tiene la posibilidad de desarrollar carcinoma de colón?

R: Hemorroides // Pólipo adenomatoso // Divertículos // Fístula perianal // Hemangioma

26. ***Px femenina de 59 años a la cual se realizó endoscopia por referir reflujo gastro-
esofágico y en el reporte endoscopio se describe la presencia de bilis ¿Cual es la
interpretación clínica?

R: Importante valorar si tiene hernia hiatal / Se requiere de manometría / Presenta


incompetencia del esfínter de Oddi / No es patológico / Es portadora de reflujo alcalino

27. Px masculino de 55 años, el cual tiene como antecedente sangrado rectal en 3 ocasiones
el día de hoy se presenta en urgencias por cuadro doloroso abdominal. Al explorarlo
encuentra que se palpa zona indurada en región de sigmoides con dolor importante en
esta área. La temperatura es de 39.5 grados. ¿Cuál sería el dx más probable?

R: Cáncer de colon / apendicitis de localización anómala / colon toxico / Sx de colon irritable


/ Diverticulitis de colon perforado y sellado

28. An 80 year old man with a history of symptomatic cholekithiasis presents with signs
and symptoms of a small-bowel obstruction. Which of the following findings would
provide the most help in ascertaining the dx?

R: Pneumobilia / A palpable mass in the pelvis / A leukocyte count of 40,000/mL / Coffee-


grounds aspirate from the stomach/ A pH of 7.5 PCO2 of 50 kPa, and paradoxically acid urine

29. Femenina de 30 con microlitiasis vesicular tiene dolor con amilasa de 1000, fosfatasa
de 500. Tinte ictérico con evolución de 12 h. Acción adecuada

R: Ninguna // Colecistectomía abierta // Colecistectomía abierta y coledocotomía //


Colecistectomía laparoscópica // CPRE + esfinterotomía
30. Portador de Hernia ventral con evolución de 3 años será sometido a plastia de pared; el
defecto mide 12x10 cm. Opción adecuada

R: Malla pre-peritoneal // Puntos seprados con prolene // Malla a tensión // Técnica de Marcy
// Técnica de Bassini

31. Se encuentra pasando visita y uno de los px presenta dificultad respiratoria cianosis
peribucal, el estudio gasométrico revela CO2 50, O2 50. ¿Qué acción deberá realizar
de inmediato?

R: Intubación / solicitar sodio y cloro para valorar A/G / Oxigeno con puntas nasales /
Traqueostomía / Tomar nueva gasometría para confirmar

32. Px con trauma abdominal por objeto punzo cortante. Signo clínico de aire libre en
cavidad peritoneal

R: Cullen // Jaubert // Grey Turner // Morris // Murphy

33. Usted decide intubar a un px por datos de insuficiencia respiratoria. ¿Cómo se


corrobora la intubación exitosa?

R: Ruidos respiratorios en 4 puntos y SaO2 +80% // Gasometría con CO2 45 mmHg y 02 de


70 mmHg // Observación de tubo en bronquios en Rx // Ruidos respiratorios en dos sitios del
tórax // Paso del tubo por la hipofaringe

34. Px portador de enfermedad diverticular colónica. Ha presentado varios episodios de


agudeza; si presenta neumaturia, ¿Qué problema presenta? ”FÍSTULA”

R: Colón-vesical // Colón-ileón // Colón-sigmoideo // Colon-útero // Sigmoides e íleon

19 04 21 32 27 00 02--- folio

35. Px de 35 años con rectorragia de 3 meses. Estudio para hacer Dx

R: Rectosigmoidoscopia // TAC // ECO // Exploración anal // Colonoscopía


36. A 35 year old woman undergoes an elective laparoscopic cholecystectomy for
symptomatic cholelithiasis. Type of wound

R: G4 or dirty // G2 or clean--contaminated // G1 or clean // G3 or contaminated // None

37. Masculino de 78 ingresa por abdomen agudo y choque séptico, probablemente debidos
a una enfermedad diverticular complicada. Para realizar la medición de PVC requiere
realizar el siguiente procedimiento
R: Vena de la fosa antecubital por punción // Aplicar catéter en vena subclavia // Vena safena
interna por venodisección // Aplicar catéter en femoral por punción // Colocación de sello de
agua

38. Masculino de 80 con perforación por proceso ulceroso prepilorico de 24 h. ¿Qué


proceso Qx haría si el px presenta datos de choque mixto y comórbidos como DM y
neumopatía por tabaquismo?

R: Laparotomía exploradora, cierre de perforación con vycril 1 // Laparotomía de cierre de


perforación con crómico, vagotomía troncular, drenaje de cavidad // Laparoscopia cierre de
perforación y no drenajes // Laparoscopia aplicación de parche de epiplón, lavado de cavidad,
vagotomía troncular y drenaje // Laparotomía cierre de perforación con seda 2-0, lavado de
cavidad y drenajes.

39. A 3 year old boy is brought after spilling bleach into his lower extremities. In addition
to resuscitation, which is the most appropriate initial management

R: Neutralize the burn wound with weak acids // Wound debridement in the OR // Lavage of
the burn with large volumes of water // Tx the burn with calcium gluconate gel // Tx of the
burn with antimicrobial agents

40. A 25 year old man is brought after sustaining burns during a fire. He has blistering and
erythema on his face, left upper extremity and chest. He also has circumferential frank
charring of his right upper extremity with decreased capillary refill. He is agitated,
hypotensive, and tachycardic. Initial management

R: Split-thickness skin grafts over the areas of third degree burns // Escharotomy of the right
upper extremity // Excision of all third degree burns // Excision of facial and hand burns //
Topical antibiotics to the wounds
2do Banco

CIRUGÍA LA MEDIDA PREVENTIVA PARA ESTA


COMPLICACIÓN:
FEMENINO DE 42ª DE EDAD A QUIEN SE LE USO DE MEDIAS PLASTICAS
REALIZO COLICISTECTOMIA ABIERTA POR
COLECISTITIS CRÓNICA LITIASICA AL SALIR FEMENINO DE 48ª MULTIPARA, OBESA,
DE QUIRÓFANO TIENE COLOCADA UNA PROGRAMADA PARA HISTERECTOMÍA
SONDA NASOGASTRICA, SONDA VESICAL, VAGINAL POR PROLAPSO UTERINO, COMO
VENOCLISIS CON HARTMAN GLUCOSADA ANTECEDENTE MENCIONA SER DIABÉTICA Y
RESPIRACIÓN 18 X MIN. PULSO 75X´. TA PRESENTA INSUFICIENCIA VENOSA DE
130/85. TEMP 36.5° ESTE PACIENTE DEBERA MIEMBROS INFERIORES, CUAL DE LAS
DEAMBULAR: SIGUIENTES COMPLICACIONES
AL DIA SIGUIENTE DE LA CIRUGÍA PULMONARES ES PROBABLE QUE
PRESENTE ESTA PACIENTE EN EL
PACIENTE DE 56ª A QUIEN SE LE REALIZO POSTOPERATORIO:
ANTRECTOMIA CON RECONSTRUCCIÓN TROMBOEMBOLIA PULMONAR
BILIAR POR PRESENTAR ULCERA GÁSTRICA
PENETRADA A PÁNCREAS SE ENCUANTRA EN FEMENINO OBESA DE 35ª LA CUAL VA A SER
EL 3ER DIA POSTOPERATORIO CON SONDA SOMETIDA A COLECISTECTOMIA POR
NASOGASTRICA, SONDA VESICAL CON PRESENTAR COLECISTITIS CRÓNICA
HARTMAN GLUCOSADA Y CIPROFLOXACINO LITIASICA ADEMÁS DE TRATAMIENTO
500MG IV CADA 12 HRS, RESP 16X/ PULSO 75X ANTIRREFLUJO POR INCOMPETENCIA DEL
TA 130/85. CUANDO SE DEBERA RETIRAR LA ESFÍNTER ESOFÁGICO INFERIOR, LAS
SONDA DE LEVIN EN ESTE PACIENTE: ORDENES PREOPERATORIOS DEBERAN
AL PRESENTAR RUIDOS INTESTINALES INCLUIR:
AUTORIZACIÓN DE LA CIRUGÍA
INGRESA MASCULINO DE 22 AÑOS AL
SERVICIO DE URGENCIAS REFIRIENDO FEMENINO DE 40ª CON SIRPA Y
DOLOR EN FOSA ILIACA DERECHA. SE DESNUTRICIÓN PROTEICO CALORICA EN LA
DIAGNOSTICA APENDICITIS Y SE REALIZA ALIMENTACIÓN INTRAVENOSA PERIFERICA
APENDICECTOMIA. EL HALLAZGO EL INCREMENTO DE CALORÍAS SE LOGRARA
QUIRÚRGICO FUE APENDICITIS EN FASE MEDIANTE:
EDEMATOSA AL MOMENTO DE SU EGRESO LÍPIDOS AL 10%
HOSPITALARIO SE LE RETIRARAN LOS
PUNTOS A LOS: PACIENTE DE 50ª CON CANCER DE ESÓFAGO
7 DIAS Y CON INANICIÓN EN ESTA CIRCUNSTANCIA
SU CORAZON Y MÚSCULO TIENEN
MASCULINO DE 22ª EL CUAL REFIERE DOLOR PREDILECCIÓN POR UNO DE LOS SIGUIENTES
EN FOSA ILIACA DERECHA EL PACIENTE FUE AMINOÁCIDOS:
SOMETIDO A APENDICECTOMIA EL DE CADENA RAMIFICADA
HALLAZGO QUIRÚRGICO FUE APENDICITIS
PERFORADA (FASEIV) QUE TIPO DE CIERRE MASCULINO DE 36ª QUIEN DURANTE LA
PARA LA PIEL Y TEJIDO CELULAR INANICIÓN AGUDA DEVIDO A CANCER
SUBCUTÁNEO ES RECOMENDADO EN ESTE LARINGEO. TIENE UNA PERDIDA NETA DE
CASO: AMINOÁCIDOS, YA QUE LA SINTESIS DE
DIFERIDO PROTEINA MUSCULAR DESAPARECE O
DESCIENDE TANTO QUE EL CATABOLISMO
FEMENINO DE 38ª MULTIPARA, CON DE LA MISMA PERMANECE SIN CAMBIOS
CISTOCELE PROGRAMADA PARA ESTA SITUACIÓN ES DEVIDA A:
COLPOPERINEOPLASTIA LA PREPARACIÓN CONCENTRACIONES BAJAS DE INSULINA
DE LA VAGINA SE RELIZA EN:
QUIRÓFANO FEMENINO DE 28ª POLITRAUMATIZADO QUE
SE ENCUENTRA EN EL POSTOPERATORIO
MASCULINO DE 22ª SOMETIDO A QUE PESA 90KG TALLA 1.5M, QUE INGIERE
LAPAROTOMIA CON HALLAZGO DE 10GM DE PROTEINAS AL DIA.EN 2 LITROS DE
PERFORACIÓN DE ILEON POR ORINA ESTA ELIMINANDO 950MG/ DIA, DE
SALMONELLOSIS AL 3ER DIA NITRÓGENO UREICO. CON ESTO EL BALENCE
POSTOPERATORIO INICIA CON FIEBRE, DE NITRÓGENO AL DIA ES DE:
DOLOR, TUMEFACCIÓN, HIPEREMIA Y 21.4GM
AUMENTO DE LA TEMPERATURA LOCAL.
CUAL DE LAS SIGUIENTES ES LA CAUSA PACIENTE QUE FUE INTERVENIDO
PROBABLE CON ESTA COMPLICAION: QUIRÚRGICAMENTE REALIZÁNDOSELE
CIERRE PRIMARIO DE LA HERIDA COLECISTECTOMIA POR PRESENTAR
COLECISTITIS CRÓNICA LITIASICA, CON EL
MASCULINO DE 72ª OBESO, SOMETIDO A POSTOPERATORIO MEDIATO PRESENTO
COLECISTECTOMIA ABIERTA POR DEHISCIENCIA DE LA HERIDA QUIRÚRGICA.
COLECISTITIS AGUDA AL 4TO, DIA POST PARA REPARAR LA COMPLICAION Y CERRAR
OPERATORIO PRESENTA EDEMA. DOLOR, LA APONEUROSIS ELIGIRA EL SIGUIENTE
HIPERTEMIA Y COLORACIÓN VIOLACEA EN MATERIAL DE SUTURA:
EL MIEMBRO INFERIOR DERECHO, CUAL ES PROLENE 1
MASCULINO DE 45ª EL CUAL SUFRIO DE AUTOMOVILISTICO CON LESIONES LEVES.
TRAUMATISMO ABDOMINAL EN ACCIDENTE PERO FUE VICTIMA DE CRISIS EMOCIONAL,
AUTOMOVILISTICO Y QUIEN A SU INGRESO A SE LE TOMA GASOMETRIA REVELANDO UNA
URGENCIAS LLEGA EN ESTADO DE CHOQUE PCO2 DE 30MMHG TAL CIFRA Y EL CUADRO
HIPOVOLEMICO PARA QUE DESENCADENE CLINICO SUGIEREN:
LA RESPUESTA BIOLÓGICA AL TRAUMA ALCALOSIS RESPIRATORIA
DEBERA EXISTIR:
INTEGRACIÓN NOCICEPTIVA EN LE CEREBRO ANTE UN PACIENTE EL CUAL PRESENTA
VOMITOS FRECUENTES DE CONTENIDO
FEMNINO DE 34ª QUIEN SUFRIO GÁSTRICO SECUNDARIO A ESTENOSIS
POLITRAUMATISMO EN ACCIDENTE PILORICA LA ANORMALIDAD METABÓLICA
AUTOMOVILISTICO Y QUE ES VISTO EN EL PREDOMINANTE QUE ESPERARIA
SERVICIO DE URGENCIAS. EN EL ESTUDIO DE ENCONTRAR SERIA:
ESTE PACIENTE SE SABE QUE DENTRO DE LA ALCALOSIS HIPOCLOREMICA
RESPUETA METABÓLICA AL TRAUMA SE
PRODUCE UNA CERIE DE HORMONAS MASCULINO DE 22ª QUIEN SE CAYO DESDE
LLAMADAS CONTRARREGULADORAS DOS UN CUARTO PISO SE LE REALIZO
DE LAS CUALES SON: HEPATECTOMIA PARCIAL, RESECCION DE
EPINEFRINA Y NORADRENALINA 30CM DE INTESTINO DELGADO ASI COMO
ESPLENECTOMIA. RECIBIO 15UNIDADES DE
SANGRE EN EL PRE Y TRANS OPERATORIO. 8
LITROS DE SOLUCION HARTMAN Y 4
UNIDADES DE PLASMA FRESCO CONGELADO
ANTE UN PACIENTE QUE SE ENCUENTRA DURANTE EL CIARRE DE LA LAPAROTOMÍA
CURSANDO EL PRIMER DIA DEL SE NOTO SANGRADO DIFUSO. DE LAS
POSTOPERATORIO POR HABER SIDO SIGUIENTES CAUSAS CUAL ES LA MAS
SOMETIDO A TIREIDOCTOMIA SUBTOTAL PROBABLE DE ESTE SANGRADO:
POR PRESENTAR BOCIO MULTINODULAR PLAQUETOPENIA
UNA VEZ SUPERADA LA FASE EBB, USTED
ESPERA QUE EL PACIENTE SE ENCUENTRE EN FEMENINO DE 28ª LA CUAL PRESENTO
LA FASE: TRAUMA ABDOMINAL CERRADO CON
DE FLUJO RUTURA HEPÁTICA E ISQUEMIA INTESTINAL
POR DESGARRO DEL MESENTERIO, DURANTE
FEMENINO DE 30ª QUIEN SUFRE ACCIDENTE UNA INTERVENCIÓN QUIRÚRGICA PRESENTA
EN CARRETERA. EN EL LUGAR DEL SANGRADO DIFUSO DE LA
ACCIDENTE SE LE ENCUENTRA PARED ABDOMINAL Y EN HIGADO DEBIDO A
SEMICONSCIENTE, TAQUICARDIA, EN UNA PROBABLE CID, EL TRATAMIENTO QUE
TORAX PULMONES VENTILADOS, ABDOMEN RELIZARA EN ESTE MOMENTO SERA:
CON PARISTALSIS DISMINUIDA Y DOLOR EMPAQUETAMIENTO ABDOMINAL
GENERALIZADO. SE LE CANALIZA CON
HARTMAN Y ES LLEVADO AL CENTRO DE FEMENINO DE 39ª CON DOLOR EN
TRAUMA MAS CERCANO. DE LAS HIPOCONDRIO DERECHO SE REALIZA USG DE
SIGUIENTES CUAL ES LA QUE DISMINUYE EN PÁNCREAS. HIGADO Y VIA BILIAR Y SE
LA FASE INICIAL EN LA RESPUESTA DETECTA ENFERMEDAD CALCULOSA DE LA
BIOLÓGICA AL TRAUMA: VESÍCULA POR LO QUE FUE SOMETIDA A
CORTISOL CELECISTECTOMIA SIMPLE SE SOLICITAN
EXAMENES PREOPERATORIOS LOS CUALES
MASCULINO DE 68ª EL CUAL FUE SOMETIDO SON NORMALES, SI USTED DESEA DETECTAR
A RESECCION ABDOMINOPERINEAL POR DEFICIENCIA DEL FACTOR II CUAL ES EL
PRESENTAR CANCER DE RECTO, DURANTE ESTUDIO UTIL PARA EVALUARLO:
LA CIRUGÍA LA PERDIDA SANGUÍNEA FUE TIEMPO DE PROTROMBINA
RESPUESTA VOLUMEN A VOLUMEN, EN EL
POSTOPERATORIO FUE MANEJADO CON MASCULINO DE 42ª QUIEN INGRESA CON
SOLUCIONES DE CLORURO DE SODIO AL SANGRADO DE TUBO DIGETIVO ALTO.
0.45% PRESENTANDO DEBILIDAD, LETARGIA, ENTRE SUS ANTECEDENTES MENCIONA
CONVULSIONES Y POSTERIORMENTE COMA ALCOHOLISMO CRÓNICO. LABORATORIO:
POR LO REFERIDO EL PACIENTE ESTA HB 10GM/ DL, HTO 33%, PLAQUETAS 300,000.
CURSANDO CON: TPT 40´. A LA EXPLORACIÓN FÍSICA
HIPONATREMIA PACIENTE TRANQUILO; CONSIENTE TA.
100/60, SE COLOCA SONDA NASOGASTRICA Y
PACIENTE QUE PRESENTO TRAUMATISMO SE LE REALIZA LAVADO GÁSTRICO, DENTRO
ABDOMINAL CERRADO Y CHOQUE DEL MANEJO DE ESTE PACIENTE INDICA
HIPOVOLEMICO, EL PUNTO DE VISTA TRANSFUSIÓN DE :
FISIOPATOLOGICA A NIVEL HORMONAL, CONCENTRADO PLAQUETARIO
ESPERARIA ENCONTRAR AUMENTO DE LA
SECRECION DE LA SIGUIENTE HORMONA: PACIENTE EL CUAL FUE SOMETIDO A
CORTISOL HEMICOLECTOMIA POR CANCER DE COLON.
EN EL POSTOPERATORIO PRESENTA CUADRO
FEMENINO DE 25ª QUIEN SUFRIO ACCIDENTE COMPATIBLE CON CID LA ALTERACIÓN
LABORATORIAL QUE ESPERARIA COMPORTAMIENTOS DE LOS LIQUIDOS
ENCONTRAR SERA: CORPORALES ES:
PRESENCIA DE P. D. F NA +

MASCULINO DE 45ª A QUIEN SE LE REALIZO FEMENINO DE 40ª LA CUAL INGRESO A LA


CESAREA Y RESECCION DE 30CM DE UCI POR PRESENTAR CHOQUE SÉPTICO
INTESTINO DELGADO POR TRAUMATISMO SECUNDARIO A UNA PERFORACIÓN
ABDOMINAL CERRADO. DURANTE LA INTESTITAL. SE LE HAN APLICADO
CIRUGÍA SE LE TRANSFUNDIERON 10 VOLÚMENES ADECUADOS DE RINGER
UNIDADES DE SANGRE DEL BANCO. A LACTADO CONTROLADOS POR LA PRESION
PRESENTADO SANGRADO DIFUSO Y PIENSA EN CUÑA PULMONAR, DOPAMINA Y
EN CID, ESTA COMPLICAION SE NOREPINEFRINA A PESAR DE LAS MEDIDAS
EVIDENCIARA POR EL SIGUIENTE ANTERIORES LA PACIENTE NO HA
RESULTADO DE LABORATORIO: MOSTRADO MEJORIA, ES MAS SE HA
PRESENCIA DE PRODUCTOS DE DIVISIÓN DE AGREGADO ICTERICIA Y CONFUCION
FIBRINA MENTAL. POR LO QUE MUESTRA EL
PACIENTE ESTA PRESENTANDO:
FEMENINO DE 28ª SE LE REALIZO CESAREA SÍNDROME DE DISFUNCIÓN ORGANICA
POR PRESENTAR OBITO DE 24 SEMANAS DE MÚLTIPLE
GESTACIÓN Y A QUIEN EN EL
POSTOPERATORIO SE LE DIAGNOSTICO CID. MASCULINO DE 55ª EL CUAL FUE SOMETIDO
CON EL OBJETO DE REPONER LA MAYOR A GASTRECTOMÍA SUBTOTAL POR
CANTIDAD DE LOS FACTORES DE LA PRESENTAR CANCER IN SITU EN EL
COAGULACION CONSUMIDOS, CUAL DE LAS POSTOPERATORIO MEDIATO ES
SIGUIENTES MEDIDAS ES LA MAYOR FUENTE INDISPENSABLE EL USO DE ANELGESICOS
DE LOS FACTORES DE LA COAGULACIÓN ´´. CON HORARIO, PARA PREVENIR LA
PLASMA FRESCO CONGELADO SIGUIENTE COMPLICACIÓN:
SÍNDROME DE DISFUNCIÓN ORGANICA
MÚLTIPLE
* MASCULIJNO DE 53ª QUIEN INGRESO A UCI
POR PRESENTAR HIPOTENCION SEVERA CON
SIFRAS TENCIONALES DE TA 70/40. MASCULINO DE 55ª EL CUAL FUE SOMETIDO
REVELADO A LA APLICAION DE VOLÚMENES A GASTRECTOMÍA SUBTOTAL POR
ADECUAOS DE RINGE LACTATO E PRESENTAR CANCER IN SITU EN EL
INOTROPICOS, ADEMÁS DE FIEBRE E POSTOPERATORIO MEDIATO ES
INSUFICIENCIA RENAL, SI FINALMENTE CON INDISPENSABLE EL USO DE ANALGÉSICOS
LAS MEDIDAD ANTERIORES LA TA NO SE CON HORARIOS, PARA PREVENIR LA
RECUPERA USTED OPTARIA POR INDICAR: SIGUIENTE COMPLICACIÓN:
APLICACION DE VASOPRESINA ATELECTASIA

MASCULINO DE 23ª QUIEN PRESENTA DESDE


HACE 3 DIAS, ICTERICIA, FIEBRE, DOLOR
ABDOMINAL, HECES HIPERPIGMENTADAS Y
FEMENINO DE 28ª QUIEN INGRESA POR ORINA CLARA. A LA EXPLORACIÓN EXISTE
SANGRADO TRANSVAGINAL DOLOR A LA PALPACIÓN DELCUADRANTE
DETERMINÁNDOSE COMO CAUSA ABORTO IZQUIERDO DONDE SE PALPA UNA MASA
INCOMPLETO EN EL PRIMER TRIMESTRE POR QUE SOBRE SALE DEL BORDE COSTAL Y
LO QUE SE RELIZA LEGRADO UTERINO A SIGUE LOS MOVIMIENTOS RESPIRATORIOS,
CONSECUENCIA DE ELLO LA PACIENTE ESTA ESTE CUADRO SE REPITE UNA O DOS VECES
EN CHOQUE HIPOVOLEMICO LA DIFERENCIA AL AÑOS Y SE PRESENTA TAMBIEN EN 2
DE O2 ARTERIO VENOSO EN PULMON EN LA HERMANOS SUYOS, EL DIAGNOSTICO
FASE TEMPRANA DEL CHOQUE PROBABLE ES:
HIPOVOLEMICO SE ENCUENTRA: HIPERESPLENISMO
PEQUEÑA DIFERENCIA DEL O2
ARTERIOVENOSO
MASCULINO DE 47ª EL CUAL HA PADECIDO
MASCULINO DE 53ª QUE INGRESO A LA UCI DE DOLOR ABDOMINAL RECURRENTE
POR PRESENTAR ESTABILIZACIÓN DEL DURANTE LOS ULTIMOS 3 AÑOS.
CHOQUE HIPOVOLEMICO Y MEJOR ACTUALMENTE PRESENTA DOLOR
CONTROL, POR LO QUE SE COLOCA UN EPIGASTRICO INTENSO Y CONTINUO,
CATETER DE SWANZ GANZ PARA VALORAR: NAUSEAS DE 6 HORAS DE VOMITOS QUE NO
P. V. C ALIVIAN EL DOLOR EN LOS ULTIMOS
MOMENTOS EL DOLOR TIENDE A
MASCULINO DE 60ª A QUIEN SE LE REALIZO IRRADIARSE AL DORSO Y HEMIABDOMEN
RESECCION INTESTINAL EN EL IZQUIERDO A LA EXPLORACIÓN PRESENTA
POSTOPERATORIO INMEDIATO EL RESISTENCIA ABDOMINAL EPIGASTRICA.
ELECTROLITO QUE REFLEJA MEJOR LA CON LOS DATOS
ALTERACIÓN EN LA COMPOSICIÓN DE LOS
ANTERIORES EL PADECIMENTO SERA:
PANCREATITIS AGUDA FEMENINO QUE ABSRCA UNA QUEMADURA
QUE ABRCA UNA SUPERFICIE CORPORAL
MASCULINO DE 47ª EL CUAL HA QUEMADURA EN TODA LA CARA ANTERIOR
PRESENTADO DOLOR ABDOMINAL DE TIPO DEL TRONCO, MIEMBRO SUPERIOR
ARDOSO DESDE HACE 3 AÑOS, QUE LO DERECHO, MANO IZQUIERDA EN LA CABEZA
DESPIERTA POR LAS NOCHES, DESDE HACE QUEMADURAS EN CEJAS, PESTAÑAS Y
UN AÑO TOMA DE FORMA IRREGULAR VIBRISAS QUEMADAS Y EXPECTORACION
ANTIÁCIDOS Y BLOQUEADORES H2 CARBONEADA ADEMÁS DE INCONCIENCIA.
PRESCRITOS POR FACULTATIVO, SIN SEGÚN ELCALCULO DE LAS REGIONES
MEJORIA APARENTE, ACTUALMENTE EL LESIONADAS. POR SU SUPERFICIE
DOLOR ES CONTINUO, MAS INTENSO Y SE CORPORALEQUIVALE A:
IRRADIA AL DORSO. LA TERAPEUTICA 50-60%
ADECUADA ES:
GASTRECTOMÍA PARCIAL MASCULINO DE 30ª QUIEN PRESENTA
QUEMADURAS SECUNDARIAS A EXPOSICIÓN
MASCULINO DE 22ª INGRESA AL SERVICIO DE DE FUEGOS PIROTÉCNICOS EN CARA,
URGENCIAS EN DONDE USTED SE MANOS, GENITALES, TORAX 70% PARA
ENCUENTRA, REFIRIENDO DOLOR EN FOSA VALORAR LA PERFUSION RENAL DURANTE
ILIACA DERECHA POR LO QUE SE LE EL TRATAMIENTO DE REPOSICIÓN DE
DIAGNOSTICA APENDICITIS, DE LOS LIQUIDOS:
SIGUIENTES EXMENES PREOPERATORIOS DEBERA VIGILARSE DIURESIS HORARIA A
CUALES SERIAN DE IMPORTANCIA : RAZON DE 30-50
RX SIMPLE DE EBDOMEN
TP Y TPT MASCULINO DE 30ª CON QUEMADURAS DEL
BIOMETRÍA HEMATICA 60% DE SEGUNDO GRADO SUPERICIALES Y
QUÍMICA SANGUÍNEA PROFUNDAS. EL CUAL HA ESTABILIZADO
HEMODINAMICO, LA DIETA DEBE HACERSE
MASCULINO DE 67ª PROSTATICO DIABÉTICO, POR LA VIA:
OBESO Y CON EPC, QUIEN HACE UN MES ORAL
EMPEZO A NOTAR MASA TUMORAL EN REGIO
INGUINAL DERECHA QUE DESAPARECE PACIENTE QUE SUFRE ACCIDENTE
RAPIDAMENTE AL ADOPRTAR EL DECUBITO AUTOMOVILISTICO Y QUE AL EXAMEN SE
DORSAL PARA MANEJAR ESTAS ENCUENTRA DESPIERTO Y RESPONDE A LAS
CONDICIONES SIGUIENTES EL TRATAMIENTO PREGUNTAS A NIVEL DE TORAX SE
INDICADO EN ESTE PACIENTE ES: ADOPTAR ENCUANTRAN DATOS QUE SUGIEREN
EL USO DE BRANGUERO NEUMOTORAX A TENSIÓN EN HEMITORAX
DERECHO, LA MEDIDA A APLICAR PARA ESTE
PACIENTE DE 50 AÑOS A QUIEN SE LE PROBLEMA SERA: TORACOCENTESIS EN 2°
PRACTICO LAPAROTOMÍA MEDIA ESPACION INTERCOSTAL
EXPLORADORA PRESENTA EN EL 5° DIA DEL
POSTOPERATORIO UNA EVICERACION. ESTA UN PACIENTE POLITRAUMATIZADO QUIEN
COMPLICACIÓN SE DEFINE COMO: EN EL EXAMEN LE ENCONTRAMOS EN
SEPARACIÓN DE TODOS LOS PLANOS DE LA HEMITORAX IZQUIERDO SILENCIO
HERIDA RESPIRATORIO A LA PERCUSIÓN
TIMPANISMO Y CON CHOQUE DE LA PUNTA A
LA DERECHA REALIZÁNDOSELE
PACIENTE DE 45ª EL CUAL PRESENTA UNA TORACOCENTESIS DE URGENCIA, LA
MASA EN REGION INGINAL IZQUIERDA JUSTIFICACIÓN DE ESTA MEDIDA ES POR:
DESDE HACE 3 AÑOS LA CUAL SE HACE ES LA DE ELECCIÓN PARA MANEJAR LA
EVIDENTE AL ESFUERZO Y DISMINUYE CONFUSIÓN PULMONAR
CON EL REPOSO. NO LLEGANDO AL ESCROTO
EN LA EXPLORACIÓN AL PEDIRLE QUE PUJE MASCULINO DE 34ª LLEVADO A URGENCIAS
LA MASA HACE CONTACTO CON EL PULPEJO POR SUFRIR ACCIDENTE AUTOMOVILISTICO
DEL DEDO EXPLORADOR EN ESTE CASO EL AL EXAMEN ENCONTRAMOS
DEFECTO SE ENCUENTRA EN: HIPERSENSIBILIDAD LEVE EN ABDOMEN Y
DENTRO DEL TRIANGULO DE HASSELBASH HEMITORAX DERECHO CON PERISTALSIS
MUY DISMINUIDA Y AUSENCIA DE METIDEZ
HEPÁTICA CUAL ESTUDIO DIAGNOSTICO:
FEMENINO DE 45ª QUIEN REFIERE LA RUPTURA DE VICERA HUECA
PRESENCIA DE UNA MASA A NIVEL
ABDOMINAL. NO DOLOROSA. A LA EF SE MASCULINO DE 41ª EL CUAL SUFRIO
ENCUENTRA MASA DE 2X2. POCO MOVIL, ACCIDENTE AUTOMOVILISTICO EN EL
BLANDA, NO DOLORSA, ENTRE APÉNDICE MOMENTO DE ARRIBAR A LA AMBULANCIA
XIFOIDES Y CICATRIZ UMBILICAL. EN LA USTED DETECTA QUE EL, PACIENTE SE
LINEA MEDIA, EN ESTE CASO CUAL SERIA LA ENCUENTRA SEMICONSCIENTE CON
TERAPEUTICA MAS ADECUADA: CIANOSIS PERIFERICA, LESION
PLASTIA MAXILOFACIAL IMPORTANTE.
MOVIMIENTOS RESPIRATORIOS APENAS
PERCEPTIBLES Y LA PRESENCIA DE
FRACTURA EXPUESTA DE TIBIA Y PERONE, CIRCUNSTANCIAS DESDE EL PUNTO DE
CONSIDERANDO LAS CONDICIONES EN LAS VISTA LABORATORIAL PODEMOS
QUE SE ENCUANTRA EL PACIENTE CUAL ES CORRABORAR NUESTRO DIAGNOSTICO SE
LA PRIMER MEDIDA QUE APLICARIA: LA ALBÚMINA SE ENCUANTRA DENTRO DEL
CRICOTIROIDOSTOMIA SIGUIENTE RANGO:
1.5-20G/DL
MASCULINO DE 60 AÑOS QUE ES INGRESADO
A CIRUGÍA PARA LAVADO DE CAVIDAD PACIENTE QUE SUFRIO QUEMADURA DE 2°
PERITONEAL POR SEPSIS ABDOMINAL EN QUE ABARCAN EL 30% DE LA SUPERFICIE
QUIRÓFANO. PRESENTA PARO CORPORAL EN ESTA PACIENTE EL
CARDIORRESPIRATORIO, A LA EXPLORACIÓN INCREMENTO DE LAS NECESIDADES
FÍSICA RUIDOS CARDIACOS POCO AUDIBLES, CALORICAS BASALES SERAN EN UN % DE
FRECUANCIA RESPIRATORIA NO 100-200
PERCEPTIBLE. NO HAY PULSOS EN ANTE UN PACIENTE POLITRAUMATIZADO Y
CAROTIDAS Y FEMORALES, CON ESTOS QUE FUE SOMETIDO A LAPAROTOMÍA
DATOS CLINICOS ES DETERMINANTE PARA EXPLORADORA PODEMOS ENCONTRAR UNA
EL DIAGNOSTICO DE PARO: DIFERENCIA QUE NO EXISTE EN EL PACIENTE
PERDIDA DE MOVIMIENTOS RESPIRATORIOS SOMETIDO A AYUNO CUAL ES ESTA?
ACTIVACION DEL SITEMA
MASCULINO DE 29ª PRESENTA DOLOR NEUROENDOCRINO
ABDOMINAL EN FID. CONTINUO,
ACOMPAÑADO DE HIPERTEMIA Y NAUSEAS,
A LA EXPLORACIÓN FÍSICA , MC BURNEY (+),
ROVSING (-), POR LO QUE ES PROGRAMADO
A CIRUGÍA, UNA VEZ QUE USTED SE VISTIO EN UN PACIENTE POLITRAUMATIZADO SE
CON ROPA QUIRÚRGICA PARA INGRESAR AL PRODUCE UNA ACTIVACION DEL PROBLEMA
AREA GRIS ADEMÁS USTED DEBE NEUROENDOCRINO
COLOCARSE: CUYA REPERFUSION METABÓLICA
BOTAS NUTRICIONAL PRODUCE:
ACELERACIÓN DEL CATABOLISMO PROFEICO
FEMENINO DE 35ª QUIEN VA A SER SOMETIDA MUSCULAR
A COLECISTECTOMIA POR COLECISTITIS
CRÓNICA LITIASICA SIN NINGUN MASCULINO DE 35ª EL CUAL SE ENCUENTRA
ANTECEDENTE PATOLÓGICO DE EN LA UCI, CON DIAGNOSTICO DE
IMPORTANCIA, LAS ORDENES PANCREATITIS AGUDA, DESDE HACE 10 DIAS
PREOPERATORIOS DEBERAN INCLUIR: ACTALMENTE EL ORGANISMO TIENE ALTOS
FIRMA DE LA AUTORIZACIÓN DE LA CIRUGÍA NIVELES DE INSULINA

MASCULINO DE 22ª INGRESA AL SERVICIO DE FEMENINO DE 28ª CON SEPSIS ABDOMINAL A


URGENCIAS REFIRIENDO DOLOR EN FID. EL LA EF PACIENTE SOMNOLIENTA. SIGNOS
HALLAZGO QUIRÚRGICO FUE APENDICITIS Y VITALES: 80/60, FC120X, FR36X, SU
SE REALIZA TRATAMIENTO INMEDIATO
APENDICECTOMIA EN FASE 1 (EDEMATOSA) CONSISTE EN:
AL MOMENTO DE SU EGRESO HOSPITALARIO CONTROL DE LA INFECCIÓN Y
LO CITA PARA EL RETIRO DE PUNTOS A LOS: MANTENIMIENTO DEL
7 DIAS VLUMEN SANGUÍNEO

MASCULINO DE 60ª CON DIAGNOSTICO DE MASCULINO DE 55 EL CUAL FUE SOMETIDO A


HERNIA INCICIONAL, INFRAUMBILICAL GASTRECTOMÍA SUBTOTAL POR CANCER IN
GIGANTE, SE PROGRAMA CIERRE DEL SITU. EN EL POSTOPERATORIO MEDIATO ES
DEFECTO HERNIARIO, PARA EVALUAR SU INDISPENSABLE EL USO DE ANALGÉSICOS
RIESGO QUIRÚRGICO DEBEMOS EVALUAR: CON HORARIOS, PARA PREVENIR LA
HISTORIA CLINICA SIGUIENTE COMPLICACIÓN:
ATELECTASIA
MASCULINO DE 72ª OBESO, SOMETIDO A
COLECISTECTOMIA ABIERTA POR MASCULINO DE 23ª QUIEN REFIERE DOLOR
COLECISTITIS AGUDA, AL 4TO. DIA DE POST ABDOMINAL DE PREDOMINIO EN
OPERATORIO PRESENTA EDEMA, DOLOR, EPIGASTRICO Y FID, PARA POSTERIORMENTE
HIPERTERMIA Y COLORACIÓN VIOLACEA EN IRRADIARSE A TODO EL ABDOMEN CON
MIEMBRO INFERIOR DERECHO, CUAL ES HIPERTERMIA Y VOMITOS. A LA EF EN
LA MEDIDA PREVENTIVA PARA ESTA ABDOMEN PERISTALSIS AUSENBTE CON
COMPLICACIÓN: REBOTE (+++) SE INTERVIENE DE URGENCIAS
UTILIZACIÓN DE MEDIAD ELASTICAS ENCONTRÁNDOSE APENDICITIS GRADO IV.
DURANTE EL CIERRE EL MANEJO ADECUADO
ANTE UN PACIENTE QUE SE ENCUANTRA PARA LA PIEL ES:
HOSPITALIZADO Y CURSANDO 2° DIA CIERRE DEFERIDO
POSTOPERATORIO POR COMPLICACIONESDE
PANCREATITIS NECROTICA HEMORRAGICA Y MASCULINO DE 56ª QUIEN PADECE DE
DE QUIEN SE SOSPECHA QUE CURSE CON DOLOR ABDOMINAL DESDE HACE 5ª
UNA DESNUTRICIÓN GRAVE, EN ESTAS ARDOROSO EN EPIGASTRIO, TOMANDO
ANTIÁZ PRESENTA DOLOR INTENSO REGION INGUINAL DERECHA QUE
ARRADIADO TODO EL ABDOMEN. A LA EF DESAPARECE RAPIDAMENTE AL ADOPTAR EL
PERISTALSIS AUSENTE, RESISTENCIA DECUBITO DORSAL. PARA MANEJAR ESTA
MUSCULAR CON REBOTE PRESENTE (+++) SE CONDICION EL TRATAMIENTO INDICADO EN
DECIDE PASAR A CIRUGÍA DENTRO DE LAS ESTE PACIENTE ES:
INCISIONES CUAL ES LA MÁS ADECUADA: TRATAR DE DISMINUIR LA OBESIDAD DEL
MEDIA SUPRA INFRAUMBILICAL PACIENTE YDESPUÉS OPERARLO

MASCULINO DE 22ª INGRESA A URGENCIAS MASCULINO 60ª PROSTATICO, DIABÉTICO,


REFIRIENDO DOLOR EN FID POR LO QUE SE OBESO QUIEN DESDE HACE TRES DIAS
DIAGNOSTICA APENDICITIS DE LOS EMPEZO A NOTAR UNA MASA EN REGION
SIGUIENTES EXAMENES PREOPERATORIO ANGUINAL IZQUIERDA HEMIESTERICA
CUAL SERIA DE MAYOR IMPORTANCIA: QUE DESAPARECE RAPIDAMENTE AL
EKG ADOPTAR EL DECUBITO DORSAL. POR TODAS
LAS PATOLÓGICAS QUE PRESENTA ESTE
PACIENTE DE 42ª CON HISTORIA DE PACIENTE LOS MAS PROBABLES ES
CUADROS DOLOROSOS EN CUADRANTE QUE LA DEFORMACIÓN MENCIONADA
SUPERIOR DERECHO ACOMPAÑADOS DE CORRESPONDA:
NAUSEAS Y VOMITOS DE 2 AÑOS DE HERNIA INGUINAL DIRECTA
EVOLUCION ACTUELMENTE CON COLURIA,
ICTERICIA Y ACOLLA. QUE ESTUDIO SE USA MASCULINO DE 67ª CON DIAGNOCTICO DE
PARA CONFIRMAR EL DIAGNOSTICO: HERNIA INGUINAL DERECHA DENTRO DE
ULTRASONOGRAFIA LOS ESTUDIOS PARACLINICOS
PREOPERATORIOS. LOS MAS IMPORTANTES
MASCULINO DE 30ª QUIEN PRESENTA DESDE EN EL CASO SON:
HACE 3 AÑOS DOLOR EPIGASTRICO TELE DE TORAX Y ELEC TROCARDIOGRAMA
RECURRENTE, EL QUE SE PRESENTA
COMÚNMENTE POR LA MADRUGADA. DESDE
HACE 6 HORAS PRESENTA HEMATEMESIS, SU EN UN PACIENTE QUEMADO DE 7OKG DE
DIAGNOSTICO ORIENTA HACIA: PESO CON 30% DE SUPERFICIE QUEMADA DE
ULCERA PEPTICA SEGUNDO GRADO, NECESITARA DURANTE
SUS PRIMERAS 8 HORAS LA SIGUIENTE
EN UN PACIENTE CON ULCERA PEPTICA CANTIDAD DE SOLUCION (PARKLAND)
PERFORADA, DENTRO DE LA EF A LA 12,000ML
PERCUSIÓN ESPERARIA ENCONTRAR:
DESAPACRICION DE LA MATIDEZ HEPÁTICA MASCULINO DE 45ª QUEMADO POR
INTOXICACIÓN CON CO2 QUIEN PRESENTA
FEMENINO DE 45ª REFIERE LA PRESENCIA DE DISNEA Y QUEMADURAS EN CARA LAS
UNA MASA A NIVEL ABDOMINAL NO CUALES NO SON DOLOROSAS, EL PACIENTE
DOLOROSA. A LA EF SE ENCUANTRA MASA DEBERA HABER PRESENTADO POR LO
DE APROXIMIDAMENTE 2X2CM. POCO MENOS UNO DE LOS SIGUIENTES DATOS A LA
MOVIL, BLANDA NO DOLOROSA, ENTRE EXPLORACIÓN FÍSICA:
APÉNDICE, XIFOIDES Y CICATRIZ UMBILICAL CAPA CUBIERTA CON HOLLÍN NEGRO
EN LA LINEA MEDICA, CUAL SERIA LA
TERAPEUTICA:
PLASTIA FEMENINO 48ª QUE VA ASER SOMETIDA A
HISTERECTOMÍA POR PRESENTAR CACU LA
MASCULINO DE 60ª QUE PRESENTA UNA SIGUIENTE MEDIDA ANTITROMBOTICA ES:
BOLA EN REGION INGUINAL DERECHA QUE MEDIAS ELASTICAS
SE DESAPARECE CON EL REPOSO, NO
DOLOROSA QUE NO LLEGA AL ESCROTO, A FEMENINO 40ª CON DIAGNOSTICO DE
LA EF LA MASA TOCA EL PULPEJO DEL MIOMATOSIS UTERINA POR LO QUE SE VA A
DEDO EXPLORADOR A TRAVEZ DEL PISO PRACTICAR HITERECTOMIA TOTAL
INGUINAL EN LA REPARACIÓN QUIRÚRGICA ABDOMINAL, NO CUENTA CON
DE ESTE DEFECTO. CUAL ES LA SUTURA MAS ANTECEDENTES DE IMPORTANCIA, POR LO
ADECUADA: CUAL ESTARA CLASIFICADA CON UN RIESGO
PROLENE ANESTESICO QUIRÚRGICO CLASE:
I
MASCULINO DE 65ª EL CUAL PRESENTA
HERNIA INGUINAL DIRECTA. EL MANEJO DE MASCULINO DE 60ª EL CUALA VA A SER
ESTA CONDICION PUEDE HACERSE POR SOMETIDO A RESECCION TRANSURETRAL DE
CUALQUIERA DE LAS SIGUIENTES VIAS: PRÓSTATA. COMO ANTECEDENTE CUANTA
VIA RETROPERITONEAL CON INFARTO AL MIOCARDIO
VIA PERITONEAL HACE 4 MESES SE CATALOGA COMO RIESGO
VIA PERITROESCOPIA ANESTESICO QUIRÚRGICO CLASE:
VIA TRACTO ILEO PURICO IV

MASCULINO DE 67ª PROSTATICO, DIABÉTICO, FEMENINO DE 23ª LA CUAL SE ENCUANTRA


OBESO Y CON EPOG, QUIEN HACE UN MES EN URGENCIAS CON DIATROSIS, TA90/50,
EMPEZO A NOTAR MASA TUMORAL EN FR24X´, FC100X´, TEMP 38°, A LA CUAL SE LE
DIAGNOSTICO TROMBOSIS MESENTERICA, EN UN PACIENTE QUE SUFRE HERIDA POR
COMO ANTECEDENTE EL PACIENTE ES ARMA BLANCA EN TORAX IZQUIERDO. CON
DIABÉTICO DESDE HACE 10 AÑOS TA90/50. INGURGITACION YUGULAR POR LO
MANEJADO CON HIPOGLUCEMIANTES QU SE LE REALIZO PERICARDIOCENTESIS DE
ORALES SEGÚN LA CLASIFICACION DE ASA. URGENCIAS. ELTRATAMIENTO CONSISTE EN:
SU RIESGO QUIRÚRGICO SERA DE: TORACOTOMIA Y MIOCARDIOGRAMA
V
MASCULINO DE 30ª QUE SUFRE ACCIDENTE
MASCULINO DE 34ª EL CUAL SUFRIO AUTOMOVILISTICO POR LO QUE ES TRAIDO A
ACCIDENTE AUTOMOVILISTICO EN URGENCIAS, ENCONTRÁNDOSELE
CARRETERA. AL MOMENTO DE ARRIBAR SEMIINCONSCIENTE. HIPOTENSO,
CON LA AMBULANCIA USTED DETECTA QUE CIANOTICO. EN CUALLO INGURGITACION
EL PACIENTE SE ENCUENTRA YUGULAR CON GRACTURA DE ARCOS
SEMICONSCIENTE, SIGNOS VITALES: COSTALES EN HEMITORAX IZQUIERDO SE LE
TA110/70, FR90X´, FC88X´, CIANOSIS TOMA LA P.V.C. LA CUAL ESTA ELEVADA.
PERIFERICA , LESION MAXILOFACIAL CUAL ES EL DIAGNOSTICO:
IMPORTANTE, MOVIMIENTOS HEMOPERICARDIO
RESPIRATORIOS SUPERFICIALES Y LA
PRESENCIA DE FRACTURA EXPUESTA DE PACIENTE DE 28ª POLITRAUMATIZADO EN
TIBIA. CUAL ES LA CONDUCTA MAS ACCIDENTE POR MOTOCICLETA NO
IMPORTANTE QUE DEBE REALIZAR: LLEVANDO CASCO PROTECTOR ES ADMITIDO
CRICOTIROIDECTOMIA A URGENCIAS CON CIANOSIS Y SEVERA
DIFICULTAD RESPIRATORIA, TA80/40 Y
SANGRADO NASAL. ADEMÁS DE FRACTURA
MASCULINO DE 44ª EL CUAL SUFRIO EXPUESTA DE FÉMUR RUIDOS
ACCIDENTE AUTOMOVILISTICO AL RESPIRATORIOS AUSENTES EN HEMITORAX
MOMENTO DE ARRIBAR EN LA AMBULANCIA DERECHO. LA PRIORIDAD DEL MENEJO SERA:
USTED DETECTA QUE EL PACIENTE SE OBTENER ACCESO IV PARA TRANSFUSIÓN DE
ENCUENTRA SEMICONSCIENTE, CON EMERGENCIA DE SANGRE TIPO O
CIANOSIS PERIFERICA, LESION
MAXILOFACIAL IMPORTANTE, EN UN PACIENTE POLITRAUMATIZADO AL
MOVIMIENTOS RESPIRATORIOS APENAS CUAL EN LA EF SE REPORTA TA 100/60, FR32X,
PERCEPTIBLES Y LA PRESENCIA DE ABDOMEN DISTENDIDO. SILENCIO
FRACTURA EXPUESTA DE LA TIBIA ABDOMINAL , REBOTE (++), TIMPANISMO
Y PERONE. CUAL ES LA MEDIDA QUE GENERALIZADO. QUE ESTUDIO CORROBORA
APLICARIA: EL DIAGNOCTICO:
COLOCAION DE LA CANULA DE GEDELE TELE DE TORAX

PACIENTE QUE SUFRE ACCIDENTE


FEMENINO DE 27ª QUIEN A SUFRIDO CAIDA AUTOMOVILISTICO Y QUE AL EXAMEN SE
DE LESION DE BAZO, HIGADO, ASI COMO DE ENCUANTRA DESPIERTO Y RESPONDE A LAS
INTESTINO DELGADO, SE TRANSFUENDEN PREGUNTAS, A NIVEL DE TORAX SE
MAS DE 10 U DE SANGRE SI EL SANGRADO ENCUANTRAN DATOS QUE SUGIEREN
PERSISTIERA A PESAR DEL SUFIECINTE NEUMOTORAX A TENSIÓN EN HEMITORAX
REMPLAZO DE COAGULANTES SE PENSARIA DERECHO, LA MEDIDA A APLICAR EN ESTE
EN LA PRESENCIA ACTUAL DE: PROBLEMA SERA: TORACOCENTESIS EN 2°
COAGULACIÓN INTRAVASCULAR ESPACIO INTERCOSTAL
DISEMINADA
MASCULINO QUIEN EN SALA DE
OPERACIONES PREVIA A LA INTUBACIÓN,
FEMNINO DE 15ª QUIEN ES ASALTADA EN LA PRESENTA PARO CARDIORRESPIRATORIO SUS
VIA PUBLICA, A LA EF SE ENCUENTRA PULSOS CENTRALES SONDEBILES, SU TA
DISEÑA, INQUIETUD, DOLOR, FR40X 60/50. FRECUENCIA RESPIRATORIA
SILENCIO RESPIRATORIO Y CLARIDAD SUPERFICIAL, DADA LA BRADICARDIA DEL
PULMONAR AUMENTADA EN LA RX DE PACIENTE, CUAL ES EL MEDICAMENTO
TORAX EL DIAGNOSTICO ES: USADO PARA SU MANEJO:
NEUMOTORAX A TENSIÓN ADRENALINA ENDOTRAQUEAL

MASCULINO DE 29ª QUE CONDICIENDO SU MASCULINO DE 38ª QUIEN SE RECIBE EN


AUTOMOVILA ALTA VELOCIDAD SUFRE URGENCIAS CON ANTECEDENTES DE
CHOQUE, Y A LA EF SE ENCUANTRA CON TA LSIONES EN TORAX Y ABDOMEN NO
100/60, EC100X´, FR24X, EN EL RESTO DEL PENETRANTES. ASI COMO FRACTURAS
EXAMEN EN HEMITORAX DERECHO MULTIPLES EN EXTREMIDADES INFERIORES
MURMULLO VESIDULAR DESMINUIDO CON A SU LLEGADA TA 60/40 RUIDOS CARDIACOS
MATIDEZ A LA PERSECUCIÓN. CUAL SERIA EL POCO PERCEPTIBLES, RESPIRACIÓN
TRATAMIENTO TERAPÉUTICO PARA SU SUPERFICIAL, PULSOS EN FEMORAL Y EN
PROBLEMA PULMONAR: PUNCION CON CAROTIDA AUSENTES SE DIAGNOSTICA
TROCAR EN 6° ESPACIO INTERCOSTAL MENIOBRA DE COMPRESIÓN EXTERNA Y
DESFIBRILACION, PERSISTE CON ARRITMIAS
VENTRICULARES. CUAL ES LA DOSIS DE
LIDOCAINA INDICADA PARA EL PERCEPTIBLE, QUE DATO ES DETERMINANTE
TRATAMIENTO: PARA HACER EL DIAGNOSTICO:
BOLO IMG/ KG IV PERDIDA DEL PULSO EN UNA ARTERIA
CENTRAL (CAROTIDA O FEMORAL)
MASCULINO DE 50ª QUIEN INGRESA AL
SERVICIO DE URGENCIAS CON FEMENINO DE 42ª QUIEN SE LE REALIZO
ANTECEDENTE DE HABER SUFRIDO COLECISTECTOMIA ABIERTA POR
ACCIDENTE AUTOMOVILISTICO, A SU COLECISTITIS CRÓNICA LITIASICA AL SALIR
LLEGADA PRESENTA DESORIENTACIÓN, A SU DE QUIRÓFANO TIENE COLOCADA SONDA
LLEGADA PRESENTA DESORIENTACIÓN, TA NASOGASTRICA, SONDA VESICAL
50/40, FRECUENCIA CARDIACA NO AUDIBLE VENOCLISIS CON HARTMAN GLUCOSADA,
SIN RESPIRACIÓN ESPONTÁNEA, PULSOS EN RESPIRACIÓN DE 18X´, PULSO 75X´, TA 130/85,
CAROTIDAS AUSENTES. POR LO QUE SE ESTA PACIENTE DEBERA DE AMBULAR:
DIAGNOSTICA PARO AL DIA SIGUIENTE DE LA CIRUGÍA
CARDIORRESPIRATORIO. SE INICIAN
MANIOBRAS DE REANIMACION CON
COMPRESIONES TORAXICAS Y
RESPIRACIONES, EN EL MONITOR SE
OBSERVA FIBRILACION VENTRICULAR, CUAL PACIENTE DE 22ª ACUDE A URGENCIAS
ES EL SIGUIENTE PASO PAR CONTINUAR CON REFIRIENDO DOLOR EN FID SE DIAGNOSTICA
EL TRATAMIENTO: APENDICITIS Y SE REALIZA
INICIAR DESFIBRILACION E INTUBAR AL APENDICECTOMIA. EL HALLAZGO
PACIENTE QUIRÚRGICO FUE APENDICITIS EN FASE
(EDEMATOSA) 1, LO CITA PARA RETIROS DE
FEMENINO DE 55ª CON CHOQUE SÉPTICO Y PUNTOS A LOS 7 DIAS
DISFUNCIÓN ORGANICA MÚLTIPLE. A LA EF
PACIENTE SOMNOLIENTO, RUIDOS MASCULINO DE 22ª SOMETIDO A LA
CARDIACOS DISMINUIDOS, FRECUENCIA LAPAROTOMÍA CON HALLAZGO DE
RESPIRATORIA NO PERCEPTIBLE PULSOS PERFORACIÓN DEL ILEON DISTAL POR
CENTRALES AUSENTES, SE HACE SALMONELLOSIS, AL 3ER. DIA
DIAGNOSTICO DE PARO POSTOPERATORIO INICIA CON FIEBRE,
CARDIORRESPIRATORIO, SE INICIA MASAJE DOLOR, TUMEFACCIÓN, HIPEREMIA Y
CARDIACO EXTERNO LA META AUMENTO DE LA TEMPERATURA LOCAL, LA
FUNDAMENTAL EN TODA SESION DE CAUSA PROBABLE DE ESTA COMPLICAION
REANIMACION CON LA COMPRESIÓN ES: FALTA DE COLOCAION DE DRENES EN LA
ES: RESTAURAR LA FUNCION CARDIACA CAVIDAD
NORMAL LO MAS PRONTO POSIBLE
MASCULINO DE 30ª POSTOPERATORIO DE
FEMENINO DE 65ª QUIEN PRESENTA SEPSIS HERNIOPLASTIA INGUINAL DERECHA
ABDOMINALSECUNDARIA DIVERTICULITIS INDICADA EN EL 2° DIA POSTOPERATORIO
COLONICA COMPLICADA, SE REALIZA PRESENTA AUMENTO DE VOLUMEN DE LA
LAVADO Y DRENAJE DE LA CAVIDAD, EN EL HERIDA CON FLUCTUACIÓN, NO DOLOROSA,
POST OPERATORIO LA PACIENTE NO SE OBSERVAN DATOS DE CELULITIS NI
CARDIOPULMONAR LA CUAL ES EFECTIVA HIPERTERMIA CON ESTOS DATOS LA
USTED INDICA QUE LA PACIENTE ES COMPLICAION QUE PRESENTA EL PACIENTE
TRASLADADA A: SU CAMA CON CONTROL ES: DEHISCENCIA
ESRICTO DE SIGNOS VITALES
PACIENTE DE 50ª CON CANCER DE ESÓFAGO
FEMENINO DE 60ª QUIEN INGRESA A Y CON INANICIÓN EN ESTAS
URGENCIAS POR ACCIDENTE CIRCUNSTANCIAS SU CORAZON Y
AUTOMOVILISTICO AL LLEGAR AL HOSPITAL MUSCULOS TIENEN PREDILECCIÓN POR UNO
SUS SIGNOS VITALES SON: TA60/40, RUIDOS DE LOS SIGUIENTES AMINOÁCIDOS:
CARDIACOS POCO PERCEPTIBLES, DE CADENA RAMIFICADA
RESPIRACIÓN SUPERFICIAL, MINUTOS
DESPUÉS DE SU LLEGADA AL MASCULINO DE 36ª QUE DURANTE LA
SERVICIO DE URGENCIAS PRESENTA PARO INANICIÓN AGUDA DEBIDO A CANCER
CARDIACO RESPIRATORIO, CUAL ES LA LARINGEO TIENE UNA PERDIDA NETA DE
MEDIDA QUE INICIO PARA RESTAURAR LA AMINOÁCIDOS YA QUE LAS SÍNTESIS
FUNCION CARDIACA: AUMENTAR EL DE PROTEINA MUSCULAR DESAPARECE O
VOLUMEN SANGUÍNEO POR MEDIO DE DESCIENDE TANTO QUE EL CATABOLISMO
SOLUCIONES DE LA MISMA PERMANECE SIN CAMBIO
ESTA SITUACIÓN ES DEBIDA A:
FEMENINO DE 60ª QUIEN ES INGRESADA A AUMENTO DE GLUCAGON
CIRUGÍA PARA REALIZAR LAVADO DE
CAVUDAD POR SEPSIS PERITONEAL, EN EN UN PACIENTE POLITRAUMATIZADO A
QUIRÓFANO PRESENTA PARO QUIEN EN EL EXAMEN ENCONTRAMOS EN
CARDIORRESPIRATORIO A LA EXPLORACIÓN HEMOTÓRAX IZQUIERDO SILENCIO
RUIDOS CARDIACOS POCO AUDIBLES, RESPIRATORIO, A LA PERCUSIÓN
FRECUENCIA RESPIRATORIA NO TIMPANISMO Y CON CHOQUE DE PUNTA A LA
DERECHA. REALIZÁNDOSELE
TORACOCENTESIS DE URGENCIAS, LA ACOMPAÑADA DE HIPEREMIA Y NAUSEAS, A
JUSTIFICACIÓN DE ESTA MEDIDA ES POR: LA EXPLORACIÓN ROVSING (+) Y BLUNBERG
LA URGENCIA DE DESCOMPRIMIR AL (+). EN EL PUNTO DE MC BURNEY PARA EL
MEDIASTINO ASEO QUIRÚRGICO DE SUS MANOS, CUAL DE
LAS SIGUIENTES SUSTANCIAS
MASCULINO DE 34ª LLEVADO A URGENCIAS ANTISÉPTICAS UTILIZARA:
POR SUFRIR ACCIDENTE AUTOMOVILISTICO HI-BISCRUB (GLUCONATO DE
AL EXMAMEN ENCONTRAMOS CLORHEXIDINA)
HIPERSENSIBILIDAD LEVE EN EL ABDOMEN
Y HEMITORAX DERECHO. CON PIRISTALSIS EN EL PISO DE CIRUGÍA, SE DISCUTE EL
MUY DISMINUIDA Y AUSENCIA DE MATIDEZ CASO DE UN PACIENTE QUE PRESENTO
HEPÁTICA; CUAL ES SU IMPRESIÓN: DESHICENCIA DE LA HERIDA QUIRÚRGICA
RUPTURA DE VICERA HUECA EN EL 5° DIA DEL POSTOPERATORIO SE
HACE MENCIONAR QUE FUE DEBIDO A MALA
FEMNINO DE 22ª QUIEN VA A SER SOMETIDO TÉCNICA QUIRÚRGICA EN EL CIERRE DE LA
A LAPAROSCOPIA PÉLVICA POR PROBABLE HERIDA ESTA COMPLICACIÓN IMPLICA:
ENDOMETRIOSIS DURANTE LA ASEPSIA DEL SEPARACIÓN PARCIAL O TOTAL DE LOS
AREA QUIRÚRGICA (PIEL) CUAL DE LAS PLANOS DE LA HERIDA
SIGUIENTES SUSTANCIAS CONSIDERA QUE
ES LA MAS ADECUADA PARA RELIZAR ESTE FEMENINO DE 48ª QUIEN INGRESA A
PROCEDIMIENTO: URGENCIAS CON CUADRO DE ABDOMEN
YODOPOVINA (ISODINE) AGUDO SECUNDARIO A COLELITITIS
COMPLICADA. A LA EXPLORACIÓN
ENCUANTRA TALLA 1.60M PESO DE 92KG TA
FEMENINO DE 38ª QUE SE ENCUENTRA EN EL 140/90 DOLOR A LA PALPACIÓN EN
5TO DIA POSTOPERATORIO POR COLECTOMIA HIPOCONDRIO DERECHO. MURPHY (+)
TOTAL PARA EL TRATAMIENTO DE SU REBOTE POSITIVO, QUE INSICION SE
COLECISTITIS ULCERATIVA CRÓNICA RECOMIENDA: SUPRA INFRAUMBILICAL
INESPECÍFICA, LA CIRUGÍA SE EFECTUA SIN
COMPLICACIONES CERRANDO LAS CAPAS DE MASCULINO DE 28ª QUIEN FUE SOMETIDO A
LA HERIDA CON SURGENTE, COMO APENDICECTOMIA ENCONTRANDO UNA
ANTECEDENTES DE IMPORTANCIA APÉNDICE CECAL GANGRENADA. ESTE
MENCIONA SER DIABÉTICA TIPO 1, EN EL ANTECEDENTE NOS PERMITE CLASIFICAR LA
2DO DIA POSTOPERATORIO PRESENTO HERIDA COMO: SUCIA
ACUMULO DE LA SANGRE EN LA HERIDA Y
HOY OBSERVA EVICERACION.
LA CAUSA MAS PROBABLE DE LA MASCULINO DE 36ª SOMETIDO A
COMPLICACIÓN ES: HERNOPLASTIA CON RESECCION DE UN
INFECCIÓN DE LA HERIDA DIVERTICULO DE MECKEL A TRAVEZ
DEL ANILLO HERNIARIO EN EL
FEMENINO DE 17 AÑOS CON DIAGNOSTICO POSTOPERATORIO PRESENTO ABSCESO EN
DE PÚRPURA TROMBOCITOPENICA LA HERIDA QUIRÚRGICA EL MANEJO DE
IDIOPATICA REFRACTARIA A TRATAMIENTO ESTA COMPLICACION SE REALIZA CON
MEDICO, PROGRAMADA PARA DRENAJE Y CIERE: DIFERIDO
ESPLENECTOMIA. DESPUÉS DE LA ASEPSIA Y
ANTISEPSIA DEL AREA QUIRÚRGICA, LA MASCULINO DE 19ª CON APENDICITIS AGUDA
COLOCACIÓN DE CAMPOS Y SABANAS SE NO COMPLICADA CUAL ES LA INCISIÓN MÁS
INDICA CON: SABANA PODALICA CONVENIENTE PARA ESTE PACIENTE:
ROCKY DAVIS
FEMENINO DE 40ª QUIEN PRESENTA DOLOR
ABDOMINAL EN ABDOMEN SUPERIOR Y QUE CUAL ES LA HORMONA QUE DISMINUYE EN
SE LE EXACERVA CON LOS ALIMENTOS LA RESPUESTA METABÓLICA AL TRAUMA:
GRASOS. A LA EXPLORACIÓN MURPHY (+), INSULINA
EL ULTRASONIDO DE HIGADO Y VIAS
BILIARES REVELA VESÍCULA DE 11X9CM, E MASCULINO DE 53ª POLITRAUMATIZADO
IMÁGENES HIPERECOGENICAS EN SU POR ACCIDENTE AUTOMOVILISTICO EN ESTA
INTERIOR, PARA LA CIRUGÍA EN QUE AREA PACIENTE LA RESPUESTA METABÓLICA AL
SE VISTE CON LA ROPA QUIRÚRGICA: TRAUMA PROVOCA
NEGRA LIBERACIÓN DE ADRENALINA, LA QUE
PROVOCARA:
MASCULINO DE 71ª QUIEN FUE SOMETIDO A ESTIMULA LA LIPÓLISIS
COLECISTECTOMIA CON INCISIÓN MEDIA
SUPRAUMBILICAL EN EL 5TO DIA DEL PACIENTE POLITRAUMATIZADO EL CUAL
POSTOPERATORIO PRESENTO ENCUENTRA EN LA UCI DEVEMOS TENER EN
EVISCERACION. LA CAUSA MÁS PROBABLE CUENTA QUE DURANTE LA RESPUESTA
DE ESTA COMPLICACION ES: METABÓLICA AL TRAUMA LA ACCION
TÉCNICA QUIRÚRGICA INADECUADA DE LAS HORMONAS
CONTRARREGULADORAS TIENE
MASCULINO DE 22ª PRESENTA DOLOR COMO RESPUESTA EN EL ORGANISMO
ABDOMINAL EN FID CONTINUO, AGREDIDO:
AUMENTO DE LA GLUCONEOGENESIS COMPLICACIONES PULMONARES
POSTOPERATORIAS:
FEMENINO DE 28ª LA CUAL PRESENTO EJERCICIOS RESPIRATORIOS MINIMO 48
TRAUMA ABDOMINAL CERRADO CON HORAS ANTES DE LA CIRUGÍA
RUPTURA HEPÁTICA E ISQUEMIA
INTESTINAL POR DESGARRO DEL PACIENTE POLITRAUMATIZADO MANTENIDO
MESETERIO DURANTE UNA INTERVENCIÓN CON NUTRICION PALENTERAL TOTAL CON
QUIRÚRGICA, PRESENTA SANGRADO DIFUSO UNA FORMULA EQUILIBRADA, PERO QUE NO
DE LA PARED ABDOMINAL Y EN HIGADO ES SUFICIENTE PARA
DEBIDO A UN PROBABLE CID, EL CUBRIR SUS REQUERIMIENTOS
TRATAMIENTO QUE SE RELIZARA ENERGÉTICOS PARA INCREMENTAR EL
EN ESTE MOMENTO SERA: APORTE ENERGÉTICO SE CONSIDERA
EMPAQUETAMIENTO ABDOMINAL A: AUMENTAR LA FRACCIÓN DE GLUCOSA
DE LA FORMULA
PACIENTE EL CUAL FUE SOMETIDO A
HEMICOLECTOMIA POR CANCER DE COLON MASCULINO DE 28ª PROVENIENTE DE LA UCI
EN EL POSTOPERATORIO PRESENTA CUADRO DONDE FUE SOMETIDO A UNA
COMPATIBLE CON CID LA ALTERACIÓN LAPAROTOMÍA EXPLORADORA POR HABER
LABORATORIAL QUE ESPERARIA SUFRIDO ACCIDENTE AUTOMOVILISTICO
ENCONTRAR SERA: MENCIONÁNDOSE QUE SE LE REALIZO
ANTITROMBINA III DISEMINADA RESECCION INTESTINAL DE 40CM DE
INTESTINO DELGADO POR PRESENTAR
FEMENINO DE 28ª CON SEPSIS ABDOMINAL Y ISQUEMIA INTESTINAL SECUNDARIA A
A LA EXPLORACIÓN FÍSICA PACIENTE DESGARRO MESENTERICO POR
SOMNOLIENTA SIGNOS VITALES TA 80/60, FC DESACELERACION LOS RECEPTORES QUE
110X, EN EL CHOQUE SÉPTICO. LA ESTIMULAN LA LIBERACIÓN DE
HIPERVENTILIACION SE ENCUANTRA EN LA CORTICOTROPA EN ESTE PACIENTE SE
FASE: ENCUANTRAN LOCALIZADOS EN:
TEMPRANA PROTUBERANCIA Y MENSENCEFALO

FEMENINO DE 32ª QUIEN REFIERE UNA MASA ANTE UN PACIENTE QUE FUE SOMETIDO A
BLANDA, NO DEPRESIBLE, FIJA E INDOLORA DRENAJE ABIERTO DE ABSCESO HEPÁTICO
Y POR ARRIBA DE LA CICATRIZ UMBILICAL. AMIBIANO, EL CUAL SE ENCUANTRA EN LAS
EN ESTE CASO EL DEFECTO SE ENCUANTRA PRIMERAS HORAS POSTOPERATORIO FASE
EN LA: EBB, USTED ESPERARIA ENCONTRAR UN
LINEA ALBA AUMENTO DE LA SIGUIENTE SUSTANCIA
PRODUCIDA POR EL ORGANISMO. CUAL ES:
DURANTE UNA COLECISTECTOMIA ABIERTA TIROXINA
ACCIDENTALMENTE SE LE CAEN LAS
TIJERAS. LAS CUALES SON MASCULINO DE 35ª EL CUAL SE ENCUENTRA
IMPRESCINDIBLES PARA RELIZAR LA EN LA UCI CON DIAGNOSTICO DE
CIRUGÍA, NECESITAN QUE LAS TIJERAS SE PANCREATITIS AGUDA DESDE
ESTERILICEN QUE SUSTANCIA PERMITE HACE 10 DIAS ACTUALMENTE EL
REALIZAR ESTE PROCEDIMIENTO ORGANISMO TIENE: NECESIDAD DE
RAPIDAMENTE: CONSUMIR CETONAS
GLUTERHALDEHIDO ACTIVADO
*ANTE UN PACIENTE QUE EN EL
MASCULINO DE 18ª CON HERIDA PRODUCIDA POSTOPERATORIO CURSO CON
POR OBJETO CORTANTE EN REGION DESEQUILIBRIO HÍDRICO PODEMOS INTUIR
FRONTAL DE LA CARA DE 3CM APROX DE QUE EL AGUA INTRACELULAR PUEDE
LONGITUD, QUE TECNICA DE SUTURA DISMINUIR DEVIDO A UNA DE LAS
USARA PARA CERRAR LA PIEL Y TENER SIGUIENTES ALTERACIONES CUAL ES ESTA:
UN RESULTADO MAS ESTETICO: REDUCCIÓN DEL POTASIO INTRACELULAR
SUTURA SUBCUTICULAR

INGRESA PACIENTE A URGENCIAS CON


HERIDA EN LA CARA ANTERIOR DE FEMENINO DE 25 AÑOS QUIEN SUFRIO
ANTEBRAZO DE APROXIMADAMENTE 3CM, ACCIDENTE AUTOMOVILISTICO CON
DE LONGITUD, REALIZA EL ASEO DE LA LESIONES LEVES, PERO FUE VICTIMA DE
HERIDA DE LA PIEL CON ISODINE, CUAL UNA CRISIS EMOCIONAL, SE LE TOMA
ES EL TIEMPO MINIMO NECESARIO PARA GASOMETRIA REVELANDO UNA PCO2 DE
OBTENER EL EFECTO ESPERADO: 30MMHG, TAL CIFRA Y EL CUADRO CLINICO
10 MINUTOS SUGIEREN: ACIDOSIS RESPIRATORIA

PACIENTE DE 58ª PROGRAMADO PARA MASCULINO DE 35ª QUIEN ES RESCATADO DE


GASTRECTOMÍA SUBTOTAL POR CANCER ACCIDENTE AUTOMOVILISTICO Y QUE
GÁSTRICO EN ETAPA TEMPRANA COMO PRESENTA FRACTURAS MULTIPLES Y
ANTECEDENTE MECIONA PADECER EXPUESTAS EN MIEMBRO SUPERIOR
ENFERMEDAD PULMONAR. DE LAS DERECHO OCACIONANDO LESION ARTERIAL.
SIGUIENTES RESPUESTAS CUAL EVITA LAS AL INICIARSE COMO PRIMER EVENTO LA
VASOCONSTRICCIÓN SE ACTIVA EL
SIGUIENTE ELEMENTO:
MICROAGREGADO PLAQUETARIO
3er Banco

1) ¿Cual de los siguientes antibióticos pueden alcanzar


concentraciones inhibitorias en los abscesos abdominales?
a) Gentamicina
b) Metronidazol
c) Ceftriaxona
d) Tetraciclina
e) Penicilina G sodica cristalina
2) * Masculino de 34 años con estreñimiento de más de 5
años, dos días con dolor intenso al evacuar, sangrado rectal
en goteo post evacuación, dolor al sentarse. Hemoglobina
12 grs. Hematocrito 38%, leucocitos de 6 mil, TP 12”, TPT
34”. El tratamiento quirúrgico consiste en:
a) Fistulectomia
b) Drenaje a cielo abierto
c) Hemorroidectomia
d) Fisurectomia más esfinterotomia lateral
e) Cauterización de la herida
3) Femenina de 36 años, politraumatizada, sometida a
laparotomía exploradora por hemoperitoneo. Al segundo
dia postoperatorio presenta datos de insuficiencia renal
aguda. ¿Cuál de las siguientes define esta complicación?
a) Hipercalcemia
b) Eritrocituria
c) Sobrecarga de volumen
d) Azotemia
e) Piuria
4) Femenina de 48 años de edad a quien se realizó resección
pancreatoduodenal con tecnica de whipple por carcinoma
de la cabeza del páncreas. Al segundo dia postoperatorio se
encuentra en malas condiciones debido a una reposición de
líquidos insuficientes. ¿Cuál de los siguientes datos
bioquímicos se relaciona con el deficit hidrico de esta
paciente?
a) Hipoalbuminemia
b) Hiperazoemia
c) Hipocalcemia
d) Isonatremia
e) Hipoglucemia
5) Paciente post iliostomia, con gasto de 2.5L/24hr. Paciente
refiere sed intensa, astenia y adinamia, desubicado en
tiempo y espacio, volumen urinario por hora de 15 ml.
Laboratorio: Glucosa 120, Urea 80, creatinina 1.8, Na 127,
cloro 85, potasio 2. Signos vitales: TA 90/50, FC 125, FR
22, temp. 37 grados. Que tipo de transtornos
hidroelectrolitricos presenta este paciernte?
a) Transtorno de PH y de la osmolaridad
b) Transtorno obstructivo y de formacion del tercer
espacio
c) Transtornos de volumen, concentracion y composicion
d) Transtornos de distribucion e intercambio
e) Transtorno de control renal de electrolitos
6) * Dato radiologico que hace poner en duda el diagnóstico
de obstrucción mecánica del intestino?
a) Neumoperitoneo
b) Dilatación del intestino delgado
c) Dilatación del intestino grueso
d) Niveles hidro-aéreos
e) Gas en ampolla rectal
7) * Masculino de 40 años, bombero, presenta quemaduras,
sin compromiso cardio-respiratorio, lesiones en cara ant.
De tórax, extremidades, superior e inf, indoloras, duras, no
palidecen a la presión, afecta menos del 50% de la
superficie. ¿Qué solución se le indica para la reanimación
agresiva de líquidos?
a) Solucion Mixta
b) Fisiologica
c) Ringer Lactato
d) Glucosada
8) * Paciente con hernia en pared abdominal post incisional
de 8cm de longitud, la mejor opción de tratamiento
quirúrgico es:
a) Cierre primario de la aponeurosis con sutura no
absorbible
b) Desplazamiento de injerto Moo cutáneo
c) Colocar malla sustituyendo el peritoneo
d) Cierre primario de la aponeurosis con sutura
absorbible
e) Colocación de malla pre peritoneal
9) Masculino 66 años 30 cigarros diarios durante 25 años.
DM hace 3 años tratado con hipoglucemiantes orales. PA: 8
meses de evolución con pérdida de peso cuantificada en
11kg, anorexia, debilidad generalizada, dolor en epigastrio
irradiado a hipocondrio derecho, ictericia en últimos 2
meses al igual que colitis, acolia, y prurito generalizado. Se
palpa hepatomegalia, vesícula biliar aumentada de tamaño
con diámetros 9x6 cm. Bilirrubina total 28, bilirrubina
directa 25.2, fosfatasa alcalina 750. Hallazgos radiológicos
esperados con el paciente:
a) Dilatación de la vía biliar
b) Colecistitis aguda litiasica
c) Hepatomegalia sin dilatación de las vías biliares
d) Quiste hepático gigante
e) Colecistitis crónica litiásica
10) Femenina de 65 años hipertensa de más de 20 años cifras
de 160/100, de le programó una colecistectomia, ¿Qué
riesgo quirúrgico presenta?
a) 1
b) 2
c) 3
d) 4
e) 5

11) Femenina 37 años se calló de un caballo, paciente


parapléjica, con defecto de sensibilidad en metamera T10.
Signos vitales: TA: 80/50, FC: 130, FR: 22, Temp. 37°C.
Presenta piloereccion distal desde la metamera T8. ¿Que
tipo de shock presenta este paciente?
a) Anafilactico
b) Séptico
c) Neurogenico
d) Hipovolemico
e) Cardiogenico
12) Niño recién nacido en quien la madre notó, en ambas
regiones inguinales, una masa que aumenta de tamaño
cuando el bebé llora o puja. El médico del hospital comenta
que se trata de 2 hernias. ¿Cuál es el defecto congénito que
predispone a la formación de estas hernias?
a) Defecto de los pilares de anillo inguinal superficial
b) Defecto de fascia transversalis
c) Defecto de los pilares del anillo profundo
d) Permeabilidad del proceso peritoneo-vaginal
e) Defecto del mecanismo obturador del anillo inguinal
profundo
13) Paciente masculino el cual tiene varias horas de presentar
cuadro de oclusión intestinal baja actualmente con vomitos
fecaloides, ¿cuál es el mecanismo relacionado con las
caracteristicas de los vomitos?
a) La colonización del intestino delgado
b) La materia fecal tiene movimiento retrogrado
c) La oclusión es funcional
d) Existe antiperistalsis
e) Se relaciona con la perstalsis
14) Paciente de 25 años acude por quemaduras, ampollas y
Eritema en cara, extremidad superior izq y pecho.
carbonización franca circunferencial de extremidad
superior derecha con Llenado capilar reducido. ¿Cuál es el
manejo inicial más apropiado?
a) Excisión de todas las heridas de 3er grado
b) Escarotomía de la mano derecha
c) excisión de quemaduras de mano y cara
d) Antibióticos tópicos
e) injertos de piel de espesor parcial sobre las áreas que
presentan quemaduras de 3er grado
15) Paciente adulto con compromiso de vía aérea, imposible
ventilar y se requiere de emergencia ventilar al paciente.
¿Cual es posible por punción percutánea?
a) Cricotomia simple
b) Cricotiroidostomia
c) Traqueostomia
d) Intubacion
e) Traqueotomia
16) Paciente 22 años se cayo de cuarto piso, se realizó
hepatectomía parcial y resección de 30 cm del intestino
delgado, 15 unidades de paquete globular en el pre y
transoperatorio, 5 litros de solución Hartmann y 4 unidades
de plasma fresco congelado. Inicia con sangrado difuso de
la cavidad abdominal durante el cierre de la laparotomía.
¿Cuál es la causa más probable?
a) Insuficiencia hepática
b) Hiperesplenismo
c) Hipoprotombinemia
d) Deficiencia del factor IX
e) Plaquetopenia
17) Femenina de 38 años, operada de colecistectomía debido
a microlitiasis vesicular, actualmente con dolor en
epigastrio con ictericia, lab con fosfatasa alcalina 350. Que
patología se sospecha?
a) Hepatitis
b) Pancreatitis aguda
c) Úlcera péptica
d) Absceso Residual
e) Hematoma
18) Paciente con con trauma múltiple, multi transfundido con
10 unidades de paquete globular. Se realizó Hepatectomía
parcial y esplenectomía. Inicia con sangrado durante el
cierre, de consideró asociado a transfusión masiva, ¿cual
estudio apoya esta probabilidad?
a) Determinación de factor IX
b) Cuerpos de Howell-Joly
c) Pruebas de función hepatica
d) Cuento de plaquetas
e) Biopsia de médula ósea
19) Masculino 43 años se fractura ambos fémures en
accidente automovilístico. Se encuentra con Alteración del
estado de alerta, piel fría, húmeda y de aspecto marmóreo.
SV: TA 80/40 Fc 120, FR 28, temp. 35.6°C. ¿Cual explica
el evento fisiológico presente en este paciente?
a) Acúmulo de ácido volátil y desviación alcali del
bicarbonato
b) Disminución de la perfusion en cerebro y corazón
c) Apertura de esfínteres precapilares y de vasos de
capacitancia
d) Incremento del efecto de la hormona reguladora
e) Hipoperfusion fusilar distemica

20) Paciente con pérdida de estado de alerta, sufrió lesión en


región inguinal izq. Por proyectil de arma de fuego con
sangrado en orificio. Conducta que se debe adoptar:
a) Control de hemorragia
b) Continuar con ABC e ignorar hemorragia
c) Compresión directa y continuar con ABC
d) Aplicar torniquete controlado proximal y continuar
con ABC
e) Pinzado hemostático y continuar ABC

21) Paciente de 49 años de edad, con choque séptico


secundario a perforación intestinal, se le aporta solución
ringer-lactato (Hartmann) y de Han iniciado vasopresores.
Se agregó dificultad respiratoria y confusión mental. ¿Qué
complicación presenta?
a) Sepsis severa
b) Síndrome de respuesta inflamatoria sistémica
c) Hepatitis reactiva
d) Transtorno vascular cerebral agudo
e) Choque séptico refractario

22) Mejor opción de tipo de sondaje debido a sus


características y tiempo de duración para preparación
gastrica de un paciente el cual será sometido a endoscopio
diagnóstico-terapéutica por datos de retención gastrica
condicionando un megacolon
a) Nelaton
b) Rush
c) Levin
d) Foley-Alcock
e) Robisson

23) Femenina 42 años, colecistectomía abierta es neumopata,


nutrición normal. ¿Qué sutura se utiliza?
a) Catgut simple 1
b) Seda 3/0
c) Vycril 3/0
d) Catgut crómico 1
e) Polipropileno 1

24) Paciente masculino de 60 años, quien se realizó


laparotomía exploradora previa colocación de catéter
central. A los dos días del postoperatorio, durante la
deambulación presenta disnea e insuficiencia respiratoria,
en la placa de tórax se observa una zona de condensación
pulmonar basal derecha datos que se encuentran en relación
con que complicación pulmonar.
a) Derrame pleural
b) infarto pulmonar.
c) neumotorax
d) neumonia
e) Tromboembolia pulmonar.

25) Usted acude al servicio de urgencias a valorar un paciente


gue presenta cuadro de oclusión intestinal como
antecedente tiene dos cirugías previas cuál será la causa
más probable?
a) Trombosis arterial
b) Adherencias
c) Hernia Ventral
d) Neoplasia de colon

26) femenino de 45 años de edad sometida a histerectomía de


urgencia por sangrado persistente. Se encontró necrosis de
miomas uterinos. Ha recibido la transfusión de 5 unidades
de paquete globular y 5 unidades de plasma fresco. En el
tansopeatoro aparecen datos de sangrado en caps por lo que
se piensa en CID. Cual de los siguientes apoya este
diagnóstico
a) Presencia de productos de lidia de finrinogeno
b) Disminución del factor de Hageman
c) Presencia de reticulocitos
d) Antitrombina III disminuida
e) Crenocitos abundantes en orina

27) 3 year old boy spilled bleach onto lower extremities.


Diagnosed with chemical burn, all clothes are removed, in
addition to resuscitation, which of the following is the most
appropriate initial management
a) Neutralize the burn wound with weak acids
b) Treatment of the burn wound with calcium gluconate
gel
c) Wound debridement in the operating room
d) Washing of the burn wound with large volumes of
water
e) Treatment of the burn wound with anti microbial
agents

28) Una mujer de 78 años acude a Urgencias por dolor en fosa


iliaca izquierda de 24 horas de evolución asociado a fiebre
y algún vómito ocasional. A la exploración destaca dolor a
la palpación de forma selectiva en la fosa iliaca izquierda
con sensación de ocupación, defensa y descompresión
positiva. Ante la sospecha de diverticulitis aguda ¿cuál de
las siguientes afirmaciones es correcta
a) Si se produjera una peritonitis generalizada, la técnica
quinủrgica más adecuada es la práctica de una
colostomía derivativa sin resección del segmento
sigmoideo afectado
b) En caso de precisar intervención quirúrgica tras
solucionarse el episodio agudo, el abordaje
laparoscópico está contraindicado.
c) En caso de absceso pélvico contenido está indicada la
colocación de un drenaje percutáneo guiado con TAC
o ECO.
d) La exploración complementaria más segura y de
mejor rendimiento es el enema con contraste baritado
e) En caso de diverticulitis aguda no complicada está
indicada la sigmoidectomía efectiva tras la curación
del primer episodio agudo.

29) El médico de emergencias le pide evaluar a una paciente


de 46 años de edad, con dolor abdominal en el cuadrante
superior derecho, debe insistirse en la descripción del dolor,
para definir la sospecha diagnóstica ¿Cuál de los siguientes
enunciados concuerda con el cuadro clínico de esta
paciente?
a) Si el dolor se inicia después de la ingesta de alimentos,
se trata de úlcera perforada.
b) si el dolor se irradia a la región inguinal se trata de
cálculo ureteral.
c) si el dolor es postprandial e irradiado a hombro
derecho, es colecistitis
d) Si el dolor se irradia al hombro izquierdo, se trata de
colecistitis.
e) Si El dolor disminuye con el vómito, es una
pancreatitis

30) A35-year-old woman undergoes an elective laparoscopic


cholecystectomy for symptomatic cholelithiasis. Which of
the following wound classes best describes her procedure?
a) Class II, Clean/contaminated
b) Class IV, Dirty
c) Class I Clean
d) Cass III, Contaminated
e) None of the above
31) Paciente masculino de 18 años que consulta por haber
notado una masa en región inguinal derecha, la cual
apareció en forma espontánea. Al exámen físico, esa masa
puede reducirse con maniobras manuales cuidadosas,y al
introducir el dedo explorador al anillo inguinal superficial.
se percibe el contacto del saco herniario con la maniobra de
Valsalva. A qué tipo de hernia Corresponde?
a) Hernia en pantalon
b) Hernia inguinal indirecta
c) Hernia Femoral
d) Hernia de Spigel
e) Hernia inguinal directa

32) Femenino de 53 años de edad, con un peso de 70 kgs. En


estado de choque hipovolémico, por diarrea coleriforme. Se
ha iniciado la fluidoterapia, y para vigilar su respuesta se ha
colocado una sonda urinaria. Para considerar que el riñón
tenga una perfusión sanguínea adecuada. ¿ Cuánto debe de
tener de gasto urinario ?
a) 10 a 20 mi/hr
b) 35 a 70 m/hr
c) Mas de 100 m/hr
d) 15 a 30 m/hr
e) 90 a 100 mi/hr
33) Masculino de 68 años de edad, el cual fue sometido a
resección abdominoperineal por presentar cáncer de recto,
durante la cirugía las pérdidas sanguíneas fueron repuestas
volumen a volumen. En el postoperatorio fue manejado con
soluciones glucosadas al 5% presentando debilidad,
letargia, convulsiones y posteriormente coma. Por lo
referido, el paciente está cursando con cual problema
electrolítico?
a) Hiponatremia.
b) Hiperglucemia.
c) Hiperpotasemia.
d) Hipercalcemia.
e) Hipopotasemia
34) Paciente femenino de 48 años de edad que será sometida a
histerectomía por presentar metrorragia recurrente por
hiperplasia endometrial. Será intervenida una vez que
cuente con valoración por cardiólogo ya que además es
portadora de Hipertensión Arterial mal controlada. ¿Qué
estudios son indispensables para su evaluación cardiológica
preoperatoria?
a) CPK fracción MB, TGO Y Troponina
b) Prueba de esfuerzo en banda sin fin y EKG
c) Monitoreo tipo Holter
d) Ecocardiograma bajo estrés con dobutamina
e) Radiografia de torax y EKG

35) Ingresan dos pacientes varones de 28 y 25 años de edad,


quienes sufrieron volcadura Cursan segundo día de estancia
en Unidad de Cuidados Intensivos, actualmente se
encuentran en ayuno y sin necesidad de vasopresores, con
diuresis adecuada. Llama la atención que sin ser diabéticos,
mantienen cifras altas de glucemia ¿Cuál de las siguientes
sustancias promueve la gluconeogénesis evidente en estos
pacientes?
a) Hormona lut
b) Luteinizante.
c) Arelina y leptina.
d) Cortisol.
e) Hormona folículo estimulante
f) Insulina.

36) Paciente masculino de 52 años de edad, alcohólico


crónico, presenta gran distensión abdominal con anasarca y
dificultad respiratoria, su gasometría revela un PH. 7.44,
PaCO2: 28 mmHg. PaO2: 80 mmHg y HCO3: 28. ¿Cuál es
el trastorno ácido-base primario en este paciente?
a) Alcalosis mixta.
b) Alcalosis respiratoria compensada
c) Acidosis respiratoria.
d) Acidösis metabólica.
e) Alcalosis metabolica.
37) Durante reanimación de paciente con hernia penetrante de
abdomen, por proyectil por arma de fuego, que presenta
choque hipovolemico. ¿Cual de las siguientes soluciones
es adecuada para iniciar reanimación?
a) Dextran 40
b) Cloruro de sodio al 0.9%
c) Hemacel
d) Plasma fresco
e) Hartman
38) Femenino de 58 años, post colecistomia abierta con
exploración de vías biliares, por colecistitis crónica litiasica
más coledocolutiasis. La intervención quirúrgica duró 4
horas, cursando con hipotension transoperatoria asociada a
sangrado y medicamentos. Se le administran 500 ml de
solución Hartmann y dos horas después la paciente sólo ha
orinado 50 ml. ¿Cual es la Fisiopatología?
a) Respuesta patológica por falta de líquidos
b) Respuesta fisiológica por vasopresina
c) Obstrucción de la sonda urinaria
d) Respuesta fisiológica por aldosterona
e) Respuesta fisiológica a la anemia

1.- cual es la prueba complementaria MENOS útil en el


diagnóstico de la obstrucción intestinal de las citadas?
a) Enema opaco
b) Rx de torax
c) Rx simple de abdomen
d) Eco abdominal
e) Rx de abdomen en bipedestación

2.- ¿cuál es el esteroide mas abundante y con mayor importancia


fisiológica en el organismo?
a) Progesterona
b) Prednisona
c) Corticosterona
d) Cortisol
e) Aldosterona

3.- paciente de sexo femenino de 39 años con antecedente de


funduplicatura laparoscópica hace 4 dias, reintervención por
perforación de fundus gástrico hace36 hrs; presenta en las
últimas 8 horas hipoperfusión tisular marcada, choque
hipovolémico grave, FC 132lpm, disminución del retorno
venoso, anuria, hipoxia severa, hipercarbia, incremento del pico
inspiratorio de ventilador y la presión intravesical de 38 mmHg.
El diagnostico mas probable es
a) sx de isquemia aguda mesentérica
b) sangrado post operatorio
c) choque séptico
d) sx de compartimiento abdominal
e) sx de respuesta inflamatoria sistémica

4.- se trata de un paciente el cual se encuentra con


manifestaciones clínicas de hiponatremia por intoxicación
hídrica usted decide instalar solución salina al 3% ¿Cuál es la
metodología adecuada para evitar la melanosis pontina?
a) Primero se pasará una carga Hartman de 20 ml por kg
b) Corrección de sodio hasta 130 meq en una hora
c) No deberá suspenderse la solución glucosada
d) No deberá pasar de 1 a 2 meq por hora
e) Se agrega solución de ringer

5.- femenina de 27 años operada de colecistectomía y al día


siguiente presenta por drenaje salida de material biliar en poca
cantidad si se considera que la causa es una anomalía anatómica
¿Cuál sería la causa?
a) Ligadura inadecuada del cístico
b) Hepático anómala
c) Presencia de conductos de luschka
d) Lesión hepático-derecha
e) Lesión de colédoco

6.- paciente masculino de 65 años, fue sometido a


hemicolectomía izquierda por adenocarcinoma de colon, con
anastomosis primaria, para continuar con vigilancia deberá de
tener seguimiento ¿Cuál de los siguientes es el marcador
indicado?
a) GC
b) ACE
c) CA 125
d) AFP
e) AP

7-. Cuando un paciente es portador de esofagitis por reflujo es


importante dar tratamiento para corregir dicha alteración, ya que
puede en un año tener el 1% de posibilidades de presentar la
siguiente alteración
a) Epitelio eritematoso
b) Epitelio metaplásico
c) Erosiones
d) Epitelio displásico
e) Estenosis
4to Banco

CIRUGÍA. PREGUNTAS PRIMER PARCIAL


33. After complete removal of a sessile polyp of 2.0 cm by 1.5
cm found 1 finger length above the anal mucocutaneous margin,
the pathologist reports it to have been a villous adenoma that
contained carcinoma in situ. Which of the following is the most
appropriate next step in management?
a. Rescission of the biopsy site with wider margins
b. No further therapy
c. Anterior resection of the rectum
d. External radiation therapy to the rectum
e. Abdominoperineal rectosigmoid resection

32. A 42-year-old man has bouts of intermittent crampy


abdominal pain and rectal bleeding. Colonoscopy is performed
and demonstrates multiple hamartomatous polyps. The patient is
successfully treated by removing as many polyps as possible
with the aid of intraoperative endoscopy and polypectomy.
Which of the following is the most likely diagnosis?
a. Peutz-Jeghers syndrome
b. Villous adenomas
c. Familial polyposis
d. Ulcerative colitis
e. Crohn colitis

31. An 80-year-old man with history of symptomatic


cholelithiasis presents with signs and symptoms of a small-
bowel obstruction. Which of the following findings would
provide the most help in ascertaining the diagnosis?
a. A leukocyte count of 40,000/ml
b. Coffee-ground aspirate from the stomach
c. A pH of 7.5, PCO2 of 50 kPa, and paradoxically acid urine
d. A palpable mass in the pelvis
e. Pneumobilia

12. Paciente hombre con sintomatología esofágica y


extraesofágica, antecedentes de ERGE, se presenta con dolor
abdominal agudo, intenso localizado en región del epigastrio
con irradiación generalizada, tiene rebote positivo,
bioquímicamente tiene leucocitosis con desviación hacia la
izquierda (neutrofilia), por lo que se integra abdomen
intraperitoneal quirúrgico. Se decide realizar laparoscopía
diagnóstica, pero usted recuerda que el procedimiento de
laparoscopía se considera estándar de oro para:
a. Apendicectomía
b. Colecistectomía
c. Gastrectomía
d. Esofagectomía
e. Funduplicatura

26. Al estado crónico del absceso perineal se le conoce cómo?


a. Síndrome hemorroidal
b. Fisura
c. Fístula
d. Absceso perineal
e. Incontinencia anal

11. Paciente masculino de 56 años de edad con manifestaciones


clínicas de sangrado rectal con evolución de seis semanas, sin
pérdida de peso. ¿Cuál es el estudio más apropiado para el
diagnóstico?
a. Radiografía simple de abdomen de pie y en decúbito y tele de
tórax
b. Colon por enema con doble contraste
c. Resonancia magnética nuclear con gadolinio
d. Colonoscopia
e. TAC abdominal virtual

22. Dentro del manejo de los pacientes quirúrgicos, el equilibrio


hidroelectrolítico es esencial para la pronta recuperación. El
proceso inflamatorio que impone la enfermedad y el manejo
quirúrgico influyen directamente en la movilización de los
líquidos corporales. Cuando estas pérdidas por distribución son
importantes, el paciente puede desarrollar hipovolemia. ¿Cuál
sería el trastorno de líquidos más común de este problema
hipovolémico?
a. Pleuresia por trasudado
b. Deficiencia de líquido extracelular
c. Intersticio patológico
d. Ascitis
e. Infiltrado acuoso

2. Se trata de paciente el cual acude a consulta por sensación de


cansancio, dolor, edema vespertino y limitación de los arcos de
movimiento con presencia de tortuosidades de ambas
extremidades inferiores, así como hiperpigmentación. Entre los
signos que presenta el paciente uno de estos corresponde a signo
de afección grave:
a. Dolor
b. Limitación de los arcos de movimiento
c. Edema
d. Hiperpigmentación
e. Venas tortuosas

29. ¿Cuál de los siguientes antibióticos puede alcanzar


concentraciones inhibitorias en los abscesos abdominales?
a. Gentamicina
b. Metronidazol
c. Tetraciclina
d. Ceftriaxona
e. Penicilina G sódica cristalina

24. En el área de hospitalización de cirugía hay 2 pacientes con


antecedente de transfusión y reanimación con líquidos por
diversos grados de choque hipovolémico. Hacia el quinto día los
electrolitos séricos son normales. Cada paciente muestra un
aumento en la diuresis llegando hasta los 100 ml/h; todos niegan
sed, los signos vitales son normales estables. Por lo anterior los
médicos restringen el aporte parenteral de cristaloides. ¿Por qué
se debe restringir el aporte de líquidos en estos pacientes?
a. Porque la hiperglucemia de la respuesta al trauma aumenta la
poliuria
b. Porque la pérdida de sodio del espacio intracelular puede
disminuirse
c. Porque no es necesario aportar el requerimiento basal de
glucosa
d. Porque la falla prerrenal puede mejorarse con esta medida
e. Porque durante la redistribución de líquidos aumenta el
flujo intravascular de retorno

27. Es uno de los mediadores más tempranos y potentes de la


respuesta ---
a. Interferón Gamma
b. Serotonina
c. IL-1
d. TNF alfa 2
e. Histamina

9. Paciente hombre de 18 años con dolor abdominal agudo de 8


horas de evolución el cual refiere signos y síntomas compatibles
con apendicitis aguda no complicada y no modificada, con una
escala de Alvarado de 8 puntos. Acorde a lo mencionado en el
libro de texto Schwartz décima edición. Con respecto al
diagnóstico de apendicitis aguda, una de las siguientes NO es
correcta:
a. Disminución de la concentración de proteína C reactiva y
fiebre
b. Dolor en flanco derecho
c. Respuesta inflamatoria sistémica con leucocitosis y
neutropenia
d. Síntomas gastrointestinales que comienzan después del inicio
del dolor
e. Síntomas gastrointestinales que comienzan antes del inicio del
dolor

15. Paciente mujer de 36 años, politraumatizada, sometida a


laparotomía exploradora por hemoperitoneo. En el segundo día
de postoperatorio manifiesta datos de insuficiencia renal aguda.
¿Cuál de las siguientes opciones define esta complicación?
a. Sobrecarga de volumen
b. Eritrocituria
c. Hipercalcemia
d. Azotemia
e. Piuria

13. Paciente mujer de 23 años la cual presenta dos situaciones


clínicas a considerar, cursa con embarazo y tiene colelitiasis la
cual ha condicionado estados frecuentes de cólico biliar, acude
con usted para su programación quirúrgica. Acorde a lo
establecido en las recomendaciones internacionales y el libro de
texto recomiendan que la colecistectomía se realice en el:
a. Primeros dos meses de embarazo
b. Últimos dos meses de embarazo
c. Tercer trimestre de embarazo
d. Segundo trimestre de embarazo
e. Primer trimestre de embarazo
19. Paciente mujer de 48 años de edad a quien se realizó
resección pancreatoduodenal con tecina de Whipple por
carcinoma de la cabeza del páncreas. Actualmente cursa el
segundo día de postoperatorio y debido a una reposición
insuficiente de líquidos se encuentra en malas condiciones
generales. ¿Cuál de los siguientes datos bioquímicos se
relaciona con el déficit hídrico de esta paciente?
a. Hipoglucemia
b. Hipoalbuminemia
c. Hipocalcemia
d. Isonatremia
e. Hiperazoemia

25. Masculino de 34 años con estreñimiento crónico de más de 5


años, dos días con dolor intenso al evacuar, así como sangrado
rectal en goteo post evaluación, dolor al sentarse. Hemoglobina
de 12 grs. Hematocrito de 38%, leucocitos de 6000, TP 12”,
TPT 34”. El tratamiento quirúrgico del caso consiste en:
a. Fistulectomia
b. Drenaje a cielo abierto
c. Hemorroidectomía
d. Cauterización de la herida
e. Fistulectomia más esfinterotomia lateral

21. Paciente hombre de 30 años de edad, cursa el sexto día de


postoperatorio por resección intestinal secundaria a perforación
traumática, periodo durante el cual ha permanecido en ayuno. El
día de ayer comenzó a fugar liquido intestinal en abundante
cantidad y se desarrolló síndrome de respuesta inflamatoria
sistémica. Signos vitales: TA 110/70 mmHg, FC 95 lpm, FR 16
rpm y temperatura 39°C. ¿Qué signos y síntomas específicos
debe considerar para clasificar el grado de déficit de agua en
este paciente?
a. Análisis clínico, análisis de laboratorio, calculo y prescripción
b. Presencia de sed, pérdidas aumentadas y signos de perfusión
c. Presencia de sed, signos ortostáticos y signos de
hipoperfusión
d. Presencia de sed e hipertermia asociada a SIRS
e. Presencia de sed, pérdidas aumentadas y tipo de material de
fuga intestinal

20. Usted evalúa a un paciente postoperado, a quien se le realizo


una ileostomía, el gasto fue de 2.5 L/24 hrs. El paciente se
refiere con sed intensa, astenia y adinamia, se encuentra
desubicado en tiempo y espacio y su volumen urinario por hora
de 15 ml. El laboratorio actual reporta; Glucosa 120 mg/dl, Urea
80, Creatinina 1.8, Na 127, cloro 85, Potasio 2. Los signos
vitales muestran TA 90/50 mmHg, FC 125 lpm, FR 22 rpm,
temperatura 37°C. ¿Qué tipo de trastornos hidroelectrolíticos
presenta este paciente?
a. Trastornos del control renal de electrolitos
b. Trastornos de volumen, concentración y composición
c. Trastorno de pH y de la osmolaridad
d. Trastornos de distribución e intercambio
e. Trastorno obstructivo y de formación del tercer espacio

10. Femenino de 23 años de edad la cual presenta dolor


abdominal de 48 hrs de evolución, periumbilical que se irradia
hacia fosa iliaca derecha, a la exploración encuentra signos
apendiculares positivos. Leucocitosis de 16,000 cel/mm3, a la
revisión por el servicio de cirugía la paciente refiere mejoría
significativa de los síntomas. ¿Cuál sería la explicación más
probable al estado actual del paciente?
a. Perforación apendicular
b. Colitis
c. Torsión ovárica
d. Embarazo ectópico
e. Enfermedad pélvica inflamatoria
30. Femenina de 38 años, la cual sufrió accidente
automovilístico (choque frontal) hace 4 horas, es rescatada por
paramédicos y llevada al hospital, refiere solamente dolor
abdominal difuso. Se encuentra consciente, orientada,
intranquila con integridad neurológica y motora, tórax con
buena movilidad y ventilación adecuada, el abdomen con
distension y dolor difuso con resistencia muscular voluntaria,
ruidos peristálticos disminuidos. Frecuencia cardiaca de 105
lpm, TA 90/50 mmHg, hematocrito de 32%, hemoglobina de 10
mg/dL. Es el mejor parámetro para evaluar de manera indirecta
perfusión tisular.
a. Oximetría de pulso
b. Gasto urinario
c. Tensión arterial
d. Presión venosa central
e. Llenado capilar

3. Paciente hombre de 72 años el cual acude por claudicación


intermitente de la extremidad inferior izquierda de 20 metros de
distancia con inicio desde hace 3 meses de evolución, con inicio
insidioso y crecimiento paulatino, como antecedente fumador de
más de 40 años de evolución con índice tabáquico de moderada
intensidad así como dislipidemia por lo que recibe tratamiento
con bezafibrato y atorvastatina. En la exploración física presenta
phlegmasia alba dolens + pulsos poplíteo y pedio disminuidos
de extremidad inferior izquierda. Con este diagnóstico usted
manda realizar el estudio considerado estándar de oro el cual es:
a. Ecografía Doppler
b. Ecografía en modo B
c. Angiografía diagnóstica con medio de contraste
d. Angiografía por resonancia con medio de contraste
e. Angiografía por tomografía con medio de contraste
8. Femenino de 23 años quien acude a urgencias con dolor
abdominal de 48 horas de evolución en cuadrante inferior
derecho, de inicio súbito en esta área. Se acompaña de nauseas y
vomito alimenticio, fiebre no especificada, disuria, descarga
vaginal. Presenta menstruación. Exploración con facies
dolorosa, diaforesis, taquicardia, deshidratada, abdomen con
dolor a la palpación profunda y a la descompresión en fosa iliaca
derecha; dolor contralateral al palpar fosa iliaca izquierda, dolor
al pellizcar piel de cresta iliaca derecha. Leucocitos de 22,500
con bandas 25% y segmentados aumentados. Se establece el
diagnostico de apendicitis aguda. La mortalidad aproximada por
apendicitis aguda perforada en esta paciente se estima en:
a. 0.3%
b. 50%
c. 0%
d. 1.7%
e. 10%

1, Masculino de 73 años con ingesta de múltiples medicamentos


antiinflamatorios que presento dolor abdominal de 36 horas de
evolución, nausea y fiebre de 38.7°C; a la exploración física se
identificó abdomen “en madera”. Laboratorio: Leucocitosis de
22,500, neutrófilos del 87% y 6% de bandas. ¿Qué estudio de
gabinete solicitaría de primera elección?
a. Radiografías simples de abdomen y tórax
b. Endoscopia de tubo digestivo alto
c. Ultrasonido abdominal
d. Tomografía computada de abdomen alto
e. Gammagrafía con tecnecio 99

¿Cuál es el dato radiológico que hace poner en duda el


diagnostico de obstrucción mecánica del intestino?
a. Neumoperitoneo
b. Dilatación de intestino delgado
c. Dilatación de intestino grueso
d. Niveles hidroaéreos
e. Gas en ampolla rectal

Paciente masculino de 40 años de edad, bombero, el cual sufre


quemaduras por fuego directo al estar trabajando, a la
exploración sin compromiso cardiorespiratorio, presenta
lesiones en cara anterior de tórax, extremidades, superior e
inferior, indoloras, duras, que no palidecen a la presión, que
afectan menos del 50% de superficie. ¿Para la reanimación
agresiva de líquidos que solución es la más indicada?
a. Solución Mixta
b. Solución Fisiológica
c. Solución Ringer Lactato
d. Solución glucosada
5to Banco
6to Banco

Preguntas Examen Cirugía:

1.- En un paciente que presenta Neumotorax del 50 %secuandario a la colocación de un


catéter sublcavio. ¿Cuál es el tratamiento que debe efectuarse?
R= colocación de sonda pleural.

2.- Masculino de 38 años de edad, víctima de accidente. Sufrío trauma abdominal que
curso con choque hipovolémico. Se le intervino Qx, le fueron admon un total de 8L
volumen, al inicio del 2ndo dia postoperatorio sus SV: TA 130/80mmHg, FC 80 x´, FR 16X
´, T 37°C. El paciente presenta quemosis, estertores crepitantes en ambas bases
pulmonares, y signo de Godette + en ambas extremidades inferiores. Su laboratorio
reporta. HB 9.2 , GLC 120, Urea de 20, Cr. 0.6, Na 140, Cl 100, K 4, ALB 3.
¿cuál de las siguientes consideraciones debe hacerse en el manejo de este paciente?
R= reducir el aporte exógeno de soluciones.

3.- Usted revisa a un paciente de 70 años de edad, masc, cursa con cuadro de oclusión
intestinal, tiene persistencia de vómitos de contenido intestinal, no tiene Cx previas.
¿Cuál puede ser la causa de la obstrucción mas frecuente en este caso?
R= Hernia inguinal incarcerada.

4.- Femenina de 35 años de edad la cual por ecosonograma se reposta Colelitiasis y


coledocolitiasis, usted decide operarla por laparoscopia. ¿Cuál sería la acción correcta
precia a la cirugía?
R= CPRE-esfinteroromía-.

5.- Tipo de sonda empleada para crear una fístula controlada, la cual se emplea en la vía
biliar.
R= sonda en T

6.- Escolar de 19 años de edad, es llevado a urgencias por sus padres con sospecha de
apendicitis. A la inspección se encuentra en posición antálgica, al interrogatorio refiere
que el dolor intenso inició hace dos días por la noche, por lo que no pudo dormír. La
intensidad del dolor disminuyo después de 36h, sin embargo desde hace 2 horas el dolor
reaparecio. Siendo el dolor mas intenso y difuso. ¿Cómo explica la evolución del dolor en
este paciente?
R= existe perforación apendicular.

7.- Acude al servicio de urgencias a valorar un paciente que presenta cuadro de oclusión
intestrinal, como antecedente tiene 2 Cx previas. ¿Cuál seria la causa mas probable?
R= adherencias.

8.- Indicar cual de las siguientes condiciones ponen en peligro inmediato la vida de
pacientes politraumatizados.
R= hipoxemia-hipercapnia por trauma de torax.

9.-Topografía donde debe realizarse la colocación de un mini sello pleura.


R= segundo espacio intercostal – línea media clavicular.

10.- Paciente femenino de 42 años de edad, 2 días de evolución con dolor abdominal
intenso en cuadrante superior derecho, que se irradia al hombro derecho el cual no le
permite inspirar en forma profunda, SV; TA 130/90, FC 90, FR 16, T 39.5°C. Tele de
tórax con elevación de hemidiafragma derecho y en el ultrasonido reporta zona
hipoecogenica localizada en la cúpula del lóbulo derecho del hígado con diagnóstico de
absceso hepático. Si este sufriera ruptura hacia cavidad abdominal. Que manifestación
clínica esperaría encontrar.
R= signos de irritación peritoneal.

11.- Paciente femenina de 42 años con lesión circular de 1.2 cm en región plantar
derecha, asimétrica, bordes irregulares, hipo e hipercromía en su interior. Según su
impresión diagnóstica, ¿Cuál es su tratamiento de elección?
R= Resección más terapia adyuvante.

12.- Paciente que sufrió trauma abdominal y se encuentra en terapia intensiva, tiene 20
dias de hospitalización, sus condiciones tienen mal pronóstico, actualmente abdomen
con cavidad abierta,
¿Cuál puede ser causa de mortalidad?
R= Choque séptico.

13.- La envían a usted a retirar un pen-rose de un paciente post-operado de perforación


intestinal, ¿Qué características debería de tener el líquido drenado para que usted debe
retirar el drenaje?
R: Serohematico

14.- Femenino de 69 años de edad diabética de 20 años acude a consulta por dolor
abdominal de 5 días de evolución localizado en hipocondrio derecho que se exacerba a
la inspiración profunda y que se irradia al hombro derecho, SV: TA 130-90 FC 90 FR 21 T
38.8c. Las escleras muestran leve tinte ictérico con presencia de Murphy se palpa en
cuadrante superior derecho, zona indurada y bastante dolorosa datos de peritonitis
localizada ¿cuál sería su diagnóstico más probable?
R: Piocolecisto.

15.- Paciente el cual exportador de un tumor en piel altamente sospechoso de ser


maligno ¿Cuál procedimiento deberá realizarse?
R: Biopsia excisional

16.- Paciente masculino de 54 años de edad portador de estenosis pilórica por proceso
cicatrizal de ulceración ¿Qué procedimiento quirúrgico es el adecuado?
R: Piloroplastia y vagotomía tropular.

17.- Paciente masculino de 3 años de edad quien presenta cuadro clínico de apendicitis
aguda no complicada no se cuenta con laparoscopia ¿Cuál es la incisión más conveniente
para el paciente?
R: Rockey Davis

18.- Femenino de 75 años traída el servicio de urgencias por presentar dolor abdominal
intenso de inicio gradual y continuo de 3 horas evolución. SV: TA 110/70, FC 110 FR 26 T
37.6c. Tórax con ruidos ventilatorios y cardiacos normales abdomen con dolor a la
palpación media y profunda con resistencia voluntaria y sin rebote se solicitó una tira
reactiva para orina en la cual se observa glucosuria y cuerpos cetonicos como
diagnóstico diferencial de abdomen agudo no quirúrgico ¿Qué trastorno presenta este
paciente?
R: Cetoacidosis diabética.
19.- Paciente de 38 años se encuentra en manejo por pancreatitis grave se tiene que
realizar curación de cavidad abdominal dos veces por día ¿Qué complicaciones se pueden
presentar por este manejo?
R: Hemorragia y fistulas.
20.- En estudio endoscópico se encontró ulceración a nivel del antro gástrico ¿Cuál es el
mecanismo por lo cual el helicobacter contribuye a la presentación de la ulcera gástrica?
R: Disminuye la protección de la mucosa gástrica.

21.- Masculino de 22 años de edad PO apendicetomía no complicada dado de alta,


¿Cuándo se deben retirar los puntos?
R: 7 días.

22.- Femenino de 23 años de edad con dolor abdominal nauseas, y vomito de contenido
intestinal de 6 horas de evolución como antecedente de importancia fue sometida a
apendicetomía hace dos meses SV: TA 120/70 FC 110 FR 20 T 37c. Presenta palidez
diaforesis y distención abdominal a la auscultación ruidos metálicos de lucha a la
percusión timpanismo difuso rebote positivo en los cuatro cuadrantes ¿Qué diagnostico
considera que debe escribir en la solicitud de interconsulta de cirugía?
R: Bridas.

23.- Paciente portadora de pancreatitis edematosa con presencia de cálculo enclavado


en colédoco a nivel ampular ¿Qué acción deberá realizarse en las primeras horas para
evitar el desarrollo de pancreatitis grave?
R: Esfinterectomia de Oddie

24.- Paciente masculino 80 años de edad con dolor abdominal difuso antecedente de
ingesta de aspirinas en forma crónica usted sospecha de perforación gástrica ¿Qué
estudio solicitaría para descubrir datos que sugieran perforación?
R: Placa de tórax de pie

25.- Paciente femenino de 80 años de edad a la que se practicó laparotomía exploratoria


a través de una incisión media supraumbilical presenta en el 5 día postoperatorio
evisceración. La aponeurosis no cerro con vicril del 1 sutura continúa ¿Cuál de las
siguientes opciones define a esta complicación?
R: Exteriorización de vísceras abdominal.

26.- Masculino de 16 años de edad, con sospecha de apendicitis complicada. Despues del
interrogatorio en la exploración física de abdomen. Debera realizar maniobras
especiales. SV:
TA 120/70, FC 100, FR 14, T 38°C. ¿Cuál de las siguientes signos le orienta a diagnosticar
la peritonitis en fosa iliaca derecha?
R= Robbsing, Plumberg e IleoPsoas.

27.- Paciente del cual es portador de perforacion de vicera si se aplica antibioticos antes
de la cirugía, ¿cuál de los siguientes es concepto correcto en relación a esta cirugía?
R= Requiere de antibiótico curativo.

28.- Paciente sometida a la laparotomía exploratoria por embarazo tubárico roto. Al dia
siguiente del postoperatorio se detecta hematoma en la herida quirúrgica de las
siguientes. ¿Cuál de las siguientes es la cuasa directa de esta complicación?
R= Mala técnica hemostásica.

29.- Paciente femenino de 35 años de edad, internada en terapia intensiva con


diagnóstico postoperatorio de drenaje de absceso pélvico. El estudio de Quimica
sanguínea revela
Urea 240, Creatinina 5. Debido a que la gasometría reporto hipoxemia e hipercapnia, ha
sido necesario el apoyo ventilatorio mecánico con lo que presento mejoría. Los SV: TA
120/80. FC 110, FR 25, T 38°C. ¿Qué complicación presente esta paciente?
R= Falla orgánica múltiple.

30.- Paciente masculino de 21 años de edad. Víctima de choque automovilístico. TA


80/60, FC 130, FR 26, T 37.5°C SaO2 56%. Presenta esfuerzos inspiratorios
predominantemente abdominal.
Sx de Horner derecho y piloerección distal al dermatoma de C5. Los ruidos respiratorios
se encuentran disminuidos bilateralmente y los ruidos cardiacos son rítmicos y de buena
intensidad. Los paramédicos que los trasladan refieren sospecha de fráctura en la
columna vertebral. ¿qué manejo debe aplicar de inmediato en este paciente?
R= Establecer via aérea permeable.

31.- Al realizar laparotomía se encuentra con presencia de contenido intestinal colónico


con presencia de E. Coli de acuerdo a la clasificación de las heridas quirúrgicas ¿ a qué
clase corresponde?
R= Clase 4.

32.- Paciente masculino de 21 años de edad, que ingresa con antecedente de trauma
abdominal contuso ( no penetrante). A la evaluación inicial presente palidez de
tegumentos, diaforesis fría. SV TA 100/60, FC 110, FR 24, T36°C. En este momento el
paciente se encuentra en la fase aguda a la respuesta a la lesión. La elevación de una de
las siguientes sustancias caracteriza esta fase.
R= Córtisol.

33.- Masculino de 36 años de edad, diabético con sx de Fournier, gangrena sinérgica de


periné. Que cursa 2ndo dia de estancia en el aislado de UCI. Cada 12 horas es llevado a
quirófano séptico para la realización de aseo quirúrgico del área infectada. El dia de hoy
empeoro su estado general y ha sido necesaria la asistencia ventilatoria mecánica,
debemos considerar en la fisiopatología de este paciente que el agua pulmonar se ha
incrementado. ¿ A qué se debe este cambio patológico?
R= Al sx de fuga capilar que caracteriza el choque séptico.

34.- La compresión neuropática lesión frecuente en mujeres de 40 años de forma crónica


se da en trabajadoras que usan sus manos, presentando calambres y dolor región palmar
en 2,3 y cuarto dedo. ¿ A qué tipo de sx corresponde?
R= Sx túnel carpeano.

35.- Paciente masculino de 50 años de edad, se realiza hemicolectomía izquierda con


anastomosis por una adenocarcinoma. ¿Cuál considera factor intrínseco?
R= Falla renal.

36.- Masculino de 31 años de edad, víctima de asalto, presenta herida penetrante de


abdomen por arma blanca. Durante la laparotomía se evacuan 3 litros de sangre y se
encuentra sección de la arteria mesentérica inferior. Como antecedente de importancia
durante la reanimación preoperatoria, se administraron 2 litros de cristaloides y 2
unidades de sangre almacenada. En el transoperatorio se han transfundido otras 3
unidades de sangre almacenada. El cirujano nota sangrado en capa por diversos sitios de
la cavidad. De las siguientes opciones, ¿cuál es la causa de este sangrado?
R= Plaquetopenia y consumo de factores de la coagulación.

37.- Paciente masculino de 20 años de edad a quien se le practicó laparotomía por


apendicitis perforada. Al quinto día del postoperatorio, presenta fiebre persistente,
venas del cuello colapsadas, SV: TA 90/50 FC 145 FR32 T39c. llenado capilar de 4
segundos y pulsos débiles, la diuresis en 8 horas de 100ml ¿Qué diagnostico integra con
estos datos?
R: Choque Séptico

38.- Paciente masculino 30 años de edad dolor abdominal de 24 horas de evolución con
antecedentes de cirugías previas, radiografía muestra niveles hidroaereos ¿Cuál es en
este momento el procedimiento aceptado para mejorar la distención abdominal?
R: Colocar sonda nasogástrica.

39.- Femenino de 34 años de edad con ictericia ¿Cuál dato clínico demostraría que la
ictericia es por bilirrubinemia directa?
R: Coluria

40.- Paciente masculino de 35 años de edad con herida penetrante en región poplítea. Al
retirar sus vestimentas se puede observar que no hay sangrado actual pero que podemos
referirnos al sangrado previo como hemorragia dado a que el paciente muestra palidez e
hipotensión y su ropaje está impregnado en sangre. ¿Qué componente de la hemostasia
se adhiere al colágeno que ha salvado de desangramiento al paciente?
R: Plaquetas

41.- Femenino de 43 años de edad 70 kg en estado de choque hipovolémico por diarrea


coleriforme se ha iniciado fluido terapia y para vigilar su repuesta se ha colocado una
sonda urinaria para considerar que el riñón tengo una perfusión sanguínea adecuada
¿cuánto debe que tener degasto urinario?
R: 35-70ml/hora
71-0 Banco

Preguntas Examen Cirugía:

1.- En un paciente que presenta Neumotorax del 50 %secuandario a la colocación de un


catéter sublcavio. ¿Cuál es el tratamiento que debe efectuarse?
R= colocación de sonda pleural.

2.- Masculino de 38 años de edad, víctima de accidente. Sufrío trauma abdominal que
curso con choque hipovolémico. Se le intervino Qx, le fueron admon un total de 8L
volumen, al inicio del 2ndo dia postoperatorio sus SV: TA 130/80mmHg, FC 80 x´, FR 16X
´, T 37°C. El paciente presenta quemosis, estertores crepitantes en ambas bases
pulmonares, y signo de Godette + en ambas extremidades inferiores. Su laboratorio
reporta. HB 9.2 , GLC 120, Urea de 20, Cr. 0.6, Na 140, Cl 100, K 4, ALB 3.
¿cuál de las siguientes consideraciones debe hacerse en el manejo de este paciente?
R= reducir el aporte exógeno de soluciones.

3.- Usted revisa a un paciente de 70 años de edad, masc, cursa con cuadro de oclusión
intestinal, tiene persistencia de vómitos de contenido intestinal, no tiene Cx previas.
¿Cuál puede ser la causa de la obstrucción mas frecuente en este caso?
R= Hernia inguinal incarcerada.

4.- Femenina de 35 años de edad la cual por ecosonograma se reposta Colelitiasis y


coledocolitiasis, usted decide operarla por laparoscopia. ¿Cuál sería la acción correcta
precia a la cirugía?
R= CPRE-esfinteroromía-.

5.- Tipo de sonda empleada para crear una fístula controlada, la cual se emplea en la vía
biliar.
R= sonda en T

6.- Escolar de 19 años de edad, es llevado a urgencias por sus padres con sospecha de
apendicitis. A la inspección se encuentra en posición antálgica, al interrogatorio refiere
que el dolor intenso inició hace dos días por la noche, por lo que no pudo dormír. La
intensidad del dolor disminuyo después de 36h, sin embargo desde hace 2 horas el dolor
reaparecio. Siendo el dolor mas intenso y difuso. ¿Cómo explica la evolución del dolor en
este paciente?
R= existe perforación apendicular.

7.- Acude al servicio de urgencias a valorar un paciente que presenta cuadro de oclusión
intestrinal, como antecedente tiene 2 Cx previas. ¿Cuál seria la causa mas probable?
R= adherencias.

8.- Indicar cual de las siguientes condiciones ponen en peligro inmediato la vida de
pacientes politraumatizados.
R= hipoxemia-hipercapnia por trauma de torax.

9.-Topografía donde debe realizarse la colocación de un mini sello pleura.


R= segundo espacio intercostal – línea media clavicular.

10.- Paciente femenino de 42 años de edad, 2 días de evolución con dolor abdominal
intenso en cuadrante superior derecho, que se irradia al hombro derecho el cual no le
permite inspirar en forma profunda, SV; TA 130/90, FC 90, FR 16, T 39.5°C. Tele de
tórax con elevación de hemidiafragma derecho y en el ultrasonido reporta zona
hipoecogenica localizada en la cúpula del lóbulo derecho del hígado con diagnóstico de
absceso hepático. Si este sufriera ruptura hacia cavidad abdominal. Que manifestación
clínica esperaría encontrar.
R= signos de irritación peritoneal.

11.- Paciente femenina de 42 años con lesión circular de 1.2 cm en región plantar
derecha, asimétrica, bordes irregulares, hipo e hipercromía en su interior. Según su
impresión diagnóstica, ¿Cuál es su tratamiento de elección?
R= Resección más terapia adyuvante.

12.- Paciente que sufrió trauma abdominal y se encuentra en terapia intensiva, tiene 20
dias de hospitalización, sus condiciones tienen mal pronóstico, actualmente abdomen
con cavidad abierta,
¿Cuál puede ser causa de mortalidad?
R= Choque séptico.

13.- La envían a usted a retirar un pen-rose de un paciente post-operado de perforación


intestinal, ¿Qué características debería de tener el líquido drenado para que usted debe
retirar el drenaje?
R: Serohematico

14.- Femenino de 69 años de edad diabética de 20 años acude a consulta por dolor
abdominal de 5 días de evolución localizado en hipocondrio derecho que se exacerba a
la inspiración profunda y que se irradia al hombro derecho, SV: TA 130-90 FC 90 FR 21 T
38.8c. Las escleras muestran leve tinte ictérico con presencia de Murphy se palpa en
cuadrante superior derecho, zona indurada y bastante dolorosa datos de peritonitis
localizada ¿cuál sería su diagnóstico más probable?
R: Piocolecisto.

15.- Paciente el cual exportador de un tumor en piel altamente sospechoso de ser


maligno ¿Cuál procedimiento deberá realizarse?
R: Biopsia excisional

16.- Paciente masculino de 54 años de edad portador de estenosis pilórica por proceso
cicatrizal de ulceración ¿Qué procedimiento quirúrgico es el adecuado?
R: Piloroplastia y vagotomía tropular.

17.- Paciente masculino de 3 años de edad quien presenta cuadro clínico de apendicitis
aguda no complicada no se cuenta con laparoscopia ¿Cuál es la incisión más conveniente
para el paciente?
R: Rockey Davis

18.- Femenino de 75 años traída el servicio de urgencias por presentar dolor abdominal
intenso de inicio gradual y continuo de 3 horas evolución. SV: TA 110/70, FC 110 FR 26 T
37.6c. Tórax con ruidos ventilatorios y cardiacos normales abdomen con dolor a la
palpación media y profunda con resistencia voluntaria y sin rebote se solicitó una tira
reactiva para orina en la cual se observa glucosuria y cuerpos cetonicos como
diagnóstico diferencial de abdomen agudo no quirúrgico ¿Qué trastorno presenta este
paciente?
R: Cetoacidosis diabética.
19.- Paciente de 38 años se encuentra en manejo por pancreatitis grave se tiene que
realizar curación de cavidad abdominal dos veces por día ¿Qué complicaciones se pueden
presentar por este manejo?
R: Hemorragia y fistulas.
20.- En estudio endoscópico se encontró ulceración a nivel del antro gástrico ¿Cuál es el
mecanismo por lo cual el helicobacter contribuye a la presentación de la ulcera gástrica?
R: Disminuye la protección de la mucosa gástrica.

21.- Masculino de 22 años de edad PO apendicetomía no complicada dado de alta,


¿Cuándo se deben retirar los puntos?
R: 7 días.

22.- Femenino de 23 años de edad con dolor abdominal nauseas, y vomito de contenido
intestinal de 6 horas de evolución como antecedente de importancia fue sometida a
apendicetomía hace dos meses SV: TA 120/70 FC 110 FR 20 T 37c. Presenta palidez
diaforesis y distención abdominal a la auscultación ruidos metálicos de lucha a la
percusión timpanismo difuso rebote positivo en los cuatro cuadrantes ¿Qué diagnostico
considera que debe escribir en la solicitud de interconsulta de cirugía?
R: Bridas.

23.- Paciente portadora de pancreatitis edematosa con presencia de cálculo enclavado


en colédoco a nivel ampular ¿Qué acción deberá realizarse en las primeras horas para
evitar el desarrollo de pancreatitis grave?
R: Esfinterectomia de Oddie

24.- Paciente masculino 80 años de edad con dolor abdominal difuso antecedente de
ingesta de aspirinas en forma crónica usted sospecha de perforación gástrica ¿Qué
estudio solicitaría para descubrir datos que sugieran perforación?
R: Placa de tórax de pie

25.- Paciente femenino de 80 años de edad a la que se practicó laparotomía exploratoria


a través de una incisión media supraumbilical presenta en el 5 día postoperatorio
evisceración. La aponeurosis no cerro con vicril del 1 sutura continúa ¿Cuál de las
siguientes opciones define a esta complicación?
R: Exteriorización de vísceras abdominal.

26.- Masculino de 16 años de edad, con sospecha de apendicitis complicada. Despues del
interrogatorio en la exploración física de abdomen. Debera realizar maniobras
especiales. SV:
TA 120/70, FC 100, FR 14, T 38°C. ¿Cuál de las siguientes signos le orienta a diagnosticar
la peritonitis en fosa iliaca derecha?
R= Robbsing, Plumberg e IleoPsoas.

27.- Paciente del cual es portador de perforacion de vicera si se aplica antibioticos antes
de la cirugía, ¿cuál de los siguientes es concepto correcto en relación a esta cirugía?
R= Requiere de antibiótico curativo.

28.- Paciente sometida a la laparotomía exploratoria por embarazo tubárico roto. Al dia
siguiente del postoperatorio se detecta hematoma en la herida quirúrgica de las
siguientes. ¿Cuál de las siguientes es la cuasa directa de esta complicación?
R= Mala técnica hemostásica.

29.- Paciente femenino de 35 años de edad, internada en terapia intensiva con


diagnóstico postoperatorio de drenaje de absceso pélvico. El estudio de Quimica
sanguínea revela
Urea 240, Creatinina 5. Debido a que la gasometría reporto hipoxemia e hipercapnia, ha
sido necesario el apoyo ventilatorio mecánico con lo que presento mejoría. Los SV: TA
120/80. FC 110, FR 25, T 38°C. ¿Qué complicación presente esta paciente?
R= Falla orgánica múltiple.

30.- Paciente masculino de 21 años de edad. Víctima de choque automovilístico. TA


80/60, FC 130, FR 26, T 37.5°C SaO2 56%. Presenta esfuerzos inspiratorios
predominantemente abdominal.
Sx de Horner derecho y piloerección distal al dermatoma de C5. Los ruidos respiratorios
se encuentran disminuidos bilateralmente y los ruidos cardiacos son rítmicos y de buena
intensidad. Los paramédicos que los trasladan refieren sospecha de fráctura en la
columna vertebral. ¿qué manejo debe aplicar de inmediato en este paciente?
R= Establecer via aérea permeable.

31.- Al realizar laparotomía se encuentra con presencia de contenido intestinal colónico


con presencia de E. Coli de acuerdo a la clasificación de las heridas quirúrgicas ¿ a qué
clase corresponde?
R= Clase 4.

32.- Paciente masculino de 21 años de edad, que ingresa con antecedente de trauma
abdominal contuso ( no penetrante). A la evaluación inicial presente palidez de
tegumentos, diaforesis fría. SV TA 100/60, FC 110, FR 24, T36°C. En este momento el
paciente se encuentra en la fase aguda a la respuesta a la lesión. La elevación de una de
las siguientes sustancias caracteriza esta fase.
R= Córtisol.

33.- Masculino de 36 años de edad, diabético con sx de Fournier, gangrena sinérgica de


periné. Que cursa 2ndo dia de estancia en el aislado de UCI. Cada 12 horas es llevado a
quirófano séptico para la realización de aseo quirúrgico del área infectada. El dia de hoy
empeoro su estado general y ha sido necesaria la asistencia ventilatoria mecánica,
debemos considerar en la fisiopatología de este paciente que el agua pulmonar se ha
incrementado. ¿ A qué se debe este cambio patológico?
R= Al sx de fuga capilar que caracteriza el choque séptico.

34.- La compresión neuropática lesión frecuente en mujeres de 40 años de forma crónica


se da en trabajadoras que usan sus manos, presentando calambres y dolor región palmar
en 2,3 y cuarto dedo. ¿ A qué tipo de sx corresponde?
R= Sx túnel carpeano.

35.- Paciente masculino de 50 años de edad, se realiza hemicolectomía izquierda con


anastomosis por una adenocarcinoma. ¿Cuál considera factor intrínseco?
R= Falla renal.

36.- Masculino de 31 años de edad, víctima de asalto, presenta herida penetrante de


abdomen por arma blanca. Durante la laparotomía se evacuan 3 litros de sangre y se
encuentra sección de la arteria mesentérica inferior. Como antecedente de importancia
durante la reanimación preoperatoria, se administraron 2 litros de cristaloides y 2
unidades de sangre almacenada. En el transoperatorio se han transfundido otras 3
unidades de sangre almacenada. El cirujano nota sangrado en capa por diversos sitios de
la cavidad. De las siguientes opciones, ¿cuál es la causa de este sangrado?
R= Plaquetopenia y consumo de factores de la coagulación.

37.- Paciente masculino de 20 años de edad a quien se le practicó laparotomía por


apendicitis perforada. Al quinto día del postoperatorio, presenta fiebre persistente,
venas del cuello colapsadas, SV: TA 90/50 FC 145 FR32 T39c. llenado capilar de 4
segundos y pulsos débiles, la diuresis en 8 horas de 100ml ¿Qué diagnostico integra con
estos datos?
R: Choque Séptico

38.- Paciente masculino 30 años de edad dolor abdominal de 24 horas de evolución con
antecedentes de cirugías previas, radiografía muestra niveles hidroaereos ¿Cuál es en
este momento el procedimiento aceptado para mejorar la distención abdominal?
R: Colocar sonda nasogástrica.

39.- Femenino de 34 años de edad con ictericia ¿Cuál dato clínico demostraría que la
ictericia es por bilirrubinemia directa?
R: Coluria

40.- Paciente masculino de 35 años de edad con herida penetrante en región poplítea. Al
retirar sus vestimentas se puede observar que no hay sangrado actual pero que podemos
referirnos al sangrado previo como hemorragia dado a que el paciente muestra palidez e
hipotensión y su ropaje está impregnado en sangre. ¿Qué componente de la hemostasia
se adhiere al colágeno que ha salvado de desangramiento al paciente?
R: Plaquetas

41.- Femenino de 43 años de edad 70 kg en estado de choque hipovolémico por diarrea


coleriforme se ha iniciado fluido terapia y para vigilar su repuesta se ha colocado una
sonda urinaria para considerar que el riñón tengo una perfusión sanguínea adecuada
¿cuánto debe que tener degasto urinario?
R: 35-70ml/hora
PREGUNTAS DE MANEJO DE INSTRUMENTAL.

¿Cuántas caras tiene el Mango del Bisturí?

1. 1
2. 2
3. 3
4. 4
5. 5

¿Qué instrumento se utiliza para montar la hoja en el mango del bisturí?

1. Kelly curvas
2. Kelly Rectas
3. Pinza de Anillos
4. Pinza de Disección
5. Porta-agujas

¿Cuándo el bisturí se toma como Lápiz se realizan cortes?

1. Largos
2. Medios
3. En circulo
4. Pequeños
5. Profundos

¿Cuando el bisturí se toma como arco de violín se realizan cortes?

1. Intermedios
2. Largos y profundos
3. Lisos y superficiales
4. Pequeños
5. cortos

¿Cuantos bisturís se usan en el acto quirúrgico?

1. Cinco
2. Cuatro
3. Dos
4. Tres
5. Uno

Dra. Ismene Anabel Verdugo Zazueta Universidad Autónoma de Guadalajara


¿Como se considera el bisturí después de cortar la piel?

1. Aséptico
2. Contaminado
3. estéril
4. Limpio
5. Antiséptico

¿Para qué se utilizan las tijeras curvas?

1. Antisepsia
2. Cortar gasas
3. Cortar Nylon
4. Cortar sutura
5. Disección de tejidos

¿Las tijeras rectas las utilizamos para?

1. Cortar Aponeurosis
2. Cortar Celular subcutáneo
3. Cortar Cutícula
4. Cortar Peritoneo
5. Sutura

¿Para realizar la disección de los tejidos la Tijera curva se coloca?

1. Con la Punta Para Atrás


2. Con la Punta de Lado
3. Con la curva para Arriba
4. Con la recta
5. Con la recta para arriba

¿Cuáles son los instrumentos de ayuda?

1. Pinzas de Anillos
2. Pinzas de Disección
3. Pinzas de Kelly
4. Pinzas de Mayo
5. Pinzas de Metzenbaum

Dra. Ismene Anabel Verdugo Zazueta Universidad Autónoma de Guadalajara


¿Cuáles son instrumentos de ayuda?

1. Pinza Allys
2. Pinza de Anillos
3. Pinza de Kelly
4. Pinza de Piel y campo
5. Porta-agujas

¿Dónde se Utilizan Las Pinzas sin dientes?

1. Celular Subcutáneo
2. Facia Lata
3. Grasa
4. Musculo
5. Peritoneo

¿Cuántos tipos de separadores hay?

1. Elásticos
2. Flexibles
3. Manuales y automáticos
4. Abiertos y cerrados
5. Compactos

¿Cuál es la forma correcta de entregar los Instrumentos?

1. Abiertas
2. De lado
3. De la Punta
4. De los Aros
5. Doblados

¿Cuál es la forma correcta de entregar las pinzas que tengan curvatura?

1. Con la curvatura Abierta


2. Con la curvatura de lado
3. Con la curvatura para abajo
4. Con la Curvatura Para Arriba
5. Con la curvatura para Atrás

Dra. Ismene Anabel Verdugo Zazueta Universidad Autónoma de Guadalajara


¿Cuáles son instrumentos de tracción?

1. Allis y Pinza de Anillos


2. Pinzas Kelly
3. Pinzas de Metzenbaum
4. Pinzas de Kelly curva
5. Pinzas de Mayo

¿Las pinza de piel y campo donde se utilizan

1. Para hacer antisepsia


2. Para Fijar los campos
3. Para fijar las sondas
4. Para realizar hemostasia
5. Para suturar

¿Para qué se utilizan las pizas de allys?

1. Para traccionar campos


2. Para traccionar fascia lata
3. Para traccionar peritoneo
4. Para traccionar tendones
5. Para Traccionar gasas

¿Cuáles es parte de una pinza?

1. Circulo
2. Cremellera
3. Cuello
4. Extremidades
5. Extensiones

¿Cuáles son pinzas de hemostasia?

1. Kelly curvas y rectas


2. Piel y Campo
3. Porta-agujas
4. Tijera de Mayo
5. Tijera de metzenbaum

Dra. Ismene Anabel Verdugo Zazueta Universidad Autónoma de Guadalajara


CIRUGÍA LA MEDIDA PREVENTIVA PARA ESTA
COMPLICACIÓN:
FEMENINO DE 42ª DE EDAD A QUIEN SE LE USO DE MEDIAS PLASTICAS
REALIZO COLICISTECTOMIA ABIERTA POR
COLECISTITIS CRÓNICA LITIASICA AL SALIR FEMENINO DE 48ª MULTIPARA, OBESA,
DE QUIRÓFANO TIENE COLOCADA UNA PROGRAMADA PARA HISTERECTOMÍA
SONDA NASOGASTRICA, SONDA VESICAL, VAGINAL POR PROLAPSO UTERINO, COMO
VENOCLISIS CON HARTMAN GLUCOSADA ANTECEDENTE MENCIONA SER DIABÉTICA Y
RESPIRACIÓN 18 X MIN. PULSO 75X´. TA PRESENTA INSUFICIENCIA VENOSA DE
130/85. TEMP 36.5° ESTE PACIENTE DEBERA MIEMBROS INFERIORES, CUAL DE LAS
DEAMBULAR: SIGUIENTES COMPLICACIONES
AL DIA SIGUIENTE DE LA CIRUGÍA PULMONARES ES PROBABLE QUE
PRESENTE ESTA PACIENTE EN EL
PACIENTE DE 56ª A QUIEN SE LE REALIZO POSTOPERATORIO:
ANTRECTOMIA CON RECONSTRUCCIÓN TROMBOEMBOLIA PULMONAR
BILIAR POR PRESENTAR ULCERA GÁSTRICA
PENETRADA A PÁNCREAS SE ENCUANTRA EN FEMENINO OBESA DE 35ª LA CUAL VA A SER
EL 3ER DIA POSTOPERATORIO CON SONDA SOMETIDA A COLECISTECTOMIA POR
NASOGASTRICA, SONDA VESICAL CON PRESENTAR COLECISTITIS CRÓNICA
HARTMAN GLUCOSADA Y CIPROFLOXACINO LITIASICA ADEMÁS DE TRATAMIENTO
500MG IV CADA 12 HRS, RESP 16X/ PULSO 75X ANTIRREFLUJO POR INCOMPETENCIA DEL
TA 130/85. CUANDO SE DEBERA RETIRAR LA ESFÍNTER ESOFÁGICO INFERIOR, LAS
SONDA DE LEVIN EN ESTE PACIENTE: ORDENES PREOPERATORIOS DEBERAN
AL PRESENTAR RUIDOS INTESTINALES INCLUIR:
AUTORIZACIÓN DE LA CIRUGÍA
INGRESA MASCULINO DE 22 AÑOS AL
SERVICIO DE URGENCIAS REFIRIENDO FEMENINO DE 40ª CON SIRPA Y
DOLOR EN FOSA ILIACA DERECHA. SE DESNUTRICIÓN PROTEICO CALORICA EN LA
DIAGNOSTICA APENDICITIS Y SE REALIZA ALIMENTACIÓN INTRAVENOSA PERIFERICA
APENDICECTOMIA. EL HALLAZGO EL INCREMENTO DE CALORÍAS SE LOGRARA
QUIRÚRGICO FUE APENDICITIS EN FASE MEDIANTE:
EDEMATOSA AL MOMENTO DE SU EGRESO LÍPIDOS AL 10%
HOSPITALARIO SE LE RETIRARAN LOS
PUNTOS A LOS: PACIENTE DE 50ª CON CANCER DE ESÓFAGO
7 DIAS Y CON INANICIÓN EN ESTA CIRCUNSTANCIA
SU CORAZON Y MÚSCULO TIENEN
MASCULINO DE 22ª EL CUAL REFIERE DOLOR PREDILECCIÓN POR UNO DE LOS SIGUIENTES
EN FOSA ILIACA DERECHA EL PACIENTE FUE AMINOÁCIDOS:
SOMETIDO A APENDICECTOMIA EL DE CADENA RAMIFICADA
HALLAZGO QUIRÚRGICO FUE APENDICITIS
PERFORADA (FASEIV) QUE TIPO DE CIERRE MASCULINO DE 36ª QUIEN DURANTE LA
PARA LA PIEL Y TEJIDO CELULAR INANICIÓN AGUDA DEVIDO A CANCER
SUBCUTÁNEO ES RECOMENDADO EN ESTE LARINGEO. TIENE UNA PERDIDA NETA DE
CASO: AMINOÁCIDOS, YA QUE LA SINTESIS DE
DIFERIDO PROTEINA MUSCULAR DESAPARECE O
DESCIENDE TANTO QUE EL CATABOLISMO
FEMENINO DE 38ª MULTIPARA, CON DE LA MISMA PERMANECE SIN CAMBIOS
CISTOCELE PROGRAMADA PARA ESTA SITUACIÓN ES DEVIDA A:
COLPOPERINEOPLASTIA LA PREPARACIÓN CONCENTRACIONES BAJAS DE INSULINA
DE LA VAGINA SE RELIZA EN:
QUIRÓFANO FEMENINO DE 28ª POLITRAUMATIZADO QUE
SE ENCUENTRA EN EL POSTOPERATORIO
MASCULINO DE 22ª SOMETIDO A QUE PESA 90KG TALLA 1.5M, QUE INGIERE
LAPAROTOMIA CON HALLAZGO DE 10GM DE PROTEINAS AL DIA.EN 2 LITROS DE
PERFORACIÓN DE ILEON POR ORINA ESTA ELIMINANDO 950MG/ DIA, DE
SALMONELLOSIS AL 3ER DIA NITRÓGENO UREICO. CON ESTO EL BALENCE
POSTOPERATORIO INICIA CON FIEBRE, DE NITRÓGENO AL DIA ES DE:
DOLOR, TUMEFACCIÓN, HIPEREMIA Y 21.4GM
AUMENTO DE LA TEMPERATURA LOCAL.
CUAL DE LAS SIGUIENTES ES LA CAUSA PACIENTE QUE FUE INTERVENIDO
PROBABLE CON ESTA COMPLICAION: QUIRÚRGICAMENTE REALIZÁNDOSELE
CIERRE PRIMARIO DE LA HERIDA COLECISTECTOMIA POR PRESENTAR
COLECISTITIS CRÓNICA LITIASICA, CON EL
MASCULINO DE 72ª OBESO, SOMETIDO A POSTOPERATORIO MEDIATO PRESENTO
COLECISTECTOMIA ABIERTA POR DEHISCIENCIA DE LA HERIDA QUIRÚRGICA.
COLECISTITIS AGUDA AL 4TO, DIA POST PARA REPARAR LA COMPLICAION Y CERRAR
OPERATORIO PRESENTA EDEMA. DOLOR, LA APONEUROSIS ELIGIRA EL SIGUIENTE
HIPERTEMIA Y COLORACIÓN VIOLACEA EN MATERIAL DE SUTURA:
EL MIEMBRO INFERIOR DERECHO, CUAL ES PROLENE 1
MASCULINO DE 45ª EL CUAL SUFRIO DE AUTOMOVILISTICO CON LESIONES LEVES.
TRAUMATISMO ABDOMINAL EN ACCIDENTE PERO FUE VICTIMA DE CRISIS EMOCIONAL,
AUTOMOVILISTICO Y QUIEN A SU INGRESO A SE LE TOMA GASOMETRIA REVELANDO UNA
URGENCIAS LLEGA EN ESTADO DE CHOQUE PCO2 DE 30MMHG TAL CIFRA Y EL CUADRO
HIPOVOLEMICO PARA QUE DESENCADENE CLINICO SUGIEREN:
LA RESPUESTA BIOLÓGICA AL TRAUMA ALCALOSIS RESPIRATORIA
DEBERA EXISTIR:
INTEGRACIÓN NOCICEPTIVA EN LE CEREBRO ANTE UN PACIENTE EL CUAL PRESENTA
VOMITOS FRECUENTES DE CONTENIDO
FEMNINO DE 34ª QUIEN SUFRIO GÁSTRICO SECUNDARIO A ESTENOSIS
POLITRAUMATISMO EN ACCIDENTE PILORICA LA ANORMALIDAD METABÓLICA
AUTOMOVILISTICO Y QUE ES VISTO EN EL PREDOMINANTE QUE ESPERARIA
SERVICIO DE URGENCIAS. EN EL ESTUDIO DE ENCONTRAR SERIA:
ESTE PACIENTE SE SABE QUE DENTRO DE LA ALCALOSIS HIPOCLOREMICA
RESPUETA METABÓLICA AL TRAUMA SE
PRODUCE UNA CERIE DE HORMONAS MASCULINO DE 22ª QUIEN SE CAYO DESDE
LLAMADAS CONTRARREGULADORAS DOS UN CUARTO PISO SE LE REALIZO
DE LAS CUALES SON: HEPATECTOMIA PARCIAL, RESECCION DE
EPINEFRINA Y NORADRENALINA 30CM DE INTESTINO DELGADO ASI COMO
ESPLENECTOMIA. RECIBIO 15UNIDADES DE
SANGRE EN EL PRE Y TRANS OPERATORIO. 8
LITROS DE SOLUCION HARTMAN Y 4
UNIDADES DE PLASMA FRESCO CONGELADO
ANTE UN PACIENTE QUE SE ENCUENTRA DURANTE EL CIARRE DE LA LAPAROTOMÍA
CURSANDO EL PRIMER DIA DEL SE NOTO SANGRADO DIFUSO. DE LAS
POSTOPERATORIO POR HABER SIDO SIGUIENTES CAUSAS CUAL ES LA MAS
SOMETIDO A TIREIDOCTOMIA SUBTOTAL PROBABLE DE ESTE SANGRADO:
POR PRESENTAR BOCIO MULTINODULAR PLAQUETOPENIA
UNA VEZ SUPERADA LA FASE EBB, USTED
ESPERA QUE EL PACIENTE SE ENCUENTRE EN FEMENINO DE 28ª LA CUAL PRESENTO
LA FASE: TRAUMA ABDOMINAL CERRADO CON
DE FLUJO RUTURA HEPÁTICA E ISQUEMIA INTESTINAL
POR DESGARRO DEL MESENTERIO, DURANTE
FEMENINO DE 30ª QUIEN SUFRE ACCIDENTE UNA INTERVENCIÓN QUIRÚRGICA PRESENTA
EN CARRETERA. EN EL LUGAR DEL SANGRADO DIFUSO DE LA
ACCIDENTE SE LE ENCUENTRA PARED ABDOMINAL Y EN HIGADO DEBIDO A
SEMICONSCIENTE, TAQUICARDIA, EN UNA PROBABLE CID, EL TRATAMIENTO QUE
TORAX PULMONES VENTILADOS, ABDOMEN RELIZARA EN ESTE MOMENTO SERA:
CON PARISTALSIS DISMINUIDA Y DOLOR EMPAQUETAMIENTO ABDOMINAL
GENERALIZADO. SE LE CANALIZA CON
HARTMAN Y ES LLEVADO AL CENTRO DE FEMENINO DE 39ª CON DOLOR EN
TRAUMA MAS CERCANO. DE LAS HIPOCONDRIO DERECHO SE REALIZA USG DE
SIGUIENTES CUAL ES LA QUE DISMINUYE EN PÁNCREAS. HIGADO Y VIA BILIAR Y SE
LA FASE INICIAL EN LA RESPUESTA DETECTA ENFERMEDAD CALCULOSA DE LA
BIOLÓGICA AL TRAUMA: VESÍCULA POR LO QUE FUE SOMETIDA A
CORTISOL CELECISTECTOMIA SIMPLE SE SOLICITAN
EXAMENES PREOPERATORIOS LOS CUALES
MASCULINO DE 68ª EL CUAL FUE SOMETIDO SON NORMALES, SI USTED DESEA DETECTAR
A RESECCION ABDOMINOPERINEAL POR DEFICIENCIA DEL FACTOR II CUAL ES EL
PRESENTAR CANCER DE RECTO, DURANTE ESTUDIO UTIL PARA EVALUARLO:
LA CIRUGÍA LA PERDIDA SANGUÍNEA FUE TIEMPO DE PROTROMBINA
RESPUESTA VOLUMEN A VOLUMEN, EN EL
POSTOPERATORIO FUE MANEJADO CON MASCULINO DE 42ª QUIEN INGRESA CON
SOLUCIONES DE CLORURO DE SODIO AL SANGRADO DE TUBO DIGETIVO ALTO.
0.45% PRESENTANDO DEBILIDAD, LETARGIA, ENTRE SUS ANTECEDENTES MENCIONA
CONVULSIONES Y POSTERIORMENTE COMA ALCOHOLISMO CRÓNICO. LABORATORIO:
POR LO REFERIDO EL PACIENTE ESTA HB 10GM/ DL, HTO 33%, PLAQUETAS 300,000.
CURSANDO CON: TPT 40´. A LA EXPLORACIÓN FÍSICA
HIPONATREMIA PACIENTE TRANQUILO; CONSIENTE TA.
100/60, SE COLOCA SONDA NASOGASTRICA Y
PACIENTE QUE PRESENTO TRAUMATISMO SE LE REALIZA LAVADO GÁSTRICO, DENTRO
ABDOMINAL CERRADO Y CHOQUE DEL MANEJO DE ESTE PACIENTE INDICA
HIPOVOLEMICO, EL PUNTO DE VISTA TRANSFUSIÓN DE :
FISIOPATOLOGICA A NIVEL HORMONAL, CONCENTRADO PLAQUETARIO
ESPERARIA ENCONTRAR AUMENTO DE LA
SECRECION DE LA SIGUIENTE HORMONA: PACIENTE EL CUAL FUE SOMETIDO A
CORTISOL HEMICOLECTOMIA POR CANCER DE COLON.
EN EL POSTOPERATORIO PRESENTA CUADRO
FEMENINO DE 25ª QUIEN SUFRIO ACCIDENTE COMPATIBLE CON CID LA ALTERACIÓN
LABORATORIAL QUE ESPERARIA COMPORTAMIENTOS DE LOS LIQUIDOS
ENCONTRAR SERA: CORPORALES ES:
PRESENCIA DE P. D. F NA +

MASCULINO DE 45ª A QUIEN SE LE REALIZO FEMENINO DE 40ª LA CUAL INGRESO A LA


CESAREA Y RESECCION DE 30CM DE UCI POR PRESENTAR CHOQUE SÉPTICO
INTESTINO DELGADO POR TRAUMATISMO SECUNDARIO A UNA PERFORACIÓN
ABDOMINAL CERRADO. DURANTE LA INTESTITAL. SE LE HAN APLICADO
CIRUGÍA SE LE TRANSFUNDIERON 10 VOLÚMENES ADECUADOS DE RINGER
UNIDADES DE SANGRE DEL BANCO. A LACTADO CONTROLADOS POR LA PRESION
PRESENTADO SANGRADO DIFUSO Y PIENSA EN CUÑA PULMONAR, DOPAMINA Y
EN CID, ESTA COMPLICAION SE NOREPINEFRINA A PESAR DE LAS MEDIDAS
EVIDENCIARA POR EL SIGUIENTE ANTERIORES LA PACIENTE NO HA
RESULTADO DE LABORATORIO: MOSTRADO MEJORIA, ES MAS SE HA
PRESENCIA DE PRODUCTOS DE DIVISIÓN DE AGREGADO ICTERICIA Y CONFUCION
FIBRINA MENTAL. POR LO QUE MUESTRA EL
PACIENTE ESTA PRESENTANDO:
FEMENINO DE 28ª SE LE REALIZO CESAREA SÍNDROME DE DISFUNCIÓN ORGANICA
POR PRESENTAR OBITO DE 24 SEMANAS DE MÚLTIPLE
GESTACIÓN Y A QUIEN EN EL
POSTOPERATORIO SE LE DIAGNOSTICO CID. MASCULINO DE 55ª EL CUAL FUE SOMETIDO
CON EL OBJETO DE REPONER LA MAYOR A GASTRECTOMÍA SUBTOTAL POR
CANTIDAD DE LOS FACTORES DE LA PRESENTAR CANCER IN SITU EN EL
COAGULACION CONSUMIDOS, CUAL DE LAS POSTOPERATORIO MEDIATO ES
SIGUIENTES MEDIDAS ES LA MAYOR FUENTE INDISPENSABLE EL USO DE ANELGESICOS
DE LOS FACTORES DE LA COAGULACIÓN ´´. CON HORARIO, PARA PREVENIR LA
PLASMA FRESCO CONGELADO SIGUIENTE COMPLICACIÓN:
SÍNDROME DE DISFUNCIÓN ORGANICA
MÚLTIPLE
* MASCULIJNO DE 53ª QUIEN INGRESO A UCI
POR PRESENTAR HIPOTENCION SEVERA CON
SIFRAS TENCIONALES DE TA 70/40. MASCULINO DE 55ª EL CUAL FUE SOMETIDO
REVELADO A LA APLICAION DE VOLÚMENES A GASTRECTOMÍA SUBTOTAL POR
ADECUAOS DE RINGE LACTATO E PRESENTAR CANCER IN SITU EN EL
INOTROPICOS, ADEMÁS DE FIEBRE E POSTOPERATORIO MEDIATO ES
INSUFICIENCIA RENAL, SI FINALMENTE CON INDISPENSABLE EL USO DE ANALGÉSICOS
LAS MEDIDAD ANTERIORES LA TA NO SE CON HORARIOS, PARA PREVENIR LA
RECUPERA USTED OPTARIA POR INDICAR: SIGUIENTE COMPLICACIÓN:
APLICACION DE VASOPRESINA ATELECTASIA

MASCULINO DE 23ª QUIEN PRESENTA DESDE


HACE 3 DIAS, ICTERICIA, FIEBRE, DOLOR
ABDOMINAL, HECES HIPERPIGMENTADAS Y
FEMENINO DE 28ª QUIEN INGRESA POR ORINA CLARA. A LA EXPLORACIÓN EXISTE
SANGRADO TRANSVAGINAL DOLOR A LA PALPACIÓN DELCUADRANTE
DETERMINÁNDOSE COMO CAUSA ABORTO IZQUIERDO DONDE SE PALPA UNA MASA
INCOMPLETO EN EL PRIMER TRIMESTRE POR QUE SOBRE SALE DEL BORDE COSTAL Y
LO QUE SE RELIZA LEGRADO UTERINO A SIGUE LOS MOVIMIENTOS RESPIRATORIOS,
CONSECUENCIA DE ELLO LA PACIENTE ESTA ESTE CUADRO SE REPITE UNA O DOS VECES
EN CHOQUE HIPOVOLEMICO LA DIFERENCIA AL AÑOS Y SE PRESENTA TAMBIEN EN 2
DE O2 ARTERIO VENOSO EN PULMON EN LA HERMANOS SUYOS, EL DIAGNOSTICO
FASE TEMPRANA DEL CHOQUE PROBABLE ES:
HIPOVOLEMICO SE ENCUENTRA: HIPERESPLENISMO
PEQUEÑA DIFERENCIA DEL O2
ARTERIOVENOSO
MASCULINO DE 47ª EL CUAL HA PADECIDO
MASCULINO DE 53ª QUE INGRESO A LA UCI DE DOLOR ABDOMINAL RECURRENTE
POR PRESENTAR ESTABILIZACIÓN DEL DURANTE LOS ULTIMOS 3 AÑOS.
CHOQUE HIPOVOLEMICO Y MEJOR ACTUALMENTE PRESENTA DOLOR
CONTROL, POR LO QUE SE COLOCA UN EPIGASTRICO INTENSO Y CONTINUO,
CATETER DE SWANZ GANZ PARA VALORAR: NAUSEAS DE 6 HORAS DE VOMITOS QUE NO
P. V. C ALIVIAN EL DOLOR EN LOS ULTIMOS
MOMENTOS EL DOLOR TIENDE A
MASCULINO DE 60ª A QUIEN SE LE REALIZO IRRADIARSE AL DORSO Y HEMIABDOMEN
RESECCION INTESTINAL EN EL IZQUIERDO A LA EXPLORACIÓN PRESENTA
POSTOPERATORIO INMEDIATO EL RESISTENCIA ABDOMINAL EPIGASTRICA.
ELECTROLITO QUE REFLEJA MEJOR LA CON LOS DATOS
ALTERACIÓN EN LA COMPOSICIÓN DE LOS
ANTERIORES EL PADECIMENTO SERA:
PANCREATITIS AGUDA FEMENINO QUE ABSRCA UNA QUEMADURA
QUE ABRCA UNA SUPERFICIE CORPORAL
MASCULINO DE 47ª EL CUAL HA QUEMADURA EN TODA LA CARA ANTERIOR
PRESENTADO DOLOR ABDOMINAL DE TIPO DEL TRONCO, MIEMBRO SUPERIOR
ARDOSO DESDE HACE 3 AÑOS, QUE LO DERECHO, MANO IZQUIERDA EN LA CABEZA
DESPIERTA POR LAS NOCHES, DESDE HACE QUEMADURAS EN CEJAS, PESTAÑAS Y
UN AÑO TOMA DE FORMA IRREGULAR VIBRISAS QUEMADAS Y EXPECTORACION
ANTIÁCIDOS Y BLOQUEADORES H2 CARBONEADA ADEMÁS DE INCONCIENCIA.
PRESCRITOS POR FACULTATIVO, SIN SEGÚN ELCALCULO DE LAS REGIONES
MEJORIA APARENTE, ACTUALMENTE EL LESIONADAS. POR SU SUPERFICIE
DOLOR ES CONTINUO, MAS INTENSO Y SE CORPORALEQUIVALE A:
IRRADIA AL DORSO. LA TERAPEUTICA 50-60%
ADECUADA ES:
GASTRECTOMÍA PARCIAL MASCULINO DE 30ª QUIEN PRESENTA
QUEMADURAS SECUNDARIAS A EXPOSICIÓN
MASCULINO DE 22ª INGRESA AL SERVICIO DE DE FUEGOS PIROTÉCNICOS EN CARA,
URGENCIAS EN DONDE USTED SE MANOS, GENITALES, TORAX 70% PARA
ENCUENTRA, REFIRIENDO DOLOR EN FOSA VALORAR LA PERFUSION RENAL DURANTE
ILIACA DERECHA POR LO QUE SE LE EL TRATAMIENTO DE REPOSICIÓN DE
DIAGNOSTICA APENDICITIS, DE LOS LIQUIDOS:
SIGUIENTES EXMENES PREOPERATORIOS DEBERA VIGILARSE DIURESIS HORARIA A
CUALES SERIAN DE IMPORTANCIA : RAZON DE 30-50
RX SIMPLE DE EBDOMEN
TP Y TPT MASCULINO DE 30ª CON QUEMADURAS DEL
BIOMETRÍA HEMATICA 60% DE SEGUNDO GRADO SUPERICIALES Y
QUÍMICA SANGUÍNEA PROFUNDAS. EL CUAL HA ESTABILIZADO
HEMODINAMICO, LA DIETA DEBE HACERSE
MASCULINO DE 67ª PROSTATICO DIABÉTICO, POR LA VIA:
OBESO Y CON EPC, QUIEN HACE UN MES ORAL
EMPEZO A NOTAR MASA TUMORAL EN REGIO
INGUINAL DERECHA QUE DESAPARECE PACIENTE QUE SUFRE ACCIDENTE
RAPIDAMENTE AL ADOPRTAR EL DECUBITO AUTOMOVILISTICO Y QUE AL EXAMEN SE
DORSAL PARA MANEJAR ESTAS ENCUENTRA DESPIERTO Y RESPONDE A LAS
CONDICIONES SIGUIENTES EL TRATAMIENTO PREGUNTAS A NIVEL DE TORAX SE
INDICADO EN ESTE PACIENTE ES: ADOPTAR ENCUANTRAN DATOS QUE SUGIEREN
EL USO DE BRANGUERO NEUMOTORAX A TENSIÓN EN HEMITORAX
DERECHO, LA MEDIDA A APLICAR PARA ESTE
PACIENTE DE 50 AÑOS A QUIEN SE LE PROBLEMA SERA: TORACOCENTESIS EN 2°
PRACTICO LAPAROTOMÍA MEDIA ESPACION INTERCOSTAL
EXPLORADORA PRESENTA EN EL 5° DIA DEL
POSTOPERATORIO UNA EVICERACION. ESTA UN PACIENTE POLITRAUMATIZADO QUIEN
COMPLICACIÓN SE DEFINE COMO: EN EL EXAMEN LE ENCONTRAMOS EN
SEPARACIÓN DE TODOS LOS PLANOS DE LA HEMITORAX IZQUIERDO SILENCIO
HERIDA RESPIRATORIO A LA PERCUSIÓN
TIMPANISMO Y CON CHOQUE DE LA PUNTA A
LA DERECHA REALIZÁNDOSELE
PACIENTE DE 45ª EL CUAL PRESENTA UNA TORACOCENTESIS DE URGENCIA, LA
MASA EN REGION INGINAL IZQUIERDA JUSTIFICACIÓN DE ESTA MEDIDA ES POR:
DESDE HACE 3 AÑOS LA CUAL SE HACE ES LA DE ELECCIÓN PARA MANEJAR LA
EVIDENTE AL ESFUERZO Y DISMINUYE CONFUSIÓN PULMONAR
CON EL REPOSO. NO LLEGANDO AL ESCROTO
EN LA EXPLORACIÓN AL PEDIRLE QUE PUJE MASCULINO DE 34ª LLEVADO A URGENCIAS
LA MASA HACE CONTACTO CON EL PULPEJO POR SUFRIR ACCIDENTE AUTOMOVILISTICO
DEL DEDO EXPLORADOR EN ESTE CASO EL AL EXAMEN ENCONTRAMOS
DEFECTO SE ENCUENTRA EN: HIPERSENSIBILIDAD LEVE EN ABDOMEN Y
DENTRO DEL TRIANGULO DE HASSELBASH HEMITORAX DERECHO CON PERISTALSIS
MUY DISMINUIDA Y AUSENCIA DE METIDEZ
HEPÁTICA CUAL ESTUDIO DIAGNOSTICO:
FEMENINO DE 45ª QUIEN REFIERE LA RUPTURA DE VICERA HUECA
PRESENCIA DE UNA MASA A NIVEL
ABDOMINAL. NO DOLOROSA. A LA EF SE MASCULINO DE 41ª EL CUAL SUFRIO
ENCUENTRA MASA DE 2X2. POCO MOVIL, ACCIDENTE AUTOMOVILISTICO EN EL
BLANDA, NO DOLORSA, ENTRE APÉNDICE MOMENTO DE ARRIBAR A LA AMBULANCIA
XIFOIDES Y CICATRIZ UMBILICAL. EN LA USTED DETECTA QUE EL, PACIENTE SE
LINEA MEDIA, EN ESTE CASO CUAL SERIA LA ENCUENTRA SEMICONSCIENTE CON
TERAPEUTICA MAS ADECUADA: CIANOSIS PERIFERICA, LESION
PLASTIA MAXILOFACIAL IMPORTANTE.
MOVIMIENTOS RESPIRATORIOS APENAS
PERCEPTIBLES Y LA PRESENCIA DE
FRACTURA EXPUESTA DE TIBIA Y PERONE, CIRCUNSTANCIAS DESDE EL PUNTO DE
CONSIDERANDO LAS CONDICIONES EN LAS VISTA LABORATORIAL PODEMOS
QUE SE ENCUANTRA EL PACIENTE CUAL ES CORRABORAR NUESTRO DIAGNOSTICO SE
LA PRIMER MEDIDA QUE APLICARIA: LA ALBÚMINA SE ENCUANTRA DENTRO DEL
CRICOTIROIDOSTOMIA SIGUIENTE RANGO:
1.5-20G/DL
MASCULINO DE 60 AÑOS QUE ES INGRESADO
A CIRUGÍA PARA LAVADO DE CAVIDAD PACIENTE QUE SUFRIO QUEMADURA DE 2°
PERITONEAL POR SEPSIS ABDOMINAL EN QUE ABARCAN EL 30% DE LA SUPERFICIE
QUIRÓFANO. PRESENTA PARO CORPORAL EN ESTA PACIENTE EL
CARDIORRESPIRATORIO, A LA EXPLORACIÓN INCREMENTO DE LAS NECESIDADES
FÍSICA RUIDOS CARDIACOS POCO AUDIBLES, CALORICAS BASALES SERAN EN UN % DE
FRECUANCIA RESPIRATORIA NO 100-200
PERCEPTIBLE. NO HAY PULSOS EN ANTE UN PACIENTE POLITRAUMATIZADO Y
CAROTIDAS Y FEMORALES, CON ESTOS QUE FUE SOMETIDO A LAPAROTOMÍA
DATOS CLINICOS ES DETERMINANTE PARA EXPLORADORA PODEMOS ENCONTRAR UNA
EL DIAGNOSTICO DE PARO: DIFERENCIA QUE NO EXISTE EN EL PACIENTE
PERDIDA DE MOVIMIENTOS RESPIRATORIOS SOMETIDO A AYUNO CUAL ES ESTA?
ACTIVACION DEL SITEMA
MASCULINO DE 29ª PRESENTA DOLOR NEUROENDOCRINO
ABDOMINAL EN FID. CONTINUO,
ACOMPAÑADO DE HIPERTEMIA Y NAUSEAS,
A LA EXPLORACIÓN FÍSICA , MC BURNEY (+),
ROVSING (-), POR LO QUE ES PROGRAMADO
A CIRUGÍA, UNA VEZ QUE USTED SE VISTIO EN UN PACIENTE POLITRAUMATIZADO SE
CON ROPA QUIRÚRGICA PARA INGRESAR AL PRODUCE UNA ACTIVACION DEL PROBLEMA
AREA GRIS ADEMÁS USTED DEBE NEUROENDOCRINO
COLOCARSE: CUYA REPERFUSION METABÓLICA
BOTAS NUTRICIONAL PRODUCE:
ACELERACIÓN DEL CATABOLISMO PROFEICO
FEMENINO DE 35ª QUIEN VA A SER SOMETIDA MUSCULAR
A COLECISTECTOMIA POR COLECISTITIS
CRÓNICA LITIASICA SIN NINGUN MASCULINO DE 35ª EL CUAL SE ENCUENTRA
ANTECEDENTE PATOLÓGICO DE EN LA UCI, CON DIAGNOSTICO DE
IMPORTANCIA, LAS ORDENES PANCREATITIS AGUDA, DESDE HACE 10 DIAS
PREOPERATORIOS DEBERAN INCLUIR: ACTALMENTE EL ORGANISMO TIENE ALTOS
FIRMA DE LA AUTORIZACIÓN DE LA CIRUGÍA NIVELES DE INSULINA

MASCULINO DE 22ª INGRESA AL SERVICIO DE FEMENINO DE 28ª CON SEPSIS ABDOMINAL A


URGENCIAS REFIRIENDO DOLOR EN FID. EL LA EF PACIENTE SOMNOLIENTA. SIGNOS
HALLAZGO QUIRÚRGICO FUE APENDICITIS Y VITALES: 80/60, FC120X, FR36X, SU
SE REALIZA TRATAMIENTO INMEDIATO
APENDICECTOMIA EN FASE 1 (EDEMATOSA) CONSISTE EN:
AL MOMENTO DE SU EGRESO HOSPITALARIO CONTROL DE LA INFECCIÓN Y
LO CITA PARA EL RETIRO DE PUNTOS A LOS: MANTENIMIENTO DEL
7 DIAS VLUMEN SANGUÍNEO

MASCULINO DE 60ª CON DIAGNOSTICO DE MASCULINO DE 55 EL CUAL FUE SOMETIDO A


HERNIA INCICIONAL, INFRAUMBILICAL GASTRECTOMÍA SUBTOTAL POR CANCER IN
GIGANTE, SE PROGRAMA CIERRE DEL SITU. EN EL POSTOPERATORIO MEDIATO ES
DEFECTO HERNIARIO, PARA EVALUAR SU INDISPENSABLE EL USO DE ANALGÉSICOS
RIESGO QUIRÚRGICO DEBEMOS EVALUAR: CON HORARIOS, PARA PREVENIR LA
HISTORIA CLINICA SIGUIENTE COMPLICACIÓN:
ATELECTASIA
MASCULINO DE 72ª OBESO, SOMETIDO A
COLECISTECTOMIA ABIERTA POR MASCULINO DE 23ª QUIEN REFIERE DOLOR
COLECISTITIS AGUDA, AL 4TO. DIA DE POST ABDOMINAL DE PREDOMINIO EN
OPERATORIO PRESENTA EDEMA, DOLOR, EPIGASTRICO Y FID, PARA POSTERIORMENTE
HIPERTERMIA Y COLORACIÓN VIOLACEA EN IRRADIARSE A TODO EL ABDOMEN CON
MIEMBRO INFERIOR DERECHO, CUAL ES HIPERTERMIA Y VOMITOS. A LA EF EN
LA MEDIDA PREVENTIVA PARA ESTA ABDOMEN PERISTALSIS AUSENBTE CON
COMPLICACIÓN: REBOTE (+++) SE INTERVIENE DE URGENCIAS
UTILIZACIÓN DE MEDIAD ELASTICAS ENCONTRÁNDOSE APENDICITIS GRADO IV.
DURANTE EL CIERRE EL MANEJO ADECUADO
ANTE UN PACIENTE QUE SE ENCUANTRA PARA LA PIEL ES:
HOSPITALIZADO Y CURSANDO 2° DIA CIERRE DEFERIDO
POSTOPERATORIO POR COMPLICACIONESDE
PANCREATITIS NECROTICA HEMORRAGICA Y MASCULINO DE 56ª QUIEN PADECE DE
DE QUIEN SE SOSPECHA QUE CURSE CON DOLOR ABDOMINAL DESDE HACE 5ª
UNA DESNUTRICIÓN GRAVE, EN ESTAS ARDOROSO EN EPIGASTRIO, TOMANDO
ANTIÁZ PRESENTA DOLOR INTENSO REGION INGUINAL DERECHA QUE
ARRADIADO TODO EL ABDOMEN. A LA EF DESAPARECE RAPIDAMENTE AL ADOPTAR EL
PERISTALSIS AUSENTE, RESISTENCIA DECUBITO DORSAL. PARA MANEJAR ESTA
MUSCULAR CON REBOTE PRESENTE (+++) SE CONDICION EL TRATAMIENTO INDICADO EN
DECIDE PASAR A CIRUGÍA DENTRO DE LAS ESTE PACIENTE ES:
INCISIONES CUAL ES LA MÁS ADECUADA: TRATAR DE DISMINUIR LA OBESIDAD DEL
MEDIA SUPRA INFRAUMBILICAL PACIENTE YDESPUÉS OPERARLO

MASCULINO DE 22ª INGRESA A URGENCIAS MASCULINO 60ª PROSTATICO, DIABÉTICO,


REFIRIENDO DOLOR EN FID POR LO QUE SE OBESO QUIEN DESDE HACE TRES DIAS
DIAGNOSTICA APENDICITIS DE LOS EMPEZO A NOTAR UNA MASA EN REGION
SIGUIENTES EXAMENES PREOPERATORIO ANGUINAL IZQUIERDA HEMIESTERICA
CUAL SERIA DE MAYOR IMPORTANCIA: QUE DESAPARECE RAPIDAMENTE AL
EKG ADOPTAR EL DECUBITO DORSAL. POR TODAS
LAS PATOLÓGICAS QUE PRESENTA ESTE
PACIENTE DE 42ª CON HISTORIA DE PACIENTE LOS MAS PROBABLES ES
CUADROS DOLOROSOS EN CUADRANTE QUE LA DEFORMACIÓN MENCIONADA
SUPERIOR DERECHO ACOMPAÑADOS DE CORRESPONDA:
NAUSEAS Y VOMITOS DE 2 AÑOS DE HERNIA INGUINAL DIRECTA
EVOLUCION ACTUELMENTE CON COLURIA,
ICTERICIA Y ACOLLA. QUE ESTUDIO SE USA MASCULINO DE 67ª CON DIAGNOCTICO DE
PARA CONFIRMAR EL DIAGNOSTICO: HERNIA INGUINAL DERECHA DENTRO DE
ULTRASONOGRAFIA LOS ESTUDIOS PARACLINICOS
PREOPERATORIOS. LOS MAS IMPORTANTES
MASCULINO DE 30ª QUIEN PRESENTA DESDE EN EL CASO SON:
HACE 3 AÑOS DOLOR EPIGASTRICO TELE DE TORAX Y ELEC TROCARDIOGRAMA
RECURRENTE, EL QUE SE PRESENTA
COMÚNMENTE POR LA MADRUGADA. DESDE
HACE 6 HORAS PRESENTA HEMATEMESIS, SU EN UN PACIENTE QUEMADO DE 7OKG DE
DIAGNOSTICO ORIENTA HACIA: PESO CON 30% DE SUPERFICIE QUEMADA DE
ULCERA PEPTICA SEGUNDO GRADO, NECESITARA DURANTE
SUS PRIMERAS 8 HORAS LA SIGUIENTE
EN UN PACIENTE CON ULCERA PEPTICA CANTIDAD DE SOLUCION (PARKLAND)
PERFORADA, DENTRO DE LA EF A LA 12,000ML
PERCUSIÓN ESPERARIA ENCONTRAR:
DESAPACRICION DE LA MATIDEZ HEPÁTICA MASCULINO DE 45ª QUEMADO POR
INTOXICACIÓN CON CO2 QUIEN PRESENTA
FEMENINO DE 45ª REFIERE LA PRESENCIA DE DISNEA Y QUEMADURAS EN CARA LAS
UNA MASA A NIVEL ABDOMINAL NO CUALES NO SON DOLOROSAS, EL PACIENTE
DOLOROSA. A LA EF SE ENCUANTRA MASA DEBERA HABER PRESENTADO POR LO
DE APROXIMIDAMENTE 2X2CM. POCO MENOS UNO DE LOS SIGUIENTES DATOS A LA
MOVIL, BLANDA NO DOLOROSA, ENTRE EXPLORACIÓN FÍSICA:
APÉNDICE, XIFOIDES Y CICATRIZ UMBILICAL CAPA CUBIERTA CON HOLLÍN NEGRO
EN LA LINEA MEDICA, CUAL SERIA LA
TERAPEUTICA:
PLASTIA FEMENINO 48ª QUE VA ASER SOMETIDA A
HISTERECTOMÍA POR PRESENTAR CACU LA
MASCULINO DE 60ª QUE PRESENTA UNA SIGUIENTE MEDIDA ANTITROMBOTICA ES:
BOLA EN REGION INGUINAL DERECHA QUE MEDIAS ELASTICAS
SE DESAPARECE CON EL REPOSO, NO
DOLOROSA QUE NO LLEGA AL ESCROTO, A FEMENINO 40ª CON DIAGNOSTICO DE
LA EF LA MASA TOCA EL PULPEJO DEL MIOMATOSIS UTERINA POR LO QUE SE VA A
DEDO EXPLORADOR A TRAVEZ DEL PISO PRACTICAR HITERECTOMIA TOTAL
INGUINAL EN LA REPARACIÓN QUIRÚRGICA ABDOMINAL, NO CUENTA CON
DE ESTE DEFECTO. CUAL ES LA SUTURA MAS ANTECEDENTES DE IMPORTANCIA, POR LO
ADECUADA: CUAL ESTARA CLASIFICADA CON UN RIESGO
PROLENE ANESTESICO QUIRÚRGICO CLASE:
I
MASCULINO DE 65ª EL CUAL PRESENTA
HERNIA INGUINAL DIRECTA. EL MANEJO DE MASCULINO DE 60ª EL CUALA VA A SER
ESTA CONDICION PUEDE HACERSE POR SOMETIDO A RESECCION TRANSURETRAL DE
CUALQUIERA DE LAS SIGUIENTES VIAS: PRÓSTATA. COMO ANTECEDENTE CUANTA
VIA RETROPERITONEAL CON INFARTO AL MIOCARDIO
VIA PERITONEAL HACE 4 MESES SE CATALOGA COMO RIESGO
VIA PERITROESCOPIA ANESTESICO QUIRÚRGICO CLASE:
VIA TRACTO ILEO PURICO IV

MASCULINO DE 67ª PROSTATICO, DIABÉTICO, FEMENINO DE 23ª LA CUAL SE ENCUANTRA


OBESO Y CON EPOG, QUIEN HACE UN MES EN URGENCIAS CON DIATROSIS, TA90/50,
EMPEZO A NOTAR MASA TUMORAL EN FR24X´, FC100X´, TEMP 38°, A LA CUAL SE LE
DIAGNOSTICO TROMBOSIS MESENTERICA, EN UN PACIENTE QUE SUFRE HERIDA POR
COMO ANTECEDENTE EL PACIENTE ES ARMA BLANCA EN TORAX IZQUIERDO. CON
DIABÉTICO DESDE HACE 10 AÑOS TA90/50. INGURGITACION YUGULAR POR LO
MANEJADO CON HIPOGLUCEMIANTES QU SE LE REALIZO PERICARDIOCENTESIS DE
ORALES SEGÚN LA CLASIFICACION DE ASA. URGENCIAS. ELTRATAMIENTO CONSISTE EN:
SU RIESGO QUIRÚRGICO SERA DE: TORACOTOMIA Y MIOCARDIOGRAMA
V
MASCULINO DE 30ª QUE SUFRE ACCIDENTE
MASCULINO DE 34ª EL CUAL SUFRIO AUTOMOVILISTICO POR LO QUE ES TRAIDO A
ACCIDENTE AUTOMOVILISTICO EN URGENCIAS, ENCONTRÁNDOSELE
CARRETERA. AL MOMENTO DE ARRIBAR SEMIINCONSCIENTE. HIPOTENSO,
CON LA AMBULANCIA USTED DETECTA QUE CIANOTICO. EN CUALLO INGURGITACION
EL PACIENTE SE ENCUENTRA YUGULAR CON GRACTURA DE ARCOS
SEMICONSCIENTE, SIGNOS VITALES: COSTALES EN HEMITORAX IZQUIERDO SE LE
TA110/70, FR90X´, FC88X´, CIANOSIS TOMA LA P.V.C. LA CUAL ESTA ELEVADA.
PERIFERICA , LESION MAXILOFACIAL CUAL ES EL DIAGNOSTICO:
IMPORTANTE, MOVIMIENTOS HEMOPERICARDIO
RESPIRATORIOS SUPERFICIALES Y LA
PRESENCIA DE FRACTURA EXPUESTA DE PACIENTE DE 28ª POLITRAUMATIZADO EN
TIBIA. CUAL ES LA CONDUCTA MAS ACCIDENTE POR MOTOCICLETA NO
IMPORTANTE QUE DEBE REALIZAR: LLEVANDO CASCO PROTECTOR ES ADMITIDO
CRICOTIROIDECTOMIA A URGENCIAS CON CIANOSIS Y SEVERA
DIFICULTAD RESPIRATORIA, TA80/40 Y
SANGRADO NASAL. ADEMÁS DE FRACTURA
MASCULINO DE 44ª EL CUAL SUFRIO EXPUESTA DE FÉMUR RUIDOS
ACCIDENTE AUTOMOVILISTICO AL RESPIRATORIOS AUSENTES EN HEMITORAX
MOMENTO DE ARRIBAR EN LA AMBULANCIA DERECHO. LA PRIORIDAD DEL MENEJO SERA:
USTED DETECTA QUE EL PACIENTE SE OBTENER ACCESO IV PARA TRANSFUSIÓN DE
ENCUENTRA SEMICONSCIENTE, CON EMERGENCIA DE SANGRE TIPO O
CIANOSIS PERIFERICA, LESION
MAXILOFACIAL IMPORTANTE, EN UN PACIENTE POLITRAUMATIZADO AL
MOVIMIENTOS RESPIRATORIOS APENAS CUAL EN LA EF SE REPORTA TA 100/60, FR32X,
PERCEPTIBLES Y LA PRESENCIA DE ABDOMEN DISTENDIDO. SILENCIO
FRACTURA EXPUESTA DE LA TIBIA ABDOMINAL , REBOTE (++), TIMPANISMO
Y PERONE. CUAL ES LA MEDIDA QUE GENERALIZADO. QUE ESTUDIO CORROBORA
APLICARIA: EL DIAGNOCTICO:
COLOCAION DE LA CANULA DE GEDELE TELE DE TORAX

PACIENTE QUE SUFRE ACCIDENTE


FEMENINO DE 27ª QUIEN A SUFRIDO CAIDA AUTOMOVILISTICO Y QUE AL EXAMEN SE
DE LESION DE BAZO, HIGADO, ASI COMO DE ENCUANTRA DESPIERTO Y RESPONDE A LAS
INTESTINO DELGADO, SE TRANSFUENDEN PREGUNTAS, A NIVEL DE TORAX SE
MAS DE 10 U DE SANGRE SI EL SANGRADO ENCUANTRAN DATOS QUE SUGIEREN
PERSISTIERA A PESAR DEL SUFIECINTE NEUMOTORAX A TENSIÓN EN HEMITORAX
REMPLAZO DE COAGULANTES SE PENSARIA DERECHO, LA MEDIDA A APLICAR EN ESTE
EN LA PRESENCIA ACTUAL DE: PROBLEMA SERA: TORACOCENTESIS EN 2°
COAGULACIÓN INTRAVASCULAR ESPACIO INTERCOSTAL
DISEMINADA
MASCULINO QUIEN EN SALA DE
OPERACIONES PREVIA A LA INTUBACIÓN,
FEMNINO DE 15ª QUIEN ES ASALTADA EN LA PRESENTA PARO CARDIORRESPIRATORIO SUS
VIA PUBLICA, A LA EF SE ENCUENTRA PULSOS CENTRALES SONDEBILES, SU TA
DISEÑA, INQUIETUD, DOLOR, FR40X 60/50. FRECUENCIA RESPIRATORIA
SILENCIO RESPIRATORIO Y CLARIDAD SUPERFICIAL, DADA LA BRADICARDIA DEL
PULMONAR AUMENTADA EN LA RX DE PACIENTE, CUAL ES EL MEDICAMENTO
TORAX EL DIAGNOSTICO ES: USADO PARA SU MANEJO:
NEUMOTORAX A TENSIÓN ADRENALINA ENDOTRAQUEAL

MASCULINO DE 29ª QUE CONDICIENDO SU MASCULINO DE 38ª QUIEN SE RECIBE EN


AUTOMOVILA ALTA VELOCIDAD SUFRE URGENCIAS CON ANTECEDENTES DE
CHOQUE, Y A LA EF SE ENCUANTRA CON TA LSIONES EN TORAX Y ABDOMEN NO
100/60, EC100X´, FR24X, EN EL RESTO DEL PENETRANTES. ASI COMO FRACTURAS
EXAMEN EN HEMITORAX DERECHO MULTIPLES EN EXTREMIDADES INFERIORES
MURMULLO VESIDULAR DESMINUIDO CON A SU LLEGADA TA 60/40 RUIDOS CARDIACOS
MATIDEZ A LA PERSECUCIÓN. CUAL SERIA EL POCO PERCEPTIBLES, RESPIRACIÓN
TRATAMIENTO TERAPÉUTICO PARA SU SUPERFICIAL, PULSOS EN FEMORAL Y EN
PROBLEMA PULMONAR: PUNCION CON CAROTIDA AUSENTES SE DIAGNOSTICA
TROCAR EN 6° ESPACIO INTERCOSTAL MENIOBRA DE COMPRESIÓN EXTERNA Y
DESFIBRILACION, PERSISTE CON ARRITMIAS
VENTRICULARES. CUAL ES LA DOSIS DE
LIDOCAINA INDICADA PARA EL PERCEPTIBLE, QUE DATO ES DETERMINANTE
TRATAMIENTO: PARA HACER EL DIAGNOSTICO:
BOLO IMG/ KG IV PERDIDA DEL PULSO EN UNA ARTERIA
CENTRAL (CAROTIDA O FEMORAL)
MASCULINO DE 50ª QUIEN INGRESA AL
SERVICIO DE URGENCIAS CON FEMENINO DE 42ª QUIEN SE LE REALIZO
ANTECEDENTE DE HABER SUFRIDO COLECISTECTOMIA ABIERTA POR
ACCIDENTE AUTOMOVILISTICO, A SU COLECISTITIS CRÓNICA LITIASICA AL SALIR
LLEGADA PRESENTA DESORIENTACIÓN, A SU DE QUIRÓFANO TIENE COLOCADA SONDA
LLEGADA PRESENTA DESORIENTACIÓN, TA NASOGASTRICA, SONDA VESICAL
50/40, FRECUENCIA CARDIACA NO AUDIBLE VENOCLISIS CON HARTMAN GLUCOSADA,
SIN RESPIRACIÓN ESPONTÁNEA, PULSOS EN RESPIRACIÓN DE 18X´, PULSO 75X´, TA 130/85,
CAROTIDAS AUSENTES. POR LO QUE SE ESTA PACIENTE DEBERA DE AMBULAR:
DIAGNOSTICA PARO AL DIA SIGUIENTE DE LA CIRUGÍA
CARDIORRESPIRATORIO. SE INICIAN
MANIOBRAS DE REANIMACION CON
COMPRESIONES TORAXICAS Y
RESPIRACIONES, EN EL MONITOR SE
OBSERVA FIBRILACION VENTRICULAR, CUAL PACIENTE DE 22ª ACUDE A URGENCIAS
ES EL SIGUIENTE PASO PAR CONTINUAR CON REFIRIENDO DOLOR EN FID SE DIAGNOSTICA
EL TRATAMIENTO: APENDICITIS Y SE REALIZA
INICIAR DESFIBRILACION E INTUBAR AL APENDICECTOMIA. EL HALLAZGO
PACIENTE QUIRÚRGICO FUE APENDICITIS EN FASE
(EDEMATOSA) 1, LO CITA PARA RETIROS DE
FEMENINO DE 55ª CON CHOQUE SÉPTICO Y PUNTOS A LOS 7 DIAS
DISFUNCIÓN ORGANICA MÚLTIPLE. A LA EF
PACIENTE SOMNOLIENTO, RUIDOS MASCULINO DE 22ª SOMETIDO A LA
CARDIACOS DISMINUIDOS, FRECUENCIA LAPAROTOMÍA CON HALLAZGO DE
RESPIRATORIA NO PERCEPTIBLE PULSOS PERFORACIÓN DEL ILEON DISTAL POR
CENTRALES AUSENTES, SE HACE SALMONELLOSIS, AL 3ER. DIA
DIAGNOSTICO DE PARO POSTOPERATORIO INICIA CON FIEBRE,
CARDIORRESPIRATORIO, SE INICIA MASAJE DOLOR, TUMEFACCIÓN, HIPEREMIA Y
CARDIACO EXTERNO LA META AUMENTO DE LA TEMPERATURA LOCAL, LA
FUNDAMENTAL EN TODA SESION DE CAUSA PROBABLE DE ESTA COMPLICAION
REANIMACION CON LA COMPRESIÓN ES: FALTA DE COLOCAION DE DRENES EN LA
ES: RESTAURAR LA FUNCION CARDIACA CAVIDAD
NORMAL LO MAS PRONTO POSIBLE
MASCULINO DE 30ª POSTOPERATORIO DE
FEMENINO DE 65ª QUIEN PRESENTA SEPSIS HERNIOPLASTIA INGUINAL DERECHA
ABDOMINALSECUNDARIA DIVERTICULITIS INDICADA EN EL 2° DIA POSTOPERATORIO
COLONICA COMPLICADA, SE REALIZA PRESENTA AUMENTO DE VOLUMEN DE LA
LAVADO Y DRENAJE DE LA CAVIDAD, EN EL HERIDA CON FLUCTUACIÓN, NO DOLOROSA,
POST OPERATORIO LA PACIENTE NO SE OBSERVAN DATOS DE CELULITIS NI
CARDIOPULMONAR LA CUAL ES EFECTIVA HIPERTERMIA CON ESTOS DATOS LA
USTED INDICA QUE LA PACIENTE ES COMPLICAION QUE PRESENTA EL PACIENTE
TRASLADADA A: SU CAMA CON CONTROL ES: DEHISCENCIA
ESRICTO DE SIGNOS VITALES
PACIENTE DE 50ª CON CANCER DE ESÓFAGO
FEMENINO DE 60ª QUIEN INGRESA A Y CON INANICIÓN EN ESTAS
URGENCIAS POR ACCIDENTE CIRCUNSTANCIAS SU CORAZON Y
AUTOMOVILISTICO AL LLEGAR AL HOSPITAL MUSCULOS TIENEN PREDILECCIÓN POR UNO
SUS SIGNOS VITALES SON: TA60/40, RUIDOS DE LOS SIGUIENTES AMINOÁCIDOS:
CARDIACOS POCO PERCEPTIBLES, DE CADENA RAMIFICADA
RESPIRACIÓN SUPERFICIAL, MINUTOS
DESPUÉS DE SU LLEGADA AL MASCULINO DE 36ª QUE DURANTE LA
SERVICIO DE URGENCIAS PRESENTA PARO INANICIÓN AGUDA DEBIDO A CANCER
CARDIACO RESPIRATORIO, CUAL ES LA LARINGEO TIENE UNA PERDIDA NETA DE
MEDIDA QUE INICIO PARA RESTAURAR LA AMINOÁCIDOS YA QUE LAS SÍNTESIS
FUNCION CARDIACA: AUMENTAR EL DE PROTEINA MUSCULAR DESAPARECE O
VOLUMEN SANGUÍNEO POR MEDIO DE DESCIENDE TANTO QUE EL CATABOLISMO
SOLUCIONES DE LA MISMA PERMANECE SIN CAMBIO
ESTA SITUACIÓN ES DEBIDA A:
FEMENINO DE 60ª QUIEN ES INGRESADA A AUMENTO DE GLUCAGON
CIRUGÍA PARA REALIZAR LAVADO DE
CAVUDAD POR SEPSIS PERITONEAL, EN EN UN PACIENTE POLITRAUMATIZADO A
QUIRÓFANO PRESENTA PARO QUIEN EN EL EXAMEN ENCONTRAMOS EN
CARDIORRESPIRATORIO A LA EXPLORACIÓN HEMOTÓRAX IZQUIERDO SILENCIO
RUIDOS CARDIACOS POCO AUDIBLES, RESPIRATORIO, A LA PERCUSIÓN
FRECUENCIA RESPIRATORIA NO TIMPANISMO Y CON CHOQUE DE PUNTA A LA
DERECHA. REALIZÁNDOSELE
TORACOCENTESIS DE URGENCIAS, LA ACOMPAÑADA DE HIPEREMIA Y NAUSEAS, A
JUSTIFICACIÓN DE ESTA MEDIDA ES POR: LA EXPLORACIÓN ROVSING (+) Y BLUNBERG
LA URGENCIA DE DESCOMPRIMIR AL (+). EN EL PUNTO DE MC BURNEY PARA EL
MEDIASTINO ASEO QUIRÚRGICO DE SUS MANOS, CUAL DE
LAS SIGUIENTES SUSTANCIAS
MASCULINO DE 34ª LLEVADO A URGENCIAS ANTISÉPTICAS UTILIZARA:
POR SUFRIR ACCIDENTE AUTOMOVILISTICO HI-BISCRUB (GLUCONATO DE
AL EXMAMEN ENCONTRAMOS CLORHEXIDINA)
HIPERSENSIBILIDAD LEVE EN EL ABDOMEN
Y HEMITORAX DERECHO. CON PIRISTALSIS EN EL PISO DE CIRUGÍA, SE DISCUTE EL
MUY DISMINUIDA Y AUSENCIA DE MATIDEZ CASO DE UN PACIENTE QUE PRESENTO
HEPÁTICA; CUAL ES SU IMPRESIÓN: DESHICENCIA DE LA HERIDA QUIRÚRGICA
RUPTURA DE VICERA HUECA EN EL 5° DIA DEL POSTOPERATORIO SE
HACE MENCIONAR QUE FUE DEBIDO A MALA
FEMNINO DE 22ª QUIEN VA A SER SOMETIDO TÉCNICA QUIRÚRGICA EN EL CIERRE DE LA
A LAPAROSCOPIA PÉLVICA POR PROBABLE HERIDA ESTA COMPLICACIÓN IMPLICA:
ENDOMETRIOSIS DURANTE LA ASEPSIA DEL SEPARACIÓN PARCIAL O TOTAL DE LOS
AREA QUIRÚRGICA (PIEL) CUAL DE LAS PLANOS DE LA HERIDA
SIGUIENTES SUSTANCIAS CONSIDERA QUE
ES LA MAS ADECUADA PARA RELIZAR ESTE FEMENINO DE 48ª QUIEN INGRESA A
PROCEDIMIENTO: URGENCIAS CON CUADRO DE ABDOMEN
YODOPOVINA (ISODINE) AGUDO SECUNDARIO A COLELITITIS
COMPLICADA. A LA EXPLORACIÓN
ENCUANTRA TALLA 1.60M PESO DE 92KG TA
FEMENINO DE 38ª QUE SE ENCUENTRA EN EL 140/90 DOLOR A LA PALPACIÓN EN
5TO DIA POSTOPERATORIO POR COLECTOMIA HIPOCONDRIO DERECHO. MURPHY (+)
TOTAL PARA EL TRATAMIENTO DE SU REBOTE POSITIVO, QUE INSICION SE
COLECISTITIS ULCERATIVA CRÓNICA RECOMIENDA: SUPRA INFRAUMBILICAL
INESPECÍFICA, LA CIRUGÍA SE EFECTUA SIN
COMPLICACIONES CERRANDO LAS CAPAS DE MASCULINO DE 28ª QUIEN FUE SOMETIDO A
LA HERIDA CON SURGENTE, COMO APENDICECTOMIA ENCONTRANDO UNA
ANTECEDENTES DE IMPORTANCIA APÉNDICE CECAL GANGRENADA. ESTE
MENCIONA SER DIABÉTICA TIPO 1, EN EL ANTECEDENTE NOS PERMITE CLASIFICAR LA
2DO DIA POSTOPERATORIO PRESENTO HERIDA COMO: SUCIA
ACUMULO DE LA SANGRE EN LA HERIDA Y
HOY OBSERVA EVICERACION.
LA CAUSA MAS PROBABLE DE LA MASCULINO DE 36ª SOMETIDO A
COMPLICACIÓN ES: HERNOPLASTIA CON RESECCION DE UN
INFECCIÓN DE LA HERIDA DIVERTICULO DE MECKEL A TRAVEZ
DEL ANILLO HERNIARIO EN EL
FEMENINO DE 17 AÑOS CON DIAGNOSTICO POSTOPERATORIO PRESENTO ABSCESO EN
DE PÚRPURA TROMBOCITOPENICA LA HERIDA QUIRÚRGICA EL MANEJO DE
IDIOPATICA REFRACTARIA A TRATAMIENTO ESTA COMPLICACION SE REALIZA CON
MEDICO, PROGRAMADA PARA DRENAJE Y CIERE: DIFERIDO
ESPLENECTOMIA. DESPUÉS DE LA ASEPSIA Y
ANTISEPSIA DEL AREA QUIRÚRGICA, LA MASCULINO DE 19ª CON APENDICITIS AGUDA
COLOCACIÓN DE CAMPOS Y SABANAS SE NO COMPLICADA CUAL ES LA INCISIÓN MÁS
INDICA CON: SABANA PODALICA CONVENIENTE PARA ESTE PACIENTE:
ROCKY DAVIS
FEMENINO DE 40ª QUIEN PRESENTA DOLOR
ABDOMINAL EN ABDOMEN SUPERIOR Y QUE CUAL ES LA HORMONA QUE DISMINUYE EN
SE LE EXACERVA CON LOS ALIMENTOS LA RESPUESTA METABÓLICA AL TRAUMA:
GRASOS. A LA EXPLORACIÓN MURPHY (+), INSULINA
EL ULTRASONIDO DE HIGADO Y VIAS
BILIARES REVELA VESÍCULA DE 11X9CM, E MASCULINO DE 53ª POLITRAUMATIZADO
IMÁGENES HIPERECOGENICAS EN SU POR ACCIDENTE AUTOMOVILISTICO EN ESTA
INTERIOR, PARA LA CIRUGÍA EN QUE AREA PACIENTE LA RESPUESTA METABÓLICA AL
SE VISTE CON LA ROPA QUIRÚRGICA: TRAUMA PROVOCA
NEGRA LIBERACIÓN DE ADRENALINA, LA QUE
PROVOCARA:
MASCULINO DE 71ª QUIEN FUE SOMETIDO A ESTIMULA LA LIPÓLISIS
COLECISTECTOMIA CON INCISIÓN MEDIA
SUPRAUMBILICAL EN EL 5TO DIA DEL PACIENTE POLITRAUMATIZADO EL CUAL
POSTOPERATORIO PRESENTO ENCUENTRA EN LA UCI DEVEMOS TENER EN
EVISCERACION. LA CAUSA MÁS PROBABLE CUENTA QUE DURANTE LA RESPUESTA
DE ESTA COMPLICACION ES: METABÓLICA AL TRAUMA LA ACCION
TÉCNICA QUIRÚRGICA INADECUADA DE LAS HORMONAS
CONTRARREGULADORAS TIENE
MASCULINO DE 22ª PRESENTA DOLOR COMO RESPUESTA EN EL ORGANISMO
ABDOMINAL EN FID CONTINUO, AGREDIDO:
AUMENTO DE LA GLUCONEOGENESIS COMPLICACIONES PULMONARES
POSTOPERATORIAS:
FEMENINO DE 28ª LA CUAL PRESENTO EJERCICIOS RESPIRATORIOS MINIMO 48
TRAUMA ABDOMINAL CERRADO CON HORAS ANTES DE LA CIRUGÍA
RUPTURA HEPÁTICA E ISQUEMIA
INTESTINAL POR DESGARRO DEL PACIENTE POLITRAUMATIZADO MANTENIDO
MESETERIO DURANTE UNA INTERVENCIÓN CON NUTRICION PALENTERAL TOTAL CON
QUIRÚRGICA, PRESENTA SANGRADO DIFUSO UNA FORMULA EQUILIBRADA, PERO QUE NO
DE LA PARED ABDOMINAL Y EN HIGADO ES SUFICIENTE PARA
DEBIDO A UN PROBABLE CID, EL CUBRIR SUS REQUERIMIENTOS
TRATAMIENTO QUE SE RELIZARA ENERGÉTICOS PARA INCREMENTAR EL
EN ESTE MOMENTO SERA: APORTE ENERGÉTICO SE CONSIDERA
EMPAQUETAMIENTO ABDOMINAL A: AUMENTAR LA FRACCIÓN DE GLUCOSA
DE LA FORMULA
PACIENTE EL CUAL FUE SOMETIDO A
HEMICOLECTOMIA POR CANCER DE COLON MASCULINO DE 28ª PROVENIENTE DE LA UCI
EN EL POSTOPERATORIO PRESENTA CUADRO DONDE FUE SOMETIDO A UNA
COMPATIBLE CON CID LA ALTERACIÓN LAPAROTOMÍA EXPLORADORA POR HABER
LABORATORIAL QUE ESPERARIA SUFRIDO ACCIDENTE AUTOMOVILISTICO
ENCONTRAR SERA: MENCIONÁNDOSE QUE SE LE REALIZO
ANTITROMBINA III DISEMINADA RESECCION INTESTINAL DE 40CM DE
INTESTINO DELGADO POR PRESENTAR
FEMENINO DE 28ª CON SEPSIS ABDOMINAL Y ISQUEMIA INTESTINAL SECUNDARIA A
A LA EXPLORACIÓN FÍSICA PACIENTE DESGARRO MESENTERICO POR
SOMNOLIENTA SIGNOS VITALES TA 80/60, FC DESACELERACION LOS RECEPTORES QUE
110X, EN EL CHOQUE SÉPTICO. LA ESTIMULAN LA LIBERACIÓN DE
HIPERVENTILIACION SE ENCUANTRA EN LA CORTICOTROPA EN ESTE PACIENTE SE
FASE: ENCUANTRAN LOCALIZADOS EN:
TEMPRANA PROTUBERANCIA Y MENSENCEFALO

FEMENINO DE 32ª QUIEN REFIERE UNA MASA ANTE UN PACIENTE QUE FUE SOMETIDO A
BLANDA, NO DEPRESIBLE, FIJA E INDOLORA DRENAJE ABIERTO DE ABSCESO HEPÁTICO
Y POR ARRIBA DE LA CICATRIZ UMBILICAL. AMIBIANO, EL CUAL SE ENCUANTRA EN LAS
EN ESTE CASO EL DEFECTO SE ENCUANTRA PRIMERAS HORAS POSTOPERATORIO FASE
EN LA: EBB, USTED ESPERARIA ENCONTRAR UN
LINEA ALBA AUMENTO DE LA SIGUIENTE SUSTANCIA
PRODUCIDA POR EL ORGANISMO. CUAL ES:
DURANTE UNA COLECISTECTOMIA ABIERTA TIROXINA
ACCIDENTALMENTE SE LE CAEN LAS
TIJERAS. LAS CUALES SON MASCULINO DE 35ª EL CUAL SE ENCUENTRA
IMPRESCINDIBLES PARA RELIZAR LA EN LA UCI CON DIAGNOSTICO DE
CIRUGÍA, NECESITAN QUE LAS TIJERAS SE PANCREATITIS AGUDA DESDE
ESTERILICEN QUE SUSTANCIA PERMITE HACE 10 DIAS ACTUALMENTE EL
REALIZAR ESTE PROCEDIMIENTO ORGANISMO TIENE: NECESIDAD DE
RAPIDAMENTE: CONSUMIR CETONAS
GLUTERHALDEHIDO ACTIVADO
*ANTE UN PACIENTE QUE EN EL
MASCULINO DE 18ª CON HERIDA PRODUCIDA POSTOPERATORIO CURSO CON
POR OBJETO CORTANTE EN REGION DESEQUILIBRIO HÍDRICO PODEMOS INTUIR
FRONTAL DE LA CARA DE 3CM APROX DE QUE EL AGUA INTRACELULAR PUEDE
LONGITUD, QUE TECNICA DE SUTURA DISMINUIR DEVIDO A UNA DE LAS
USARA PARA CERRAR LA PIEL Y TENER SIGUIENTES ALTERACIONES CUAL ES ESTA:
UN RESULTADO MAS ESTETICO: REDUCCIÓN DEL POTASIO INTRACELULAR
SUTURA SUBCUTICULAR

INGRESA PACIENTE A URGENCIAS CON


HERIDA EN LA CARA ANTERIOR DE FEMENINO DE 25 AÑOS QUIEN SUFRIO
ANTEBRAZO DE APROXIMADAMENTE 3CM, ACCIDENTE AUTOMOVILISTICO CON
DE LONGITUD, REALIZA EL ASEO DE LA LESIONES LEVES, PERO FUE VICTIMA DE
HERIDA DE LA PIEL CON ISODINE, CUAL UNA CRISIS EMOCIONAL, SE LE TOMA
ES EL TIEMPO MINIMO NECESARIO PARA GASOMETRIA REVELANDO UNA PCO2 DE
OBTENER EL EFECTO ESPERADO: 30MMHG, TAL CIFRA Y EL CUADRO CLINICO
10 MINUTOS SUGIEREN: ACIDOSIS RESPIRATORIA

PACIENTE DE 58ª PROGRAMADO PARA MASCULINO DE 35ª QUIEN ES RESCATADO DE


GASTRECTOMÍA SUBTOTAL POR CANCER ACCIDENTE AUTOMOVILISTICO Y QUE
GÁSTRICO EN ETAPA TEMPRANA COMO PRESENTA FRACTURAS MULTIPLES Y
ANTECEDENTE MECIONA PADECER EXPUESTAS EN MIEMBRO SUPERIOR
ENFERMEDAD PULMONAR. DE LAS DERECHO OCACIONANDO LESION ARTERIAL.
SIGUIENTES RESPUESTAS CUAL EVITA LAS AL INICIARSE COMO PRIMER EVENTO LA
VASOCONSTRICCIÓN SE ACTIVA EL
SIGUIENTE ELEMENTO:
MICROAGREGADO PLAQUETARIO
- Almidón - Traqueostomía
- Lípidos - Tomar nueva gasometría para confirmar
27.Px femenina de 39 años a la cual se realizó endoscopia por referir 32.Px portador de enfermedad diverticular colónica. Ha presentado
reflujo gastro-esofágico y en el reporte endoscopio se describe la varios episodios de agudeza; si presenta neumaturia, ¿Qué problema
presencia de bilis ¿Cual es la interpretación clínica? presenta? ”FÍSTULA”
- Importante valorar si tiene hernia hiatal - Colón-vesical
- Se requiere de manometría - Colón-ileón
- Presenta incompetencia del esfínter de Oddi - Colón-sigmoideo
- No es patológico - Colon-útero
- Es portadora de reflujo alcalino - Sigmoides e íleon
28.Px con trauma abdominal por objeto punzo cortante. Signo clínico de
33.Masculino de 80 con perforación por proceso ulceroso prepilorico de
aire libre en cavidad peritoneal
24 h. ¿Qué proceso Qx haría si el px presenta datos de choque mixto y
- Cullen comórbidos como DM y neumopatía por tabaquismo?
- Jaubert - Laparotomía exploradora, cierre de perforación con vycril 1
- Grey Turner - Laparotomía de cierre de perforación con crómico, vagotomía
- Morris troncular, drenaje de cavidad
- Murphy - Laparoscopia cierre de perforación y no drenajes
29. Px masculino de 55 años, el cual tiene como antecedente sangrado
- Laparoscopia aplicación de parche de epiplón, lavado de cavidad,
vagotomía troncular y drenaje
rectal en 3 ocasiones el día de hoy se presenta en urgencias por
cuadro doloroso abdominal. Al explorarlo encuentra que se palpa
- Laparotomía cierre de perforación con seda 2-0, lavado de cavidad
y drenajes.
zona indurada en región de sigmoides con dolor importante en esta
área. La temperatura es de 39.5 grados. ¿Cuál sería el dx más
probable? DUDA
- Cáncer de colon
- apendicitis de localización anómala
- colon toxico
- Sx de colon irritable
- Diverticulitis de colon perforado y sellado
30.Portador de Hernia ventral con evolución de 3 años será sometido a
plastia de pared; el defecto mide 12x10 cm. Opción adecuada
- Malla pre-peritoneal
- Puntos seprados con prolene
- Malla a tensión
- Técnica de Marcy
- Técnica de Bassini
31.Se encuentra pasando visita y uno de los px presenta dificultad
respiratoria cianosis peribucal, el estudio gasométrico revela CO2 50,
O2 50. ¿Qué acción deberá realizar de inmediato?
- Intubación
- Solicitar sodio y cloro para valorar A/G
- Oxigeno con puntas nasales
28 de 28

También podría gustarte